PDA

نسخه کامل مشاهده نسخه کامل : اتاق حل مساله و روش های آن - دوره ی سوم



صفحه ها : [1] 2 3

mofidy1
16-07-2009, 17:22
بسم الله الرحمن الرحیم


افتتاح اتاق حل مساله (سال سوم)

با سلام به دوستان عزیزم در انجمن ریاضیات
بنابر قولی که در آخرین پست اتاق حل مساله ([ برای مشاهده لینک ، لطفا با نام کاربری خود وارد شوید یا ثبت نام کنید ])-سال دوم -به کاربران محترم سایت دادیم، اتاق حل مساله ی امسال را با روش و ترکیبی جدید آغاز می کنیم. برای آشنایی بیشتر با فعالیت های امسال اتاق، مطالب زیر را مطالعه فرمایید:

1- در این دوره با توجه به تجربیات دو سال گذشته قرار شد که اتاق، قائم به شخص نباشد و فقط یک نفر در آن مدیریت نکند.بنابر این ازپنج نفر از دوستان فعال در سایت دعوت به همکاری کردیم (به شرح ذیل):

روزهای شنبه تا پنج شنبه، هر روز یک مساله در سایت به وسیله ی کاربران زیر - که به عنوان مدیران اتاق حل مساله خواهیم شناخت - ارائه خواهد شد:

روزهای شنبه mir@
روزهای یکشنبه ali_hp
روزهای دوشنبه yugioh
روزهای سه شنبه sherlockholmz
روزهای چهارشنبه eh_mn
روزهای پنج شنبه mofidy1

هر کدام ازاین مدیران، در روز مخصوص به خودشان، به حل مساله ای که هفته ی قبل طرح کرده اند، خواهند پرداخت و اگر در طول هفته روی مسائل مطرح شده، حل و بحثی از طرف کاربران موجود باشد، مدیر مربوط به هر مساله، میزان درستی یا نادرستی آن را مشخص می کند. بعد از حل و بحث مساله ی هفته ی قبل، مدیرآن روز، مساله ی جدید مربوط به خودش را طرح خواهد کرد.

2-با هماهنگی مدیران اتاق حل مساله، سوالات هفته کاملا متنوع طرح خواهند شد تا همه ی کابران سایت از اول دبیرستان تا سطح دانشگاه بتوانند در طول هفته از مسائل مربوط به خودشان استفاده کنند.

3- با اتمام 52 هفته، سعی می کنیم تمام مسائل مطرح شده را در طول سال، همراه با حل آن ها، در یک فایل جمع آوری و خدمت دوستان تقدیم کنیم.

4- روزهای جمعه نیز این حقیرسعی می کنم مطالبی از روش های حل مساله ، المپیادهای ریاضی دانش آموزی و دانش جویی امسال و سال های گذشته در داخل و خارج از کشور ، لینک های مفید از سایت هایی که به حل مساله می پردازند و نیز لینک های المپیادهای ریاضی بین المللی و المپیادهای ریاضی کشورهای خاص به اتاق حل مساله اضافه کنم. البته ازمدیران اتاق نیز برای تکمیل این مطالب دعوت به همکاری شده است. از کاربران دبگر این اتاق نیز درخواست همکاری دراین زمینه ها داریم.

5- در آخر دوره، نفرات برتر اتاق حل مساله را - همانند سال قبل - در این اتاق معرفی و جوایز نفیسی خدمتشان تقدیم خواهیم کرد. نفرات برتر، کاربرانی هستند که بیشترین سعی و تلاش را در حل و بحث مسائل داشته اند و روش های حل ابتکاریشان، به غنای علمی اتاق کمک کرده باشد. البته با هماهنگی مدیر سایت، قرار شده است که به زودی نوع جوایز نیز مشخص شود.


-------------------------------------------


تقاضا ی برادرانه از کاربران عزیز:

1- دوستان عزیزم، در حل مسائل، از نوشتن فرمول ها و عبارات ریاضی به صورت معمولی اکیداً خودداری کنید. این کار باعث بی نظمی بسیاری در انجمن می شود. برای فراگیری روش درست انتشار عبارات و فرمول های ریاضی در سایت - که بسیار ساده هم هست - به اتاق زیر مراجعه فرمایید:


انتشار درست عبارات و فرمول های ریاضی در سایت ([ برای مشاهده لینک ، لطفا با نام کاربری خود وارد شوید یا ثبت نام کنید ])

2- موکداً توصیه می کنیم از سایت هایی مانند artofproblemsolving.com برای نمایش عبارات و فرمولهای ریاضی استفاده نکنید؛ علت آن در پست 12 در همین اتاق ([ برای مشاهده لینک ، لطفا با نام کاربری خود وارد شوید یا ثبت نام کنید ]) توضیح داده شده است.

با امید به این که اتاق حل مساله در پیشرفت علمی کاربران عزیز موثر و برای ما ذخیره ای در یوم الحساب باشد.

موفق و پیروز باشید.

25 تیر 1388


-------------------------------------------


لینک ها برای دسترسی سریع

اتاق حل مساله در سال 1387با 116 مساله ی حل شده در سطوح مختلف تحصیلی ([ برای مشاهده لینک ، لطفا با نام کاربری خود وارد شوید یا ثبت نام کنید ])

اسامی برندگان اتاق حل مساله ی سال 1387 ([ برای مشاهده لینک ، لطفا با نام کاربری خود وارد شوید یا ثبت نام کنید ])

mofidy1
16-07-2009, 18:33
با سلام

برای شماره گذاری صفحات یک کتاب بزرگ، چاپ کننده ی کتاب، 2989 رقم به کار برده است. آن کتاب چند صفحه دارد؟ (اولین صفحه ی کتاب با 1 شماره گذاری شده است.)

موفق باشید.

25 تیر 1388

saber57
16-07-2009, 19:22
با سلام

برای شماره گذاری صفحات یک کتاب بزرگ، چاپ کننده ی کتاب، 2989 رقم به کار برده است. آن کتاب چند صفحه دارد؟

موفق باشید.

25 تیر 1388
سلام

کلا 2989 رقم بکار گرفته شده است . دقت کنید :


1ز 1 تا 9 یک رقمی : 9 رقم
از 10 تا 99 هست 90 عدد 2 رقمی که مجموعا : 2*90= 180 رقم
1ز 100 تا 999 هست900 عدد 3 رقمی که مجموعا : 3*900=2700 رقم
کلا از 1 تا 999 ما 2700+180+9=2889 رقم داریم و از آنجایی که کل کتاب 2989 رقم در اون بکار رفته نتیجه میگیریم که چند تا از برگه ها 4 رقمی هستند . اما مجموع این ارقام 4 رقمی تفاضل 2889 از مجموع ارقام کل برگه هاست . پس :
تعداد کل ارقام 4 رقمی =2889-2989=100 رقم داریم حالا تعداد اعداد 4 رقمی (دقت کنید عدد نه رقم):
4/100= 25 برگه 4 رقمی داریم . از قبل هم که 2889 رقم یا همون 999 صفحه داشتیم پس مجموعا : 999+25=1024 صفحه در کتاب موجود هست .

mofidy1
17-07-2009, 16:41
بسم الله الرحمن الرحیم

رسول عظیم الشأن اسلام (صلی الله علیه و آله و سلم )

یک ساعت تفکر از هفتاد سال عبادت، برتر است.

خواجه نصیرالدین طوسی:

و چنانک به نادر افتد که مردمی که نجارت ]نجاری[ ناآموخته، تختی نیک تواند تراشید، به نادر افتد که مردمی منطق نا آموخته، علمی مکتسب بر وجهی کامل حاصل تواند کرد.


لایب نیتس:

روش حل خوب است، به شرطی که از همان آغاز بتوان پیش بینی کرد، که با دنبال کردن این روش می توان به هدف رسید.

سیلوستر:

هر چه بیشتر تمرین یا اثبات کنیم، تمرین یا اثبات برایمان ساده تر می شود.

دکارت:

هر مسأله ای که مورد مطالعه ی شماست، تا آن جا که ممکن است و تا آن جا که لازم دارید، به بخش هایی تقسیم کنید تا بتوانید این بخش ها را ساده تر حل کنید.

دکارت:

هر راه حلی که برای مساله ای پیدا می کنم، به عنوان سرمشق به من کمک می کند تا مسأله های دیگری را هم به نتیجه برسانم.

دالامبر:

پیش بروید، که فهم موضوع در پی شما خواهد آمد.

هربرت وی. پروشنو:

اگر از اشتباهات خود، عبرت نگیریم، پس دیگر چرا اشتباه کنیم.


-----------------------------------------------

برای هر یک از جملات بالا، باید هفتاد من تفسیر نوشت، تا شاید ارزش آن ها در حل مساله مشخص شود. متاسفانه بسیاری از ما - به دلایل مختلف - نه می دانیم چگونه باید مساله حل کنیم و نه می خواهیم. به تجربه در این عمر کوتاهم - که تقریباً نیمی از آن به تدریس و سر و کله زدن با کودکان و نوجوانان و جوانان گذشت - متوجه شده ام که اصلی ترین مشکل ما در زندگی - به طور اعم - و در ریاضیات - به طور اخص - ناتوانی در حل مساله است؛ این که مساله چیست، مشکل کجاست، چگونه ایجاد شده و چگونه حل می شود و راه رسیدن از مجهول به معلوم چیست!!

انشاءالله در چند جلسه، سعی می کنم در حد توانایی ام و تناسب آن با این سایت، به تفصیل مطالبی در این زمینه خدمتتان تقدیم کنم. به لطف خدا، در جلسه ی بعد منابع مهمی هم در زمینه ی روش های حل مساله، معرفی خواهم کرد.

و علی الله التوفیق

26 تیر ماه 1388

davy jones
18-07-2009, 15:44
روزهای شنبه mir@

پس چي شد؟:3: شنبه داره كم كم تموم ميشه! لطفا اگه ميخواين سوال بذارين خواهشا از صبح بذارين كه وقت كافي براي فكر كردن وجود داشته باشه.

ali_hp
19-07-2009, 08:18
سلام
سوال روز یکشنبه:
اتومبیلی مسیر صد کیلومتری را در یک سا عت طی می کند،حکم زیر را اثبات یا رد کنید:
قسمتی از مسیر به طول ده کیلومتر وجود دارد که اتومبیل ان را در شش دقیقه طی کرده است.
28 تیر 1388

chessmathter
20-07-2009, 09:10
سلام
سوال روز یکشنبه:
اتومبیلی مسیر صد کیلومتری را در یک سا عت طی می کند،حکم زیر را اثبات یا رد کنید:
قسمتی از مسیر به طول ده کیلومتر وجود دارد که اتومبیل ان را در شش دقیقه طی کرده است.
28 تیر 1388
حکم و رد میکنیم با این مثال که اتومبیل این طوری حرکت میکنه
10 کیلومتر اول رو تو
[ برای مشاهده لینک ، لطفا با نام کاربری خود وارد شوید یا ثبت نام کنید ]دقی قه طی میکنه بعد 6 دقیقه استراحت دوباره 10 کیلومتره بعدی تو [ برای مشاهده لینک ، لطفا با نام کاربری خود وارد شوید یا ثبت نام کنید ]و همینطور تا آخر یعنی
هر 10 کیلومتر و در [ برای مشاهده لینک ، لطفا با نام کاربری خود وارد شوید یا ثبت نام کنید ] دقیقه و بینشون 6 دقیقه استراحت میکنه این طوری هیچ مسیری به طول 10 کیلومتر نیست که در 6 دقیقه طی شده باشه

FlightMedia
20-07-2009, 09:24
سلام
سوال روز یکشنبه:
اتومبیلی مسیر صد کیلومتری را در یک سا عت طی می کند،حکم زیر را اثبات یا رد کنید:
قسمتی از مسیر به طول ده کیلومتر وجود دارد که اتومبیل ان را در شش دقیقه طی کرده است.
28 تیر 1388

حكم رد ميشه
اگر سرعت ثابت باشه يعني 100 كيلومتر بر ساعت كه در هر 6 دقيقه 10 كيلومتر را طي ميكند.

ولي چون سرعت ثابت نيست لزوما حكم ثابت نميشه
مثال نقض:
ابتدا با سرعت 5940 كيلومتر بر ساعت در 1 دقيقه 99 كيلومتر را طي ميكند! و سپس با سرعت 1.02 1 كيلومتر را در 59 دقيقه طي ميكند.
البته اصل پيوستگي سرعت در نظر گرفته نشده

chessmathter
20-07-2009, 09:37
حكم رد ميشه
اگر سرعت ثابت باشه يعني 100 كيلومتر بر ساعت كه در هر 6 دقيقه 10 كيلومتر را طي ميكند.
فک کنم منظورتون این بود که حکم درسته یعنی یک مسیری 10 کیلومتری هست که در 6 دقیقه طی کنه درسته!مثالی که زدین مسیری داره که هیچ به ازای هر 10 کیلومتر مسیر و در 6 دقیقه طی میکنه ولی اگه میخوان بگین حکم درسته باید اونو اثبات کنین و حکم اینه اون وقت
که اگه اتومبیل 100 کیلومتر ودر 1 ساعت به هر صورت طی کنه مسیری به طوال 10 کیلومتر وجود داره که در 6 دقیقه طی کرده باشه

chessmathter
20-07-2009, 09:40
حكم رد ميشه
اگر سرعت ثابت باشه يعني 100 كيلومتر بر ساعت كه در هر 6 دقيقه 10 كيلومتر را طي ميكند.

ولي چون سرعت ثابت نيست لزوما حكم ثابت نميشه
مثال نقض:
ابتدا با سرعت 5940 كيلومتر بر ساعت در 1 دقيقه 99 كيلومتر را طي ميكند! و سپس با سرعت 1.02 1 كيلومتر را در 59 دقيقه طي ميكند.
البته اصل پيوستگي سرعت در نظر گرفته نشده
اول این که میتونیم 10 کیلومتری پیدا کنیم که قسمتی از او 1 کیلومترآخر رو و 99 کیلومتر قبلی رو در 6 دقیقه رفته باشه

chessmathter
20-07-2009, 09:42
حکم و رد میکنیم با این مثال که اتومبیل این طوری حرکت میکنه
10 کیلومتر اول رو تو
[ برای مشاهده لینک ، لطفا با نام کاربری خود وارد شوید یا ثبت نام کنید ]دقی قه طی میکنه بعد 6 دقیقه استراحت دوباره 10 کیلومتره بعدی تو [ برای مشاهده لینک ، لطفا با نام کاربری خود وارد شوید یا ثبت نام کنید ]و همینطور تا آخر یعنی
هر 10 کیلومتر و در [ برای مشاهده لینک ، لطفا با نام کاربری خود وارد شوید یا ثبت نام کنید ] دقیقه و بینشون 6 دقیقه استراحت میکنه این طوری هیچ مسیری به طول 10 کیلومتر نیست که در 6 دقیقه طی شده باشه
غلطه همون 10 کیلومتر اول و در 6دقیقه طی کرده فوقش اینکه پنج و 4 دهم دقیقه استراحت کنه:31:! یه حس میگه حکم درسته من رفتم دنباله اثبات:31:

mofidy1
20-07-2009, 11:36
با سلام

بعضی از دوستان برای نمایش عبارات ریاضی، از سایت هایی مانند artofproblemsolving.com استفاده می کنند. به تجربه دیده شده است که این سایت ها پس از مدتی این گونه عکس ها را از روی سرور خود پاک می کنند و بنابر این پست های دوستان عزیز نیز غیر قابل استفاده خواهد شد. لذا خواهش می کنم از این گونه سایت ها برای نمایش عبارات ریاضی استفاده نفرمایید. فعلاً بهترین روش ممکن، استفاده از روش هایی است که در اتاق زیر، مفصلاً توضیح داده شده است:


انتشار درست عبارات و فرمول های ریاضی در سایت ([ برای مشاهده لینک ، لطفا با نام کاربری خود وارد شوید یا ثبت نام کنید ])

موفق باشید.

29 تیر 1388

yugioh
20-07-2009, 19:51
1.درون یک کیسه 25 گوی یک شکل هست، شامل 6 قرمز 9 سبز 10 سفید. دو نفر بازی زیر رو انجام می دهند،
اولی یک گوی بیرون میاره. اگر سبز باشه برنده است اگر قرمز یا سفید باشه بازنده.
دومی یک گوی رو بیرون میاره، اگر قرمز باشه برنده است. اگر سبز باشه بازنده است. اگر سفید باشه اون گوی رو خارج می کنه و از گویهای باقی مانده یکی دیگه رو خارج می کنه. ( اگر 10 بار هم پست سر هم گوی سفید بیرون میاد.
اگر هر دو تا برنده یا بازنده شوند بازی برابر حساب شده و مجددا بازی می کنند، تا نهایتا یک نفر برندده بشه.
اگر شما جایگزین این دو نفر بودید به جای کدام یکی بازی می کردید؟

2.حالا یه سوال دیگه از این گوی ها، 7 تا از گویها روخارج می کنیم،اگر بدونیم توی انتخابیها، سفید وجود داره با چه احتمالی حداقل 1 قرمز و 2 سبز داره؟

saber57
20-07-2009, 22:33
سلام
سوال روز یکشنبه:
اتومبیلی مسیر صد کیلومتری را در یک سا عت طی می کند،حکم زیر را اثبات یا رد کنید:
قسمتی از مسیر به طول ده کیلومتر وجود دارد که اتومبیل ان را در شش دقیقه طی کرده است.
28 تیر 1388
سلام
برای سرعت ثابت یا همان شتاب صفر مساله درست هست . اما در حالت کلی من با استفاده از اصل لانه کبوتری حل میکنم .
قضیه کاربردی اصل لانه کبوتری : اگر m کبوتر n لانه را اشغال کنند ، آنگاه در یکی از لانه ها حداقل 1 + [m-1/n] کبوتر وجود دارد . (در جایی که تعداد کبوتر بیش از لانه باشد)
حالا مسافت را کبوتر و زمان یک ساعت یا 60 دقیقه را لانه فرض میکنیم بنابراین حداقل 1 + [60/1-100]= 1 + [60/99]=2 کیلومتر وجود دارد که در مدت یک ثانیه طی شود .پس حداقل 2+2+2+2+2+2=12 کیلومتر وجود دارد که اتوبیل آنرادر 6 دقیقه طی میکند . بنابراین اتومبیل با توجه به حداقل پیمودن 2 کیلوتر بر دقیقه میتواند 12 کیلوتر را در 6 دقیقه ظی کند . پس 10 کیلومتر هم قسمتی از این مسیر پیموده شده است .

جواب سوال تصحیح شد . به صفحه 4 پست شماره 37 بروید

sherlockholmz
21-07-2009, 09:38
باسلام
سوال روز سه شنبه:
چوپاني يك گله 100 راسي از شتر، گاو وگوسفند داشت.يك روز تصميم مي گيرد كارش را عوض كند و به ساخت وساز بپردازد.پس تمام گله را مي فروشدو با پولش آپارتمان سازي شروع مي كند.اگر هر گوسفند را 1 واحد(منظور از واحد، واحد پول است!) ، هر گاو را 10 واحد و هر شتر را 30 واحد فروخته باشدو بابت اين معامله 1000 واحد نصيبش شده باشد، بفرمائيد از هر حيوان چند تا داشته است. دو نكته:
1- لطفا" مسئله را بروش سعي وخطا حل نكنيد، اين مسائل راه حل درست و حسابي دارند، باور كنيد.
2-اين چوپان هنوز هم تو كار ساخت وساز است و وضعش از صد تا تحصيل كرده و ... بهتر است، در ضمن او را مهندس صدا مي زنند!:20:
موفق باشيد.

Iron
21-07-2009, 12:31
سلام
سوال روز یکشنبه:
اتومبیلی مسیر صد کیلومتری را در یک سا عت طی می کند،حکم زیر را اثبات یا رد کنید:
قسمتی از مسیر به طول ده کیلومتر وجود دارد که اتومبیل ان را در شش دقیقه طی کرده است.
28 تیر 1388

سلام

اگر x(t)a مسافت طی شده در زمان t (برحسب دقیقه) باشه، f(t)a رو بصورت زیر تعریف می کنیم:

[ برای مشاهده لینک ، لطفا با نام کاربری خود وارد شوید یا ثبت نام کنید ](t)=x(t+10)-x(t)\,&space;\,&space;\,&space;\,&space;\,&space;\,&space;\,&space;\,&space;\,&space;\,&space;0\leq&space;x\leq&space;56

- از آنجاکه x پیوستست، تابع f نیز پیوسته می باشد.
- f نمی تواند همیشه بزرگتر از 10 باشد. چراکه در اینصورت کل مسافت طی شده در یک ساعت میشه:

[ برای مشاهده لینک ، لطفا با نام کاربری خود وارد شوید یا ثبت نام کنید ](0)+f(6)+f(12)+...+f(54)>100

- f نمی تواند همیشه کوچکتر از 10 باشد. چراکه در اینصورت کل مسافت طی شده در یک ساعت میشه:

[ برای مشاهده لینک ، لطفا با نام کاربری خود وارد شوید یا ثبت نام کنید ](0)&plus;f(6)&plus;f(12)&plus;...&plus;f(54)<100

بنابراین تابع f دارای ماکزیمم بزرگتر از 10 و مینیمم کوچکتر از 10 میباشد و از آنجاکه تابعیست پیوسته بنابر این وجود دارد T که رابطه زیر در خصوص آن برقرار باشد:

[ برای مشاهده لینک ، لطفا با نام کاربری خود وارد شوید یا ثبت نام کنید ](T)=10\Rightarrow&space;x(T&plus;6)-x(T)=10

davy jones
21-07-2009, 13:32
باسلام
سوال روز سه شنبه:
چوپاني يك گله 100 راسي از شتر، گاو وگوسفند داشت.يك روز تصميم مي گيرد كارش را عوض كند و به ساخت وساز بپردازد.پس تمام گله را مي فروشدو با پولش آپارتمان سازي شروع مي كند.اگر هر گوسفند را 1 واحد(منظور از واحد، واحد پول است!) ، هر گاو را 10 واحد و هر شتر را 30 واحد فروخته باشدو بابت اين معامله 1000 واحد نصيبش شده باشد، بفرمائيد از هر حيوان چند تا داشته است. دو نكته:
1- لطفا" مسئله را بروش سعي وخطا حل نكنيد، اين مسائل راه حل درست و حسابي دارند، باور كنيد.
2-اين چوپان هنوز هم تو كار ساخت وساز است و وضعش از صد تا تحصيل كرده و ... بهتر است، در ضمن او را مهندس صدا مي زنند!:20:
موفق باشيد.

این مساله جواب یکتا نداره. اولین جوابی که میشه اون رو در مسئله صدق داد اینه که چوپان (آقای مهندس!) فقط 100 تا گاو داشته باشه. ولی اگه فرض کنیم که از هر 3 حیوون حداقل یکی داشته باشه داریم:
فرض:
x=گوسفند
y=گاو
z=شتر

[ برای مشاهده لینک ، لطفا با نام کاربری خود وارد شوید یا ثبت نام کنید ]

[ برای مشاهده لینک ، لطفا با نام کاربری خود وارد شوید یا ثبت نام کنید ]

[ برای مشاهده لینک ، لطفا با نام کاربری خود وارد شوید یا ثبت نام کنید ]

با توجه به این که x و y و z اعداد صحیح هستن پس:

[ برای مشاهده لینک ، لطفا با نام کاربری خود وارد شوید یا ثبت نام کنید ]

از طرفی از دو معادله و 3 مجهول قبلی داریم :

[ برای مشاهده لینک ، لطفا با نام کاربری خود وارد شوید یا ثبت نام کنید ]

و باز چون x و z اعداد صحیح اند این سلسله جوابها رو داریم:

[ برای مشاهده لینک ، لطفا با نام کاربری خود وارد شوید یا ثبت نام کنید ]
[ برای مشاهده لینک ، لطفا با نام کاربری خود وارد شوید یا ثبت نام کنید ]
[ برای مشاهده لینک ، لطفا با نام کاربری خود وارد شوید یا ثبت نام کنید ]

در هر سری از جوابهای بالا شرط:
[ برای مشاهده لینک ، لطفا با نام کاربری خود وارد شوید یا ثبت نام کنید ]
صادقه. ولی یه شرط دیگه هم از دستگاه معادلات بدست میآد :

[ برای مشاهده لینک ، لطفا با نام کاربری خود وارد شوید یا ثبت نام کنید ]


که میبینیم که در هر 3 سری این شرط هم برقراره.

حال باید چک کنیم که این اعداد در معادله دوم دستگاه هم صدق میکنن یا خیر :

[ برای مشاهده لینک ، لطفا با نام کاربری خود وارد شوید یا ثبت نام کنید ]

پس نتیجه میگیریم که هر 3 دسته از اعداد، میتونن جواب مسئله باشن.

موفق باشین.
88/4/30

eh_mn
22-07-2009, 06:26
ثابت كنيد كه اگر [ برای مشاهده لینک ، لطفا با نام کاربری خود وارد شوید یا ثبت نام کنید ]{1}{a} يك عدد صحيح باشد آنگاه براي هر n ،[ برای مشاهده لینک ، لطفا با نام کاربری خود وارد شوید یا ثبت نام کنید ]^n&plus;\frac{1}{a^n} نيز عددي صحيح است.

ـــــــــــــــــــــــــ
31 / 04 / 88

Iron
22-07-2009, 08:15
سلام. عبارت بعد از ویرگول قابل خوندن نیست.

sherlockholmz
22-07-2009, 08:25
این مساله جواب یکتا نداره...
88/4/30

متشكرم پاسخ كاملا" صحيح است.
موفق باشيد.:46:

eh_mn
22-07-2009, 09:13
سلام. عبارت بعد از ویرگول قابل خوندن نیست.

ويرايش شد.

saber57
23-07-2009, 06:58
ثابت كنيد كه اگر [ برای مشاهده لینک ، لطفا با نام کاربری خود وارد شوید یا ثبت نام کنید ]{1}{a} يك عدد صحيح باشد آنگاه براي هر n ،[ برای مشاهده لینک ، لطفا با نام کاربری خود وارد شوید یا ثبت نام کنید ]^n&plus;\frac{1}{a^n} نيز عددي صحيح است.

ـــــــــــــــــــــــــ
31 / 04 / 88

فرض کنیم [ برای مشاهده لینک ، لطفا با نام کاربری خود وارد شوید یا ثبت نام کنید ]{1}{a}=m\epsilon&space;z بنابراین :


[ برای مشاهده لینک ، لطفا با نام کاربری خود وارد شوید یا ثبت نام کنید ](&space;a&space;&plus;\frac{1}{a}&space;\right&space;)^2=\left&space; (a^2&plus;\frac{1}{a^2}&space;\right&space;)&space;&plus;2\rightarrow&space;\left&space;(a ^2&plus;\frac{1}{a^2}&space;\right&space;)=m^2-2\epsilon&space;z



[ برای مشاهده لینک ، لطفا با نام کاربری خود وارد شوید یا ثبت نام کنید ](&space;a&plus;\frac{1}{a}&space;\right&space;)^3=\left&space;( &space;a&plus;\frac{1}{a}&space;\right&space;)^2.\left&space;(&space;a&plus;\frac{1}{a}&space;\r ight&space;)=\left&space;(a^3&plus;\frac{1}{a^3}&space;\right&space;)&space;&plus;3\left&space;( &space;a&plus;\frac{1}{a}&space;\right&space;)\rightarrow&space;\left&space;(a^3&plus;\fra c{1}{a^3}&space;\right&space;)&space;=m^3-3m\epsilon&space;z


به همین ترتیب میتوان نشان داد :


[ برای مشاهده لینک ، لطفا با نام کاربری خود وارد شوید یا ثبت نام کنید ](&space;a&plus;\frac{1}{a}&space;\right&space;)^4=\left&space;( &space;a^4&plus;\frac{1}{a^4}&space;\right&space;)&plus;4\left&space;(&space;a^2&plus;\frac{1}{ a^2}&space;\right&space;)&plus;6\rightarrow&space;\left&space;(&space;a^4&plus;\frac{1}{a^ 4}&space;\right&space;)=m^4-4m^2&plus;2\epsilon&space;z



[ برای مشاهده لینک ، لطفا با نام کاربری خود وارد شوید یا ثبت نام کنید ](&space;a&plus;\frac{1}{a}&space;\right&space;)^5=\left&space;( &space;a^{5}&plus;\frac{1}{a^5}&space;\right&space;)&plus;5\left&space;(&space;a^3&plus;\frac{1 }{a^3}&space;\right&space;)&plus;10\left&space;(&space;a&plus;\frac{1}{a}&space;\right&space;)\r ightarrow&space;\left&space;(&space;a^5&plus;\frac{1}{a^5}&space;\right&space;)=m^5-5m^3&plus;5m\epsilon&space;z


[ برای مشاهده لینک ، لطفا با نام کاربری خود وارد شوید یا ثبت نام کنید ](&space;a&plus;\frac{1}{a}&space;\right&space;)^6=\left&space;( &space;a^{6}&plus;\frac{1}{a^6}&space;\right&space;)&plus;6\left&space;(&space;a^4&plus;\frac{1 }{a^4}&space;\right&space;)&plus;15\left&space;(&space;a^2&plus;\frac{1}{a^2}&space;\right &space;)&plus;20\rightarrow&space;\left&space;(&space;a^6&plus;\frac{1}{a^6}&space;\right&space; )=m^6-6m^4&plus;9m^2&plus;2\epsilon&space;z


نکته مهم :46: : در حالت کلی میتوان رابطه را به صورت زیر نشان داد :(چه n زوج باشد چه فرد)


[ برای مشاهده لینک ، لطفا با نام کاربری خود وارد شوید یا ثبت نام کنید ](&space;a&plus;\frac{1}{a}&space;\right&space;)^n=\sum_{i =0}^{\left&space;[&space;\frac{n}{2}&space;\right&space;]}&space;\left&space;(a^{n-2i}&plus;\frac{1}{a^{n-2i}}&space;\right&space;)\binom{n}{i}


اگر عبارت [ برای مشاهده لینک ، لطفا با نام کاربری خود وارد شوید یا ثبت نام کنید ](&space;a^{n}&space;&plus;\frac{1}{a^{n}}&space;\right&space;) را از حاصل مجموع خارج کنیم ، نتیجه میشود :



[ برای مشاهده لینک ، لطفا با نام کاربری خود وارد شوید یا ثبت نام کنید ](&space;a^{n}&space;&plus;\frac{1}{a^{n}}&space;\right&space;)&space; =&space;\left&space;(&space;a&plus;\frac{1}{a}&space;\right&space;)^n&space;-\sum_{i=1}^{\left&space;[&space;\frac{n}{2}&space;\right&space;]}&space;\left&space;(a^{n-2i}&plus;\frac{1}{a^{n-2i}}&space;\right&space;)\binom{n}{i}



عبارت سمت راست همانطور که در بالا دیدیم ، تابعی از عدد صحیح [ برای مشاهده لینک ، لطفا با نام کاربری خود وارد شوید یا ثبت نام کنید ](&space;a&space;&plus;\frac{1}{a}&space;\right&space;)=m میباشد پس عبارت مساوی با آن یعنی رابطه چپ تساوی هم تابعی از اعداد صحیح هست .


روش دوم : اگر n عدد طبیعی باشد با استفاده از استقرا ثابت میکنیم :
فرض مساله : [ برای مشاهده لینک ، لطفا با نام کاربری خود وارد شوید یا ثبت نام کنید ]{1}{a} عددی صحیح است .


روش استقرا : اگر برای اندیس n ام [ برای مشاهده لینک ، لطفا با نام کاربری خود وارد شوید یا ثبت نام کنید ]^n&plus;\frac{1}{a^n} معتبر باشد (تا اندیس n ام) برای n+1 هم حکم ثابت شود .
اثبات : میدانیم که مجموعه اعداد صحیح نسبت به عمل ضرب بسته است . طبق فرض مساله دو عدد [ برای مشاهده لینک ، لطفا با نام کاربری خود وارد شوید یا ثبت نام کنید ]^n&plus;\frac{1}{a^n} و [ برای مشاهده لینک ، لطفا با نام کاربری خود وارد شوید یا ثبت نام کنید ]{1}{a} جزئ اعداد صحیح فرض شده اند پس حاصلضرب آنها هم به اعداد صحیح تعلق دارد :


[ برای مشاهده لینک ، لطفا با نام کاربری خود وارد شوید یا ثبت نام کنید ](a&plus;\frac{1}{a}&space;\right&space;)\left&space;(&space;a^n &plus;\frac{1}{a^n}&space;\right&space;)=\left&space;(&space;a^{n&plus;1}&plus;\frac{1}{a ^{n&plus;1}}&space;\right&space;)&plus;\left(&space;a^{n-1}&plus;\frac{1}{a^{n-1}}&space;\right&space;)\rightarrow&space;\left&space;(&space;a^{n&plus;1}&plus;\frac{1}{a ^{n&plus;1}}&space;\right&space;)=&space;\left&space;(a&plus;\frac{1}{a}&space;\right&space;)\le ft&space;(&space;a^n&plus;\frac{1}{a^n}&space;\right&space;)-&space;\left(&space;a^{n-1}&plus;\frac{1}{a^{n-1}}&space;\right&space;)



از طرفی [ برای مشاهده لینک ، لطفا با نام کاربری خود وارد شوید یا ثبت نام کنید ](a&plus;\frac{1}{a}&space;\right&space;)\left&space;(&space;a^n &plus;\frac{1}{a^n}&space;\right&space;)\epsilon&space;z و چون تا اندیس n ام حکم ثابت هست پس برای n-1 هم برقرار هست و [ برای مشاهده لینک ، لطفا با نام کاربری خود وارد شوید یا ثبت نام کنید ](&space;a^{n-1}&plus;\frac{1}{a^{n-1}}&space;\right&space;)\epsilon&space;z تفاضل هر دو هم عدد صحیح هم متعلق به مجموعه اعداد صحیح است در نتیجه [ برای مشاهده لینک ، لطفا با نام کاربری خود وارد شوید یا ثبت نام کنید ](&space;a^{n&plus;1}&plus;\frac{1}{a^{n&plus;1}}&space;\right &space;)\epsilon&space;z و حکم ثابت میشود .


1/5/88
ویرایش 5/5 88

mofidy1
23-07-2009, 10:12
با سلام

برای شماره گذاری صفحات یک کتاب بزرگ، چاپ کننده ی کتاب، 2989 رقم به کار برده است. آن کتاب چند صفحه دارد؟ (اولین صفحه ی کتاب با 1 شماره گذاری شده است.)

موفق باشید.

25 تیر 1388

با سلام

برای حل کامل و درست این مساله از saber57 تشکر می کنم. برای دیدن راه حل ایشان به پست 3 ([ برای مشاهده لینک ، لطفا با نام کاربری خود وارد شوید یا ثبت نام کنید ]) مراجعه فرمایید.

حل این مساله از طریق مساله:


مساله ای وابسته به مساله ی ما:

اگر کتاب 9 صفحه داشت، چاپ خانه چند رقم به کار می برد؟ جواب: 9 رقم
اگر کتاب 99 صفحه داشت، چاپ خانه چند رقم به کار می برد؟ جواب: 90*2+9=189 رقم
اگر کتاب 999 صفحه داشت، چاپ خانه چند رقم به کار می برد؟ جواب: 3*900+2*90+9=2889 رقم

حال اگر کتاب x صفحه داشته باشد باید معادله ی زیر را حل کنیم:


[ برای مشاهده لینک ، لطفا با نام کاربری خود وارد شوید یا ثبت نام کنید ](x-999)&space;=&space;2989

که پس از حل به دست می آید: x=1024

آموزش حل مساله:

بنابر این گاهی تخمین مقدماتی یک مجهول برای حل مساله لازم است.

موفق باشید.

1 مرداد 1388

mofidy1
23-07-2009, 10:57
با سلام

دو عبارت زیر را برای هر عدد حقیقی مثبت a و b و c ثابت کنید:


[ برای مشاهده لینک ، لطفا با نام کاربری خود وارد شوید یا ثبت نام کنید ]{align}&space;4(a^3&plus;b^3)&\geq&space;(a&plus;b)^3&space;\nonumber\\&space;9(a^3&plus;b^3&plus;c^3)&space;&\geq&space;(a&plus;b&plus;c)^3&space;\nonumber\end{align}سی و دومین المپیاد ریاضی انگلستان 1996

(از کلمه ی المپیاد نترسید. مساله ی ساده اما زیبایی است!!!)

موفق باشید.

1 مرداد 1388

CppBuilder2006
24-07-2009, 13:36
اولی که راحته:
[ برای مشاهده لینک ، لطفا با نام کاربری خود وارد شوید یا ثبت نام کنید ]

کافیه نشون بدیم:
][ برای مشاهده لینک ، لطفا با نام کاربری خود وارد شوید یا ثبت نام کنید ][

برای [ برای مشاهده لینک ، لطفا با نام کاربری خود وارد شوید یا ثبت نام کنید ]نامنفیه:
[[ برای مشاهده لینک ، لطفا با نام کاربری خود وارد شوید یا ثبت نام کنید ][]

و
[ برای مشاهده لینک ، لطفا با نام کاربری خود وارد شوید یا ثبت نام کنید ][
که همواره برقرار است.
برای دومی هنوز فکر نکردم.

saber57
24-07-2009, 13:46
با سلام

برای هر عدد حقیقی مثبت a و b و c ثابت کنید:
[centre]
[ برای مشاهده لینک ، لطفا با نام کاربری خود وارد شوید یا ثبت نام کنید ]{align}&space;4(a^3&plus;b^3)&\geq&space;(a&plus;b)^3&space;\nonumber\\&space;9(a^3&plus;b^3&plus;c^3)&space;&\geq&space;(a&plus;b&plus;c)^3&space;\nonumber\end{align}
[centre\]
سی و دومین المپیاد ریاضی انگلستان 1996

(از کلمه ی المپیاد نترسید. مساله ی ساده اما زیبایی است!!!)

موفق باشید.

1 مرداد 1388

قسمت اول سوال :


[ برای مشاهده لینک ، لطفا با نام کاربری خود وارد شوید یا ثبت نام کنید ](&space;a&plus;b&space;\right&space;)^3=a^3&plus;b^3&plus;3a^2b&plus;3ab ^2
[ برای مشاهده لینک ، لطفا با نام کاربری خود وارد شوید یا ثبت نام کنید ](&space;a&plus;b&space;\right&space;)^3&plus;3\left&space;(&space;a^3&plus;b^3&space; \right&space;)=4\left&space;(&space;a^3&plus;b^3&space;\right&space;)&plus;3ab\left&space;(&space;a&plus;b&space; \right&space;)
[ برای مشاهده لینک ، لطفا با نام کاربری خود وارد شوید یا ثبت نام کنید ](&space;a^3&plus;b^3&space;\right&space;)=\left&space;(&space;a&plus;b&space;\r ight&space;)^3&plus;3a^3&plus;3b^3-3a^2b-3ab^2
[ برای مشاهده لینک ، لطفا با نام کاربری خود وارد شوید یا ثبت نام کنید ](&space;a^3&plus;b^3&space;\right&space;)=\left&space;(&space;a&plus;b&space;\r ight&space;)^3&plus;3a^2\left&space;(&space;a-b&space;\right&space;)-3b^2\left&space;(&space;a-b&space;\right&space;)
[ برای مشاهده لینک ، لطفا با نام کاربری خود وارد شوید یا ثبت نام کنید ](&space;a^3&plus;b^3&space;\right&space;)=\left&space;(&space;a&plus;b&space;\r ight&space;)^3&plus;3\left&space;(&space;a-b&space;\right&space;)\left&space;(&space;a^2-b^2&space;\right&space;)=\left&space;(&space;a&plus;b&space;\right&space;)^3&plus;3\left&space;(&space;a-b&space;\right&space;)^2\left&space;(&space;a&plus;b&space;\right&space;)

اما عبارت [ برای مشاهده لینک ، لطفا با نام کاربری خود وارد شوید یا ثبت نام کنید ](&space;a-b&space;\right&space;)^2\left&space;(&space;a&plus;b\right&space;) بزرگتر یا مساوی صفر هست بنابراین :


[ برای مشاهده لینک ، لطفا با نام کاربری خود وارد شوید یا ثبت نام کنید ](&space;a^3&plus;b^3&space;\right&space;)\geq\left&space;(&space;a&plus;b &space;\right&space;)^3

CppBuilder2006
24-07-2009, 14:24
حالا دومی. فرض کنید [ برای مشاهده لینک ، لطفا با نام کاربری خود وارد شوید یا ثبت نام کنید ] ثابت باشند. نشان می­دهیم برای هر [ برای مشاهده لینک ، لطفا با نام کاربری خود وارد شوید یا ثبت نام کنید ] داریم:
[ برای مشاهده لینک ، لطفا با نام کاربری خود وارد شوید یا ثبت نام کنید ]




تعریف می­کنیم:
[ برای مشاهده لینک ، لطفا با نام کاربری خود وارد شوید یا ثبت نام کنید ]



داریم:

[ برای مشاهده لینک ، لطفا با نام کاربری خود وارد شوید یا ثبت نام کنید ]

و

[ برای مشاهده لینک ، لطفا با نام کاربری خود وارد شوید یا ثبت نام کنید ]


پس ثابت شد برای هر b<=x، [ برای مشاهده لینک ، لطفا با نام کاربری خود وارد شوید یا ثبت نام کنید ]

به علت تقارن برای هر [ برای مشاهده لینک ، لطفا با نام کاربری خود وارد شوید یا ثبت نام کنید ] داریم:
[ برای مشاهده لینک ، لطفا با نام کاربری خود وارد شوید یا ثبت نام کنید ]
هووووووووو دیدین چه راحت بود!


این راه حل رو می شه برای حالت کلی تر این مسأله هم به کار برد.

chessmathter
24-07-2009, 14:24
با سلام

برای هر عدد حقیقی مثبت a و b و c ثابت کنید:

[ برای مشاهده لینک ، لطفا با نام کاربری خود وارد شوید یا ثبت نام کنید ] 29&space;&%5Cgeq&space;%28a&plus;b&plus;c%29%5E3&space;%5Cend%7Balign%7D

سی و دومین المپیاد ریاضی انگلستان 1996

(از کلمه ی المپیاد نترسید. مساله ی ساده اما زیبایی است!!!)

موفق باشید.

1 مرداد 1388
با توجه به این که a,b,c مثبت اند


[ برای مشاهده لینک ، لطفا با نام کاربری خود وارد شوید یا ثبت نام کنید ]



حالا واسه 2 از یک استفاده میکنیم داریم:

[ برای مشاهده لینک ، لطفا با نام کاربری خود وارد شوید یا ثبت نام کنید ]


حالا اگر اینو اثبات کنیم مسله حله:

[ برای مشاهده لینک ، لطفا با نام کاربری خود وارد شوید یا ثبت نام کنید ]
که اخری ام از فرمول اویلر میریم:

[ برای مشاهده لینک ، لطفا با نام کاربری خود وارد شوید یا ثبت نام کنید ]

mofidy1
24-07-2009, 20:33
با سلام

عبارات زیر را به دقت مطالعه فرمایید:

«]اولین مرحله در حل یک مساله[ فهمیدن مساله است؛ باید به صورتی آشکار بدانیم که چه چیز خواسته شده است. دوم باید ببینیم که اجزاء مساله چگونه به هم پیوسته اند و ارتباط مجهول با داده ها ی مساله از چه قرار است تا از این راه اندیشه ای در خصوص حل مساله پیدا کنیم و برای آن نقشه ریزی کنیم. سوم اجرای نقشه است. چهارم پس از پایان یافتن مساله به عقب نگاه کردن و تجدید نظر کردن و بحث کردن درباره ی آن است.»

جورج پولیا - چگونه مساله را حل کنیم -ترجمه ی احمد آرام - صفحه ی 7


-------------------------------------------------------

«حل مساله، هنری عملی است. همچون شنا کردن و این هنر را می توان یاد گرفت، تنها به شرطی که از سرمشق خوبی تقلید و دائماً تمرین کنیم.»

جورج پولیا - خلاقیت ریاضی - ترجمه ی پرویز شهریاری - صفحه ی 12


-------------------------------------------------------

«آموختن حل مساله، لذت بخش و ارزشمند است. حل مساله فرایندی است برای ایجاد و گسترش قوای ذهنی، و انبانی از فنون را در اختیار شما قرار می دهد که در مطالعات دیگر و در زندگی روزمره نیز به کارتان می آید...حتماً با معلم و هم کلاسی هایتان درباره ی مسائلی که حل می کنید و فنونی که می آموزید، گفتگو کنید. سوال کردن را یاد بگیرید.»

استیون ج. کرانتس - فنون مساله حل کردن - ترجمه ی مهران اخباریفر - صفحه ی 1


-------------------------------------------------------

«یکی از راه هایی که دانش آموز را به یک مساله کن ماهر تبدیل می کند، این است که از طریق راه بردهای گوناگون به حل مساله بپردازد. عامل دوم، حل کردن تعداد زیادی مساله است. بررسی و بحث در مورد راه حل ها، عامل سومی است که دانش آموز را به مساله حل کن ماهری تبدیل می کند. راه حل را برای دوستان و هم کلاسی های خود توضیح دهید تا مورد نقد و سوال قرا گیرید. راه حل های نادرست را کم اهمیت نشمارید، زیرا گاه نسبت به راه حل های معمولی، حاوی نکات زیبا تر و ارزشمند تری هستند. همیشه پس از این که مساله ای را حل کردید، راه حل آن را با دقت بسیار بنویسید.»

آموزش هنر حل مساله - وزارت آموزش و پرورش - چاپ سال 1379- صفحه ی 3

مطالب بالا سرمشق اصلی ما در مباحث آینده است. چهار کتابی که در بالا از آنها استفاده کردیم، منابع مهم حل مساله هستند که به دوستان خودم تهیه و مطالعه ی آن ها را قویاً سفارش می کنم. البته از کتب دیگری نیز استفاده خواهیم کرد که به مرور معرفی خواهیم کرد.

موفق و پیروز باشید.

2 مرداد 1388

mir@
24-07-2009, 21:09
.
دو نفر در فاصله 20 کیلومتری هم قرار دارند و با سرعت 10 کیلومتر در ساعت به طرف هم حرکت می کنند.

هواپیمایی با سرعت 20 کیلومتر در ساعت، هم زمان با شروع حرکت دو نفر، از نفر اول شروع کرده و به سمت نفر دوم پرواز می کند. پس از رسیدن به نفر دوم بر می گردد و به سمت نفر اول می آید.

این هواپیما آن قدر بین دونفر می رود و می آید تا دو نفر به هم برسند.

این هواپیما مجموعا چند کیلومتر پرواز کرده است؟

chessmathter
25-07-2009, 00:05
.
دو نفر در فاصله 20 کیلومتری هم قرار دارند و با سرعت 10 کیلومتر در ساعت به طرف هم حرکت می کنند.

هواپیمایی با سرعت 20 کیلومتر در ساعت، هم زمان با شروع حرکت دو نفر، از نفر اول شروع کرده و به سمت نفر دوم پرواز می کند. پس از رسیدن به نفر دوم بر می گردد و به سمت نفر اول می آید.

این هواپیما آن قدر بین دونفر می رود و می آید تا دو نفر به هم برسند.

این هواپیما مجموعا چند کیلومتر پرواز کرده است؟
دو نفر در یک ساعت به هم میرسن و هواپیما هم تو این مدت 20 کیلومتر راه رفته

CppBuilder2006
25-07-2009, 07:50
.
دو نفر در فاصله 20 کیلومتری هم قرار دارند و با سرعت 10 کیلومتر در ساعت به طرف هم حرکت می کنند.

هواپیمایی با سرعت 20 کیلومتر در ساعت، هم زمان با شروع حرکت دو نفر، از نفر اول شروع کرده و به سمت نفر دوم پرواز می کند. پس از رسیدن به نفر دوم بر می گردد و به سمت نفر اول می آید.

این هواپیما آن قدر بین دونفر می رود و می آید تا دو نفر به هم برسند.

این هواپیما مجموعا چند کیلومتر پرواز کرده است؟







به این شکل نگاه کنید:
[ برای مشاهده لینک ، لطفا با نام کاربری خود وارد شوید یا ثبت نام کنید ]




هواپیما در لحظۀ t=0 از A شروع می­کنه. فرض کنید فاصلۀ A و B در این لحظه d باشد.

محور x رو در امتداد A و B و به طرف B بگیرید و مبدأ رو A قرار بدید. پس در لحظۀ 0
- همواپیما در x=0
- A در x=0
- B در x=d
قرار دارند و در لحظۀ t،
- A در x=10t
- B در x=d -10t



قرار دارد.
الف) هواپیما در حرکت اول در لحظۀ [ برای مشاهده لینک ، لطفا با نام کاربری خود وارد شوید یا ثبت نام کنید ] به B می­رسد یعنی مسافت [ برای مشاهده لینک ، لطفا با نام کاربری خود وارد شوید یا ثبت نام کنید ] را طی می­کند.




ب) فاصلۀ A و B در لحظۀ [ برای مشاهده لینک ، لطفا با نام کاربری خود وارد شوید یا ثبت نام کنید ]، برابر است با

[ برای مشاهده لینک ، لطفا با نام کاربری خود وارد شوید یا ثبت نام کنید ]
حالا همین مسأله تکرار می­شود این بار با فاصلۀ [ برای مشاهده لینک ، لطفا با نام کاربری خود وارد شوید یا ثبت نام کنید ]




پس هواپیما
در حرکت اول [ برای مشاهده لینک ، لطفا با نام کاربری خود وارد شوید یا ثبت نام کنید ]




در حرکت دوم [ برای مشاهده لینک ، لطفا با نام کاربری خود وارد شوید یا ثبت نام کنید ]
در حرکت سوم [ برای مشاهده لینک ، لطفا با نام کاربری خود وارد شوید یا ثبت نام کنید ]
و در حرکت nام [ برای مشاهده لینک ، لطفا با نام کاربری خود وارد شوید یا ثبت نام کنید ] را طی می­کنه. روی هم رفته
[ برای مشاهده لینک ، لطفا با نام کاربری خود وارد شوید یا ثبت نام کنید ]

طی می­کنه.
امیدوارم اشتباهی نکرده باشم اگه هست بگید.
ولی اگه قوانین نسبیت رو در نظر بگیریم همه چیز فرق می­کنه!

CppBuilder2006
25-07-2009, 08:16
دو نفر در یک ساعت به هم میرسن و هواپیما هم تو این مدت 20 کیلومتر راه رفته

حالا که دوباره فکر می کنم میبینم درسته! خیلی راحت حل میشه! گاهی یه ایدۀ اولیه برای حل جلوی فکر کردن به راه های دیگه رو می گیره!

ali_hp
26-07-2009, 07:44
سلام

دوست عزیزمون chessmathter در پست 10 و 11 مثال نقض هایی که برای حکم توسط خودش و FlightMedia زده شده بودو رد کرد.
saber57 عزیز،دقت کنید که منظور سوال اینه که در شش دقیقه دقیقا ده کیلومتر طی شود،نه اینکه حداقل ده کیلومتر طی شود.
راه حل Iron هم کاملا درست است،فقط یه اشتباه تایپی داره

سلام

اگر x(t)a مسافت طی شده در زمان t (برحسب دقیقه) باشه، f(t)a رو بصورت زیر تعریف می کنیم:

[ برای مشاهده لینک ، لطفا با نام کاربری خود وارد شوید یا ثبت نام کنید ](t)=x(t&plus;10)-x(t)\,&space;\,&space;\,&space;\,&space;\,&space;\,&space;\,&space;\,&space;\,&space;\,&space;0\leq&space;x\leq&space;56

- از آنجاکه x پیوستست، تابع f نیز پیوسته می باشد.
- f نمی تواند همیشه بزرگتر از 10 باشد. چراکه در اینصورت کل مسافت طی شده در یک ساعت میشه:

[ برای مشاهده لینک ، لطفا با نام کاربری خود وارد شوید یا ثبت نام کنید ](0)&plus;f(6)&plus;f(12)&plus;...&plus;f(54)>100

- f نمی تواند همیشه کوچکتر از 10 باشد. چراکه در اینصورت کل مسافت طی شده در یک ساعت میشه:

[ برای مشاهده لینک ، لطفا با نام کاربری خود وارد شوید یا ثبت نام کنید ](0)&plus;f(6)&plus;f(12)&plus;...&plus;f(54)<100

بنابراین تابع f دارای ماکزیمم بزرگتر از 10 و مینیمم کوچکتر از 10 میباشد و از آنجاکه تابعیست پیوسته بنابر این وجود دارد T که رابطه زیر در خصوص آن برقرار باشد:

[ برای مشاهده لینک ، لطفا با نام کاربری خود وارد شوید یا ثبت نام کنید ](T)=10\Rightarrow&space;x(T&plus;6)-x(T)=10
در قسمت اول تعریف f(t)l به صورت زیر است:
[ برای مشاهده لینک ، لطفا با نام کاربری خود وارد شوید یا ثبت نام کنید ](t)=x(t&plus;6)-x(t)
و محدوده x به جای 0 تا 56 باید 0 تا 54 باشد.
راه حل من هم دقیقا همین بود،دقت کنید که در قسمت اخر از قضیه مقدار میانی برای توابع پیوسته استفاده شده است.

ali_hp
26-07-2009, 08:12
الف)بیشترین و کمترین مقدار تابع f رابدست اورید:

[ برای مشاهده لینک ، لطفا با نام کاربری خود وارد شوید یا ثبت نام کنید ](x)=2|x-1|-|x-5|&plus;|3-x|

ب)قرار دهید:

[ برای مشاهده لینک ، لطفا با نام کاربری خود وارد شوید یا ثبت نام کنید ](x)=a_1|x-b_1|&plus;a_2|x-b_2|&plus;...&plus;a_n|x-b_n|

نشان دهید بیشترین و کمترین مقدار f یکی از مقادیر زیر می باشند:

[ برای مشاهده لینک ، لطفا با نام کاربری خود وارد شوید یا ثبت نام کنید ](b_1),f(b_2),...,f(b_n),\lim_{x\to\inft y}{f(x)},\lim_{x\to-\infty}{f(x)}

CppBuilder2006
26-07-2009, 09:43
الف)بیشترین و کمترین مقدار تابع f رابدست اورید:

[ برای مشاهده لینک ، لطفا با نام کاربری خود وارد شوید یا ثبت نام کنید ](x)=2|x-1|-|x-5|&plus;|3-x|

ب)قرار دهید:

[ برای مشاهده لینک ، لطفا با نام کاربری خود وارد شوید یا ثبت نام کنید ](x)=a_1|x-b_1|&plus;a_2|x-b_2|&plus;...&plus;a_n|x-b_n|

نشان دهید بیشترین و کمترین مقدار f یکی از مقادیر زیر می باشند:

[ برای مشاهده لینک ، لطفا با نام کاربری خود وارد شوید یا ثبت نام کنید ](b_1),f(b_2),...,f(b_n),\lim_{x\to\inft y}{f(x)},\lim_{x\to-\infty}{f(x)}

اولی min میشه 2- و max میشه بینهایت.

دومی از این قضیه به دست میاد:
«ماکسیمم و مینیمم هر تابع در نقاط بحرانی حاصل میشوند»

saber57
26-07-2009, 14:48
سلام

دوست عزیزمون chessmathter در پست 10 و 11 مثال نقض هایی که برای حکم توسط خودش و FlightMedia زده شده بودو رد کرد.
saber57 عزیز،دقت کنید که منظور سوال اینه که در شش دقیقه دقیقا ده کیلومتر طی شود،نه اینکه حداقل ده کیلومتر طی شود.
راه حل Iron هم کاملا درست است،فقط یه اشتباه تایپی داره

در قسمت اول تعریف f(t)l به صورت زیر است:
[ برای مشاهده لینک ، لطفا با نام کاربری خود وارد شوید یا ثبت نام کنید ](t)=x(t&plus;6)-x(t)
و محدوده x به جای 0 تا 56 باید 0 تا 54 باشد.
راه حل من هم دقیقا همین بود،دقت کنید که در قسمت اخر از قضیه مقدار میانی برای توابع پیوسته استفاده شده است.

با تشکر
من اصل لانه کبوتری رو برای این سوالتون تصحیح میکنم . امیدوارم جواب صحیح باشه . یه بار دیگه اصل لانه کبوتری رو مینویسم :

اصل لانه کبوتری دیریکله :
اگر m کبوتر بخواهند n لانه را اشغال کنند و m>=n انگاه حداقل یکی از لانه ها شامل دو یا تعداد بیشتری کبوتر خواهد بود .

اصل لانه کبوتری در حالت کلی : اگر kn+1 کبوتر یا بیشتر n لانه را اشغال کنند آنگاه در حداقل یکی از لانه ها بیش از k کبوتر (حداقل k+1) خواهد بود . البته قضیه زیر مکمل این اصل هست.

قضیه : اگر m کبوتر n لانه را اشغال کنند و تعداد کبوتران بیش از تعداد لانه ها باشد، آنگاه در یکی از لانه ها حداقل [ برای مشاهده لینک ، لطفا با نام کاربری خود وارد شوید یا ثبت نام کنید ][&space;\frac{m-1}{n}&space;\right&space;]&plus;1 کبوتر وجود دارد.

کاربرد در حل مساله : زمان یک ساعت یا 60 دقیقه را تبدیل به 10 بازه 6 دقیقه ای میکنیم . تعداد بازه ها را لانه و مسافت 100 کیلومتر را برابر با 100 کبوتر فرض میکنیم . حالا میخواهیم ببینیم در هر لانه یا 6 دقیقه حداقل چند کیلومتر پیموده شده وجود دارد . بنا به این اصل :

[ برای مشاهده لینک ، لطفا با نام کاربری خود وارد شوید یا ثبت نام کنید ][&space;\frac{100-1}{10}&space;\right&space;]&plus;1=9&plus;1=10

نکته مهم : در اصل لانه کبوتری انتخاب نوع لانه و کبوتر بر اساس سوال خیلی مهم هست .
4/5/88

ali_hp
26-07-2009, 16:01
اگر m کبوتر n لانه را اشغال کنند و تعداد کبوتران بیش از تعداد لانه ها باشد، آنگاه بنا به این اصل هر لانه شامل حداقل [ برای مشاهده لینک ، لطفا با نام کاربری خود وارد شوید یا ثبت نام کنید ][&space;\frac{m-1}{n}&space;\right&space;]&plus;1 کبوتر یا بیشتر است .

سلام
از اصل لانه کبو تری نتیجه می شود که حداقل یک لانه وجود دارد که حداقل شامل [ برای مشاهده لینک ، لطفا با نام کاربری خود وارد شوید یا ثبت نام کنید ][&space;\frac{m-1}{n}&space;\right&space;]&plus;1 کبوتر باشد.شما باا ستفاده از اصل لانه کبوتری می توانید ثابت کنید که در یک بازه شش دقیقه ای ای حداقل ده کیلومتر طی شده است،ولی فکر نمی کنم بتوانید ثابت کنید که بازه ای شش دقیقه ای وجود دارد که دقیقا ده کیلومتر در ان طی شده باشد.

saber57
26-07-2009, 16:12
سلام
از اصل لانه کبو تری نتیجه می شود که حداقل یک لانه وجود دارد که حداقل شامل [ برای مشاهده لینک ، لطفا با نام کاربری خود وارد شوید یا ثبت نام کنید ][&space;\frac{m-1}{n}&space;\right&space;]&plus;1 کبوتر باشد.شما باا ستفاده از اصل لانه کبوتری می توانید ثابت کنید که در یک بازه شش دقیقه ای ای حداقل ده کیلومتر طی شده است،ولی فکر نمی کنم بتوانید ثابت کنید که بازه ای شش دقیقه ای وجود دارد که دقیقا ده کیلومتر در ان طی شده باشد.
خب کبوترها به تعداد حداقل 10 تا در لانه ها قرار میگیرند تا 10 لانه پر شوند . 10 تا 10 کبوتر میشه 100 تا دیگه!!! پس همین حداقل همون حداکثر هم هست و در هر لانه بیشتر 1ز 10 تا قرار نمیتونه هم بگیره . این اصل میگه اول کبوترها به صورت حداقل در تمام لانه ها قرار میگیرند بعد که پر شدند دوباره سری دوم پر میشه
(با این فرض که هر لانه یک بازه 6 دقیقه ای و هر کبوتر یک کیلومتر در نظر گرفته باشه.
موفق باشید :20:

CppBuilder2006
27-07-2009, 06:01
خب کبوترها به تعداد حداقل 10 تا در لانه ها قرار میگیرند :20:
وارنا (ببخشید برای دخالت)
وارنا (ولی «لانه ها» نه، حداقل یک لانه.)
وارنا ( حداقل یک لانه هست که حداقل 10 تا کبوتر داشته باشه.)
.
.

saber57
27-07-2009, 07:40
البته من از ریاضیات گسسته برای حل مساله کمک گرفتم و مسافت گسسته فرض شد ولی مسافت پیوسته هست

chessmathter
27-07-2009, 10:31
سلام
از اصل لانه کبو تری نتیجه می شود که حداقل یک لانه وجود دارد که حداقل شامل [ برای مشاهده لینک ، لطفا با نام کاربری خود وارد شوید یا ثبت نام کنید ] کبوتر باشد.شما باا ستفاده از اصل لانه کبوتری می توانید ثابت کنید که در یک بازه شش دقیقه ای ای حداقل ده کیلومتر طی شده است،ولی فکر نمی کنم بتوانید ثابت کنید که بازه ای شش دقیقه ای وجود دارد که دقیقا ده کیلومتر در ان طی شده باشد.


خب کبوترها به تعداد حداقل 10 تا در لانه ها قرار میگیرند تا 10 لانه پر شوند . 10 تا 10 کبوتر میشه 100 تا دیگه!!! پس همین حداقل همون حداکثر هم هست و در هر لانه بیشتر 1ز 10 تا قرار نمیتونه هم بگیره . این اصل میگه اول کبوترها به صورت حداقل در تمام لانه ها قرار میگیرند بعد که پر شدند دوباره سری دوم پر میشه
(با این فرض که هر لانه یک بازه 6 دقیقه ای و هر کبوتر یک کیلومتر در نظر گرفته باشه.
موفق باشید :20:
این راه حل که گفتی اصلا معنا نداره تو تویه یه چیز پیوسته چه طور لانه کفتری میری حالا اون هیچ این 10بازه یا 10لانه چه جوریه به ترتیب یعنی 1 تا 6 6 تا 12 ... این 6 دقیقه ممکنه بازه ی 5 تا 11 باشه اگه منظورت این نیست زمان پیوستس تو که نمیتونه هرجو ر خاصی به 10 قسمت کنی (4 تا 10 یکی بعد 1 تا 4 و 10 تا 12 یک بازه!) بعد وقتی 1 کیلومترها رو (کبوترها رو تقسیم میکنی ممکنه تو 9 تا هیچی نباشه تو یکی 100

yugioh
27-07-2009, 21:38
دفعه قبل جواب ندادید. همون تکرار میشه. اگه این بار هم کسی جواب نداد جدید سوال می دم.
درون یک کیسه 25 گوی یک شکل هست، شامل 6 قرمز 9 سبز 10 سفید. دو نفر بازی زیر رو انجام می دهند،
اولی یک گوی بیرون میاره. اگر سبز باشه برنده است اگر قرمز یا سفید باشه بازنده.
دومی یک گوی رو بیرون میاره، اگر قرمز باشه برنده است. اگر سبز باشه بازنده است. اگر سفید باشه اون گوی رو خارج می کنه و از گویهای باقی مانده یکی دیگه رو خارج می کنه. ( اگر 10 بار هم پست سر هم گوی سفید بیرون میاد.
اگر هر دو تا برنده یا بازنده شوند بازی برابر حساب شده و مجددا بازی می کنند، تا نهایتا یک نفر برندده بشه.
اگر شما جایگزین این دو نفر بودید به جای کدام یکی بازی می کردید؟

حالا یه سوال دیگه از این گوی ها، 7 تا از گویها روخارج می کنیم،اگر بدونیم توی انتخابیها، سفید وجود داره با چه احتمالی حداقل 1 قرمز و 2 سبز داره؟

CppBuilder2006
28-07-2009, 10:10
درون یک کیسه 25 گوی یک شکل هست، شامل 6 قرمز 9 سبز 10 سفید. دو نفر بازی زیر رو انجام می دهند،
اولی یک گوی بیرون میاره. اگر سبز باشه برنده است اگر قرمز یا سفید باشه بازنده.
دومی یک گوی رو بیرون میاره، اگر قرمز باشه برنده است. اگر سبز باشه بازنده است. اگر سفید باشه اون گوی رو خارج می کنه و از گویهای باقی مانده یکی دیگه رو خارج می کنه. ( اگر 10 بار هم پست سر هم گوی سفید بیرون میاد.
اگر هر دو تا برنده یا بازنده شوند بازی برابر حساب شده و مجددا بازی می کنند، تا نهایتا یک نفر برندده بشه.
اگر شما جایگزین این دو نفر بودید به جای کدام یکی بازی می کردید؟

این سوال به نظر یه خرده مبهمه!
اگه یکی بازنده باشد معنیش اینه که اون یکی برنده س و بازی تمومه پس این یعنی چی:

اگر هر دو تا برنده یا بازنده شوند

این برداشتن با جایگذاری هست یا نه؟


حالا یه سوال دیگه از این گوی ها، 7 تا از گویها روخارج می کنیم،اگر بدونیم توی انتخابیها، سفید وجود داره با چه احتمالی حداقل 1 قرمز و 2 سبز داره؟
خب اول یه سفید بر میداریم.
حالا توی کیسه
6 قرمز
9 سبز
9 سفید
داریم. از این کیسه 6 گوی به تصادف انتخاب می کنیم.
تعدا کل حالات میشه 19*20*21*22*23*24
تعداد حالاتی که حداقل 1 قرمز و 2 سبز باشه برابره با 19*20*21*8*9*6
پس احتمال میشه دومی تقسیم بر اولی.
نمی دونم شاید اشتباه میکنم!
.
.

yugioh
28-07-2009, 11:50
این سوال به نظر یه خرده مبهمه!
اگه یکی بازنده باشد معنیش اینه که اون یکی برنده س و بازی تمومه پس این یعنی چی:


این برداشتن با جایگذاری هست یا نه؟


خب اول یه سفید بر میداریم.
حالا توی کیسه
6 قرمز
9 سبز
9 سفید
داریم. از این کیسه 6 گوی به تصادف انتخاب می کنیم.
تعدا کل حالات میشه 19*20*21*22*23*24
تعداد حالاتی که حداقل 1 قرمز و 2 سبز باشه برابره با 19*20*21*8*9*6
پس احتمال میشه دومی تقسیم بر اولی.
نمی دونم شاید اشتباه میکنم!
.
.

ممنون از جوابت ببین من میگم: اگه هر دو مثلا به وضعیت باخت یا هر دو به برد رسیدند مسابقه از نو انجام میشه. مثلا سبزه سبز وقرمزه قرمز دراورد.
اول یکی تا جاییکه برنده یا بازنده شه، ادامه می ده بعد گویها بر می گردند و دومی تا برد یا باخت ادامه می ده اگر ذقیقا یک نفر به هدفش رسید مثلا سبزه سبز دراورد اون برنده است. اگه هر دو به هدف رسیدند یا هیچ یک هر دو برابرند و دوباره از نو مسابقه می دن.
در مورد جایگزاری یکی شون فقط یه گوی بر میداره و برای اون یکی که ممکنه بیش از یکی برداره نه خیر با جایگذاری نیست گوی خارج شده بر گردونده نمیشه.
حالا احتمال رسیدن به هدف برای کدوم بیشتره؟
در مورد دوم هم اشتباه اینه که گویهای قرمز مثل هم اند 24 * 23 و اینا نمیشه. البته ایده درسته. راه دیگه هم استفاده از احتمال شرطی و قضیه بیزه.

عذر می خوام اگه سوال بدی. امیدوارم ببخشید دفعه بعد جبران می کنم.
از همه دوستانی که جواب میدن یا نمی دن هم ممنونم.

davy jones
28-07-2009, 13:30
دفعه قبل جواب ندادید. همون تکرار میشه. اگه این بار هم کسی جواب نداد جدید سوال می دم.
درون یک کیسه 25 گوی یک شکل هست، شامل 6 قرمز 9 سبز 10 سفید. دو نفر بازی زیر رو انجام می دهند،
اولی یک گوی بیرون میاره. اگر سبز باشه برنده است اگر قرمز یا سفید باشه بازنده.
دومی یک گوی رو بیرون میاره، اگر قرمز باشه برنده است. اگر سبز باشه بازنده است. اگر سفید باشه اون گوی رو خارج می کنه و از گویهای باقی مانده یکی دیگه رو خارج می کنه. ( اگر 10 بار هم پست سر هم گوی سفید بیرون میاد.
اگر هر دو تا برنده یا بازنده شوند بازی برابر حساب شده و مجددا بازی می کنند، تا نهایتا یک نفر برندده بشه.
اگر شما جایگزین این دو نفر بودید به جای کدام یکی بازی می کردید؟

حالا یه سوال دیگه از این گوی ها، 7 تا از گویها روخارج می کنیم،اگر بدونیم توی انتخابیها، سفید وجود داره با چه احتمالی حداقل 1 قرمز و 2 سبز داره؟

احتمال برنده شدن اولي در دور اول برابر است با 9 تقسيم بر 25 يعني 0.36
احتمال برنده شدن دومي در دور اول برابر با است با:

[ برای مشاهده لینک ، لطفا با نام کاربری خود وارد شوید یا ثبت نام کنید ] B25%7D\cdot%5Cfrac%7B6%7D%7B24%7D&plus;%5Cfrac%7B10%7D% 7B25%7D\cdot%5Cfrac%7B9%7D%7B24%7D\cdot%5Cfrac%7B6 %7D%7B23%7D&plus;...&plus;%5Cfrac%7B10%21\cdot15%21%7D%7B25% 21%7D\cdot%5Cfrac%7B6%7D%7B15%7D%5Csimeq&space;0.4

در صورتي بازي به دور دوم كشيده ميشه كه هر دو ببرن يا هر دو ببازن. پس احتمال به دور دوم رفتن برابر است با:

[ برای مشاهده لینک ، لطفا با نام کاربری خود وارد شوید یا ثبت نام کنید ]

چون مراحل بازي در صورت رسيدن به دور دوم تكرار ميشه پس احتمال برنده شدن در دور دوم هم مثل دور اوله:

نفر اول : 0.36
نفر دوم : 0.4

كه هر كدام از اينها بايد در احتمال اينكه بازي به دور دوم بكشه نيز ضرب بشه:

نفر اول در دور دوم : [ برای مشاهده لینک ، لطفا با نام کاربری خود وارد شوید یا ثبت نام کنید ]

نفر دوم در دور دوم: [ برای مشاهده لینک ، لطفا با نام کاربری خود وارد شوید یا ثبت نام کنید ]

همونطور كه انتظار ميرفت احتمال برنده شدن نفر دوم در اين مرحله نيز بيشتره.
با توجه به اينكه در هر مرحله يه عدد ثابت در هر كدوم از احتمالات افراد ضرب ميشه پس همواره احتمال برنده شدن نفر دوم در هر مرحله بيشتره.

البته اگه بخوايم تصميم بگيريم كه جاي كدومشون بازي كنيم در كل شانس بيشتري داريم بايد اين طور نگاه كنيم كه در صورتي من برنده خواهم شد ( حالا تو هر مرحله اي) كه رقيب شكست بخوره. پس احتمال برنده شدن نفر اول در كل مسابقه به اين قراره كه يا تو مرحله اول ميبره و نفر دوم ميبازه يا اينكه هر دو مرحله اول رو ميبرن و در مرحله دوم نفر اول ميبره و دومي ميبازه و .... كه داريم:

[ برای مشاهده لینک ، لطفا با نام کاربری خود وارد شوید یا ثبت نام کنید ] .528%5En=%5Cfrac%7B0.216%7D%7B1-0.528%7D%5Csimeq&space;0.457

و در مورد نفر دوم هم به همين صورت:


[ برای مشاهده لینک ، لطفا با نام کاربری خود وارد شوید یا ثبت نام کنید ] 0.528%5En=%5Cfrac%7B0.256%7D%7B1-0.528%7D%5Csimeq&space;0.542

كه باز هم مشاهده ميشود كه شانس برنده شدن نفر دوم در كل بازي هم بيشتر است. (البته به طور شهودي و با توجه به نتايج قسمت قبل واضح بود كه شانس بردن كل مسابقه براي هر كسي كه در تك تك مراحل، شانس بيشتري دارد، بيشتر است.)
پس اگر شما بخواهيد جاي يكي از اين دو بازي كنيد ترجيه ميدهيد كه جاي نفر دوم بازي كنيد.

-----------------------

در مورد سوال دوم، به زبون رياضي منظور اين سوال اينه كه در برداشتن 7 گوي:

[ برای مشاهده لینک ، لطفا با نام کاربری خود وارد شوید یا ثبت نام کنید ] geq&space;1%29=?

عبارت بالا طبق رابطه احتمال شرطي برابر است با:

[ برای مشاهده لینک ، لطفا با نام کاربری خود وارد شوید یا ثبت نام کنید ] white%5Cgeq&space;1%29%7D%7BP%28white%5Cgeq&space;1%29%7D

اما هر كدام از احتمالهاي صورت و مخرج رو ميشه از مكملشون حساب كرد:

صورت:

[ برای مشاهده لینک ، لطفا با نام کاربری خود وارد شوید یا ثبت نام کنید ] q&space;1%29=1-&space;P%28red%3C1,green%5Cgeq&space;2,white%5Cgeq&space;1%29
[ برای مشاهده لینک ، لطفا با نام کاربری خود وارد شوید یا ثبت نام کنید ]
[ برای مشاهده لینک ، لطفا با نام کاربری خود وارد شوید یا ثبت نام کنید ] &plus;&space;P%28red%3C1,green%5Cgeq&space;2,white%3C1%29
[ برای مشاهده لینک ، لطفا با نام کاربری خود وارد شوید یا ثبت نام کنید ]

(حوصله ندارم حساب كنم:31:)


و مخرج:

[ برای مشاهده لینک ، لطفا با نام کاربری خود وارد شوید یا ثبت نام کنید ]


و با جايگذاري جواب كل بدست ميآد.


موفق باشين.
88/5/6

yugioh
28-07-2009, 13:45
احتمال برنده شدن اولي در دور اول برابر است با 9 تقسيم بر 25 يعني 0.36
احتمال برنده شدن دومي در دور اول برابر با است با:

[ برای مشاهده لینک ، لطفا با نام کاربری خود وارد شوید یا ثبت نام کنید ] B25%7D*%5Cfrac%7B6%7D%7B24%7D&plus;%5Cfrac%7B10%7D%7B25 %7D*%5Cfrac%7B9%7D%7B24%7D*%5Cfrac%7B6%7D%7B23%7D&plus; ...&plus;%5Cfrac%7B10%21*15%21%7D%7B25%21%7D*%5Cfrac%7B 6%7D%7B15%7D%5Csimeq&space;0.4

در صورتي بازي به دور دوم كشيده ميشه كه هر دو ببرن يا هر دو ببازن. پس احتمال به دور دوم رفتن برابر است با:

[ برای مشاهده لینک ، لطفا با نام کاربری خود وارد شوید یا ثبت نام کنید ]*0.36&plus;%281-0.4%29*%281-0.36%29=0.528

چون مراحل بازي در صورت رسيدن به دور دوم تكرار ميشه پس احتمال برنده شدن در دور دوم هم مثل دور اوله:

نفر اول : 0.36
نفر دوم : 0.4

كه هر كدام از اينها بايد در احتمال اينكه بازي به دور دوم بكشه نيز ضرب بشه:

نفر اول در دور دوم : [ برای مشاهده لینک ، لطفا با نام کاربری خود وارد شوید یا ثبت نام کنید ]*0.36=0.19008

نفر دوم در دور دوم: [ برای مشاهده لینک ، لطفا با نام کاربری خود وارد شوید یا ثبت نام کنید ]*0.4=0.2112

همونطور كه انتظار ميرفت احتمال برنده شدن نفر دوم در اين مرحله نيز بيشتره.
با توجه به اينكه در هر مرحله يه عدد ثابت در هر كدوم از احتمالات افراد ضرب ميشه پس همواره احتمال برنده شدن نفر دوم در هر مرحله بيشتره.

البته اگه بخوايم تصميم بگيريم كه جاي كدومشون بازي كنيم در كل شانس بيشتري داريم بايد اين طور نگاه كنيم كه در صورتي من برنده خواهم شد ( حالا تو هر مرحله اي) كه رقيب شكست بخوره. پس احتمال برنده شدن نفر اول در كل مسابقه به اين قراره كه يا تو مرحله اول ميبره و نفر دوم ميبازه يا اينكه هر دو مرحله اول رو ميبرن و در مرحله دوم نفر اول ميبره و دومي ميبازه و .... كه داريم:

[ برای مشاهده لینک ، لطفا با نام کاربری خود وارد شوید یا ثبت نام کنید ]*%281-0.4%29&plus;0.528*0.36*%281-0.4%29&plus;0.528%5E2*0.36%281-0.4%29&plus;...=0.216%5Csum_%7B0%7D%5E%7B%5Cinfty&space;%7D&space;0 .528%5En=%5Cfrac%7B0.216%7D%7B1-0.528%7D%5Csimeq&space;0.457

و در مورد نفر دوم هم به همين صورت:


[ برای مشاهده لینک ، لطفا با نام کاربری خود وارد شوید یا ثبت نام کنید ]*%281-0.36%29&plus;0.528*0.4*%281-0.36%29&plus;0.528%5E2*0.4%281-0.36%29&plus;...=0.256%5Csum_%7B0%7D%5E%7B%5Cinfty&space;%7D&space; 0.528%5En=%5Cfrac%7B0.256%7D%7B1-0.528%7D%5Csimeq&space;0.542

كه باز هم مشاهده ميشود كه شانس برنده شدن نفر دوم در كل بازي هم بيشتر است. (البته به طور شهودي و با توجه به نتايج قسمت قبل واضح بود كه شانس بردن كل مسابقه براي هر كسي كه در تك تك مراحل، شانس بيشتري دارد، بيشتر است.)
پس اگر شما بخواهيد جاي يكي از اين دو بازي كنيد ترجيه ميدهيد كه جاي نفر دوم بازي كنيد.

-----------------------

در مورد سوال دوم، به زبون رياضي منظور اين سوال اينه كه در برداشتن 7 گوي:

[ برای مشاهده لینک ، لطفا با نام کاربری خود وارد شوید یا ثبت نام کنید ] geq&space;1%29=?

عبارت بالا طبق رابطه احتمال شرطي برابر است با:

[ برای مشاهده لینک ، لطفا با نام کاربری خود وارد شوید یا ثبت نام کنید ] white%5Cgeq&space;1%29%7D%7BP%28white%5Cgeq&space;1%29%7D

اما هر كدام از احتمالهاي صورت و مخرج رو ميشه از مكملشون حساب كرد:

صورت:

[ برای مشاهده لینک ، لطفا با نام کاربری خود وارد شوید یا ثبت نام کنید ] q&space;1%29=1-&space;P%28red%3C1,green%5Cgeq&space;2,white%5Cgeq&space;1%29
[ برای مشاهده لینک ، لطفا با نام کاربری خود وارد شوید یا ثبت نام کنید ]
[ برای مشاهده لینک ، لطفا با نام کاربری خود وارد شوید یا ثبت نام کنید ] &plus;&space;P%28red%3C1,green%5Cgeq&space;2,white%3C1%29
[ برای مشاهده لینک ، لطفا با نام کاربری خود وارد شوید یا ثبت نام کنید ]

(حوصله ندارم حساب كنم:31:)


و مخرج:

[ برای مشاهده لینک ، لطفا با نام کاربری خود وارد شوید یا ثبت نام کنید ]


و با جايگذاري جواب كل بدست ميآد.


موفق باشين.
88/5/6

این latex ات رو یک کاری کن همه ببینند. ببین این راخت غلط نیست ولی مساله اینه که سری توانی نمی شه . چرا؟ چون گوی سفید خارج شده بر گردونده نمیشه پس کسر عوض میشه.
در حالتی که گوی برگردونده میشه رو نوشتی. در حالت دیگه با خارج کردن گوی احتمال بردش در گویی گکه بعدش بیرون میاره بیشتر میشه. ولی احتمال باختش در اون گوی هم بیشتر میشه چون احتمال لزوم دوباره گوی در آوردن کم میشه. پس شرمنده دوباره حساب کن :27:بگو. این اند سرکاری میشه حسابش با دست. 10 11 تا پرانتز تو هم.
در مورد دو هم درسته.

CppBuilder2006
28-07-2009, 14:07
در مورد دوم هم اشتباه اینه که گویهای قرمز مثل هم اند 24 * 23 و اینا نمیشه. البته ایده درسته.




خب سعی میکنم جوامبو تصحیح کنم
اول یه سفید بر میداریم.



حالا توی کیسه
6 قرمز
9 سبز
9 سفید
داریم. از این کیسه 6 گوی به تصادف انتخاب می کنیم.
تعدا کل حالات میشه

[ برای مشاهده لینک ، لطفا با نام کاربری خود وارد شوید یا ثبت نام کنید ]



تعداد حالاتی که حداقل 1 قرمز و 2 سبز باشه برابره با
[ برای مشاهده لینک ، لطفا با نام کاربری خود وارد شوید یا ثبت نام کنید ]





پس احتمال میشه دومی تقسیم بر اولی:
[[ برای مشاهده لینک ، لطفا با نام کاربری خود وارد شوید یا ثبت نام کنید ]



خب الان درسته؟

CppBuilder2006
28-07-2009, 14:14
... اولی رو بر دومی تقیسم کردم انگار!:13:

sherlockholmz
28-07-2009, 14:40
با سلام
دريك كيسه 9 مهره سبز و6 مهره سفيد و در كيسه ديگر 7 مهره سبز و 7 مهره سفيد داريم.يكي از كيسه ها را به تصادف انتخاب كرده از داخل آن 5 مهره به تصادف بيرون آورده و بدون نگاه كردن در كيسه دوم مي ريزيم.حال اينبار از كيسه ديگر 5 مهره انتخاب كرده و باز بدون نگاه كردن در كيسه اولي مي ريزيم.اين عمل را 3 بار انجام مي دهيم.
حالا از هر كيسه يك مهره بيرون مي آوريم، احتمال غير همرنگ بودن آنها چيست؟

davy jones
28-07-2009, 15:01
این latex ات رو یک کاری کن همه ببینند. ببین این راخت غلط نیست ولی مساله اینه که سری توانی نمی شه . چرا؟ چون گوی سفید خارج شده بر گردونده نمیشه پس کسر عوض میشه.
در حالتی که گوی برگردونده میشه رو نوشتی. در حالت دیگه با خارج کردن گوی احتمال بردش در گویی گکه بعدش بیرون میاره بیشتر میشه. ولی احتمال باختش در اون گوی هم بیشتر میشه چون احتمال لزوم دوباره گوی در آوردن کم میشه. پس شرمنده دوباره حساب کن :27:بگو. این اند سرکاری میشه حسابش با دست. 10 11 تا پرانتز تو هم.
در مورد دو هم درسته.

كاملا هم سري تواني ميشه! چون برنگردوندن گويها فقط موقعيه كه نفر دوم داره بازي ميكنه. ولي اگه بازي به دور دوم بكشه همه ي گويها سر جاشون ريخته ميشه و بازي از اول تكرار ميشه. پس سري توانيه. مگه اينكه من بد متوجه شده باشم و اصل سوال اين نباشه.
در ضمن كدهاي لاتكس هم كه نميتونين ببينين چيز زياد مهمي نبود. تو اولي احتمال برنده شدن نفر دوم در دور اول رو محاسبه كرده بودم كه شدش: 0.4
و در بعدي هم احتمال اينكه بازي به دور بعد بره رو محاسبه كرده بودم كه شده بود : 0.528
بعدشم اين 0.528 رو در احتمال برنده شدن افراد در دور اول ضرب كرده بودم كه به ترتيب شدش: 0.216 و 0.256
دو قسمت آخر هم مربوط به همون سري توانيه كه دوستمون گفت (اصلا حوصله ي دوباره ي تايپ رياضي رو ندارم:31:)

موفق باشين.
88/5/6

davy jones
28-07-2009, 15:06
با سلام
دريك كيسه 9 مهره سبز و6 مهره سفيد و در كيسه ديگر 7 مهره سبز و 7 مهره سفيد داريم.يكي از كيسه ها را به تصادف انتخاب كرده از داخل آن 5 مهره به تصادف بيرون آورده و بدون نگاه كردن در كيسه دوم مي ريزيم.حال اينبار از كيسه ديگر 5 مهره انتخاب كرده و باز بدون نگاه كردن در كيسه اولي مي ريزيم.اين عمل را 3 بار انجام مي دهيم.
حالا از هر كيسه يك مهره بيرون مي آوريم، احتمال غير همرنگ بودن آنها چيست؟

يه سوال(زياد جدي نگير:27:): مگه مريضيم كه 3 دفعه 5 تا مهره رو از اين كيسه به اون كيسه كنيم؟:18:
اگه وقت اضافي داري خوب شبها پاشو برو ستاره ها رو بشمار:31:

موفق باشين.
88/5/6

yugioh
28-07-2009, 18:52
كاملا هم سري تواني ميشه! چون برنگردوندن گويها فقط موقعيه كه نفر دوم داره بازي ميكنه. ولي اگه بازي به دور دوم بكشه همه ي گويها سر جاشون ريخته ميشه و بازي از اول تكرار ميشه. پس سري توانيه. مگه اينكه من بد متوجه شده باشم و اصل سوال اين نباشه.
در ضمن كدهاي لاتكس هم كه نميتونين ببينين چيز زياد مهمي نبود. تو اولي احتمال برنده شدن نفر دوم در دور اول رو محاسبه كرده بودم كه شدش: 0.4
و در بعدي هم احتمال اينكه بازي به دور بعد بره رو محاسبه كرده بودم كه شده بود : 0.528
بعدشم اين 0.528 رو در احتمال برنده شدن افراد در دور اول ضرب كرده بودم كه به ترتيب شدش: 0.216 و 0.256
دو قسمت آخر هم مربوط به همون سري توانيه كه دوستمون گفت (اصلا حوصله ي دوباره ي تايپ رياضي رو ندارم:31:)

موفق باشين.
88/5/6

شرمنده شاید من چون نوشتمش بهتر بدونم:31: kidding
ببین این که میگه ایمجوریه مثلا ممکن پشت سرهم 5 سفید بیاره خوب بر که نمی گردونه در عوض احتمال در اومئن گوی سفید هر بار نسبت به قبل کمتر میشه. اول 10/25 بعد 9/24 بعد 8/23 و....
ببین این واضحه که اگر کسی شانس بیشتری در دور اول داشته باش اون خوبه. دور اول نه انتخاب اولین گویش. مثلا اونی که قرمزه در دور اولش 7 تا در میاره تا به سبز یا قرمز برسه. این دور اولشه. حالا در صورتی که اولین مهره اش یفید باشه برای دور بعد باز احتمال عوض میشه تا...
ببین این سوال راهش ساده ولی به شدت ضایع است ( همین راهی که می گید با منظور اصلی سوال که من بد توضیح دادم.) میشه 10 تا پرانتز.
ببخشید توضیح بد من باعث سردرگمی تون شد. عذر می خوام نیت بدی نداشتم. ایشالله دفعه بعدی شرمنده :11:نشم.:11:

eh_mn
29-07-2009, 09:50
ثابت كنيد كه اگر [ برای مشاهده لینک ، لطفا با نام کاربری خود وارد شوید یا ثبت نام کنید ]{1}{a} يك عدد صحيح باشد آنگاه براي هر n ،[ برای مشاهده لینک ، لطفا با نام کاربری خود وارد شوید یا ثبت نام کنید ]^n&plus;\frac{1}{a^n} نيز عددي صحيح است.

ـــــــــــــــــــــــــ
31 / 04 / 88

هر دو روشي كه saber57 در پست

برای مشاهده محتوا ، لطفا وارد شوید یا ثبت نام کنید
ارائه كردند كاملاً صحيح است. فقط اين نكته رو اضافه مي‏كنم كه در روش اول هم به نوعي از استقراي قوي استفاده شده است.

استقراي قوي
فرض كنيم P گزاره‏اي در مورد عدد طبيعي n باشد و
(آ) P براي n=1 درست باشد،
(ب) از درستي P براي 1، 2، ...، k درستي آن براي k+1 نتيجه مي‏شود
در اينصورت P براي همه‏ي اعداد طبيعي درست است.


براي بدست آوردن يك ايده براي حل مساله، دست بكار شويد!

براي آنكه الگوي مناسب را بيابيد، مثال سازي كنيد يا براي چند عدد كوچك مساله را بازنويسي كنيد.



ـــــــــــــــــــــ
7 / 05 / 88

eh_mn
29-07-2009, 10:01
فرض كنيم A و B دو زيرمجموعه‏ي متناهي و مجزا از اعداد حقيقي باشند به طوري كه به ازاي هر [ برای مشاهده لینک ، لطفا با نام کاربری خود وارد شوید یا ثبت نام کنید ] داشته باشيم

[ برای مشاهده لینک ، لطفا با نام کاربری خود وارد شوید یا ثبت نام کنید ] يا [ برای مشاهده لینک ، لطفا با نام کاربری خود وارد شوید یا ثبت نام کنید ]

ثابت كنيد تعداد اعضاي A دو برابر تعداد اعضاي B است.


ـــــــــــــــــ
7 / 05 / 88

davy jones
29-07-2009, 12:02
فرض كنيم A و B دو زيرمجموعه‏ي متناهي و مجزا از اعداد حقيقي باشند به طوري كه به ازاي هر [ برای مشاهده لینک ، لطفا با نام کاربری خود وارد شوید یا ثبت نام کنید ] داشته باشيم

[ برای مشاهده لینک ، لطفا با نام کاربری خود وارد شوید یا ثبت نام کنید ] يا [ برای مشاهده لینک ، لطفا با نام کاربری خود وارد شوید یا ثبت نام کنید ]

ثابت كنيد تعداد اعضاي A دو برابر تعداد اعضاي B است.


ـــــــــــــــــ
7 / 05 / 88

فکر کنم سوال یه کم کژتابی داره. هر عددی که عضو A باشه مسلما عضو اجتماع A و B هم هست. بنابراین به راحتی میشه نشون داد که همه ی اعداد صحیح عضو A هستن. در حالیکه اگه از هر کدوم از اونها 2 تا کم کنیم عضو مجموعه B میشن. اینطوری همه ی اعداد صحیح هم عضو A و هم عضو B میشن. درحالیکه تو صورت سوال گفته این دو مجموعه از هم مجزا هستن.
یکم بیشتر در مورد سوال توضیح بده.

موفق باشین.
88/5/7

CppBuilder2006
29-07-2009, 12:28
فرض كنيم A و B دو زيرمجموعه‏ي متناهي و مجزا از اعداد حقيقي باشند به طوري كه به ازاي هر [ برای مشاهده لینک ، لطفا با نام کاربری خود وارد شوید یا ثبت نام کنید ] داشته باشيم

[ برای مشاهده لینک ، لطفا با نام کاربری خود وارد شوید یا ثبت نام کنید ] يا [ برای مشاهده لینک ، لطفا با نام کاربری خود وارد شوید یا ثبت نام کنید ]

ثابت كنيد تعداد اعضاي A دو برابر تعداد اعضاي B است.

باید سوال المپیادی باشه. سخته! سعی میکنم حل کنم!

eh_mn
29-07-2009, 13:14
فکر کنم سوال یه کم کژتابی داره. هر عددی که عضو A باشه مسلما عضو اجتماع A و B هم هست. بنابراین به راحتی میشه نشون داد که همه ی اعداد صحیح عضو A هستن. در حالیکه اگه از هر کدوم از اونها 2 تا کم کنیم عضو مجموعه B میشن. اینطوری همه ی اعداد صحیح هم عضو A و هم عضو B میشن. درحالیکه تو صورت سوال گفته این دو مجموعه از هم مجزا هستن.
یکم بیشتر در مورد سوال توضیح بده.

موفق باشین.
88/5/7

ميشه بگين چطور ميشه ثابت كرد كه A همه‏ي اعداد صحيح رو داره؟

به نظرتون كدوم قسمتش مبهمه؟ اگه ممكنه بگين تا توضيح بدم.


توجه كنيد كه

فرض كنيم A و B دو زيرمجموعه‏ي متناهي و مجزا از ...

موفق باشيد

zahedy2006
29-07-2009, 13:42
فرض كنيم A و B دو زيرمجموعه‏ي متناهي و مجزا از اعداد حقيقي باشند به طوري كه به ازاي هر [ برای مشاهده لینک ، لطفا با نام کاربری خود وارد شوید یا ثبت نام کنید ] داشته باشيم

[ برای مشاهده لینک ، لطفا با نام کاربری خود وارد شوید یا ثبت نام کنید ] يا [ برای مشاهده لینک ، لطفا با نام کاربری خود وارد شوید یا ثبت نام کنید ]

ثابت كنيد تعداد اعضاي A دو برابر تعداد اعضاي B است.سلام

من استدلال رياضي ام زياد قوي نيست.

چون دو مجموعه مجزا هستند پس هيچ عضو مشتركي ندارند

اعضاي اجتماع بايد به صورت دسته هايي از 3 عدد ترتيبي باشند
مثلا بايد 3 4 5 و 8 9 10 باشند

از اين دسته اعداد كه به صورت n , n+1 , n+2 ووو m , m+1 , m+2 هستند براي برقراري شرط بايد عضو اول در هر دسته در B و 2 عضو ديگر در A باشند

عضو n ، دو تا كمتر از n+2 است و در B است
دو عضو ديگر هم كه پياپي شرط اول يعني n+1 عضو A را برقرار مي كنند

پس در كل A دو برابر B عضو دارد

CppBuilder2006
29-07-2009, 13:49
فرض كنيم A و B دو زيرمجموعه‏ي متناهي و مجزا از اعداد حقيقي باشند به طوري كه به ازاي هر [ برای مشاهده لینک ، لطفا با نام کاربری خود وارد شوید یا ثبت نام کنید ] داشته باشيم

[ برای مشاهده لینک ، لطفا با نام کاربری خود وارد شوید یا ثبت نام کنید ] يا [ برای مشاهده لینک ، لطفا با نام کاربری خود وارد شوید یا ثبت نام کنید ]

ثابت كنيد تعداد اعضاي A دو برابر تعداد اعضاي B است.






تعریف می­کنیم:
[ برای مشاهده لینک ، لطفا با نام کاربری خود وارد شوید یا ثبت نام کنید ]



[ برای مشاهده لینک ، لطفا با نام کاربری خود وارد شوید یا ثبت نام کنید ]


با توجه به [ برای مشاهده لینک ، لطفا با نام کاربری خود وارد شوید یا ثبت نام کنید ] روشن است که [ برای مشاهده لینک ، لطفا با نام کاربری خود وارد شوید یا ثبت نام کنید ] یک به یک است. و چون [ برای مشاهده لینک ، لطفا با نام کاربری خود وارد شوید یا ثبت نام کنید ] متناهی است پوشا هم هست. یعنی [ برای مشاهده لینک ، لطفا با نام کاربری خود وارد شوید یا ثبت نام کنید ] دوسویی است. بنابراین [ برای مشاهده لینک ، لطفا با نام کاربری خود وارد شوید یا ثبت نام کنید ] یک جایگشت است. از ویژگی­های جایگشت­ها می­دانیم که [ برای مشاهده لینک ، لطفا با نام کاربری خود وارد شوید یا ثبت نام کنید ] هست که
[ برای مشاهده لینک ، لطفا با نام کاربری خود وارد شوید یا ثبت نام کنید ]
منظور از [ برای مشاهده لینک ، لطفا با نام کاربری خود وارد شوید یا ثبت نام کنید ]، [ برای مشاهده لینک ، لطفا با نام کاربری خود وارد شوید یا ثبت نام کنید ] بار ترکیب [ برای مشاهده لینک ، لطفا با نام کاربری خود وارد شوید یا ثبت نام کنید ] با خودش است و [ برای مشاهده لینک ، لطفا با نام کاربری خود وارد شوید یا ثبت نام کنید ] نگاشت همانی است ([ برای مشاهده لینک ، لطفا با نام کاربری خود وارد شوید یا ثبت نام کنید ]).


حتما با تابع مشخصه آشنا هستید:


[ برای مشاهده لینک ، لطفا با نام کاربری خود وارد شوید یا ثبت نام کنید ]



برای هر [ برای مشاهده لینک ، لطفا با نام کاربری خود وارد شوید یا ثبت نام کنید ] داریم:

[ برای مشاهده لینک ، لطفا با نام کاربری خود وارد شوید یا ثبت نام کنید ]



و
[ برای مشاهده لینک ، لطفا با نام کاربری خود وارد شوید یا ثبت نام کنید ]









و با ادامۀ این کار:


[ برای مشاهده لینک ، لطفا با نام کاربری خود وارد شوید یا ثبت نام کنید ]




پس برای هر [ برای مشاهده لینک ، لطفا با نام کاربری خود وارد شوید یا ثبت نام کنید ]،


[ برای مشاهده لینک ، لطفا با نام کاربری خود وارد شوید یا ثبت نام کنید ]


و در نتیجه


[ برای مشاهده لینک ، لطفا با نام کاربری خود وارد شوید یا ثبت نام کنید ]




[ برای مشاهده لینک ، لطفا با نام کاربری خود وارد شوید یا ثبت نام کنید ]


اما چون [ برای مشاهده لینک ، لطفا با نام کاربری خود وارد شوید یا ثبت نام کنید ] جایگشت است،


[ برای مشاهده لینک ، لطفا با نام کاربری خود وارد شوید یا ثبت نام کنید ]


[ برای مشاهده لینک ، لطفا با نام کاربری خود وارد شوید یا ثبت نام کنید ]



پس
[ برای مشاهده لینک ، لطفا با نام کاربری خود وارد شوید یا ثبت نام کنید ]



[ برای مشاهده لینک ، لطفا با نام کاربری خود وارد شوید یا ثبت نام کنید ]
[ برای مشاهده لینک ، لطفا با نام کاربری خود وارد شوید یا ثبت نام کنید ]

که همان چیزی است که می­خواستیم ثابت کنیم

CppBuilder2006
29-07-2009, 14:44
اعضاي اجتماع بايد به صورت دسته هايي از 3 عدد ترتيبي باشند

اگه اینو ثابت کنیم و ثابت کنیم دسته های سه تایی عضو مشترک ندارن فکر می کنم استدلال درسته!

chessmathter
29-07-2009, 19:54
با اسقرار حل میکنیم بر روی اعضای B اثبات میکنیم
اعضای مجموعه به طور صعودی مرتب اند
فرض کنیدB یک عضو داشته باشه [ برای مشاهده لینک ، لطفا با نام کاربری خود وارد شوید یا ثبت نام کنید ] %7D
[ برای مشاهده لینک ، لطفا با نام کاربری خود وارد شوید یا ثبت نام کنید ] که در B نیست پس [ برای مشاهده لینک ، لطفا با نام کاربری خود وارد شوید یا ثبت نام کنید ] در A هست
از طرفی [ برای مشاهده لینک ، لطفا با نام کاربری خود وارد شوید یا ثبت نام کنید ] در B نیست پس باید [ برای مشاهده لینک ، لطفا با نام کاربری خود وارد شوید یا ثبت نام کنید ] در A باشد
حال اگه A شامل عضو دیگری مانند x باشد چون b یک عضو دارد پس x+1 وx+2 ,...در A وA شامل بینهات عضو میشود و متناهی نیست که تناقض است پس [ برای مشاهده لینک ، لطفا با نام کاربری خود وارد شوید یا ثبت نام کنید ] %5Cright%5C%7D
حال اگر [ برای مشاهده لینک ، لطفا با نام کاربری خود وارد شوید یا ثبت نام کنید ] ht%5C%7D که بترتیب صعودی چیده شده
طبق قبل [ برای مشاهده لینک ، لطفا با نام کاربری خود وارد شوید یا ثبت نام کنید ]و [ برای مشاهده لینک ، لطفا با نام کاربری خود وارد شوید یا ثبت نام کنید ] در A هست حالا [ برای مشاهده لینک ، لطفا با نام کاربری خود وارد شوید یا ثبت نام کنید ] ,[ برای مشاهده لینک ، لطفا با نام کاربری خود وارد شوید یا ثبت نام کنید ] مساوی [ برای مشاهده لینک ، لطفا با نام کاربری خود وارد شوید یا ثبت نام کنید ] نیست چون بعد [ برای مشاهده لینک ، لطفا با نام کاربری خود وارد شوید یا ثبت نام کنید ]
و [ برای مشاهده لینک ، لطفا با نام کاربری خود وارد شوید یا ثبت نام کنید ] در A بوده و دو مجموعه مجزا نمیشن پس [ برای مشاهده لینک ، لطفا با نام کاربری خود وارد شوید یا ثبت نام کنید ] در A هست از طرفی باز نمیتونه

[ برای مشاهده لینک ، لطفا با نام کاربری خود وارد شوید یا ثبت نام کنید ]
و دو مجموعه مجزا نمیشن پس [ برای مشاهده لینک ، لطفا با نام کاربری خود وارد شوید یا ثبت نام کنید ] درA است و چون دو مجموعه مجزا و B دو عضو دارد A نمیتواند عضو دیگری داشته باشد وگرنه نامتناهی میشود پس [ برای مشاهده لینک ، لطفا با نام کاربری خود وارد شوید یا ثبت نام کنید ] &space;&plus;&space;1,b_2&space;&plus;&space;2%7D&space;%5Cright%5C%7D
حال فرض کنید برای [ برای مشاهده لینک ، لطفا با نام کاربری خود وارد شوید یا ثبت نام کنید ] 7D&space;%5Cright%5C%7D
داریم [ برای مشاهده لینک ، لطفا با نام کاربری خود وارد شوید یا ثبت نام کنید ] &space;&plus;&space;1,b_2&space;&plus;&space;2,...,b_m&space;&plus;&space;1,b_m&space;&plus;&space;2%7D&space;%5Cright% 5C%7D
برای [ برای مشاهده لینک ، لطفا با نام کاربری خود وارد شوید یا ثبت نام کنید ] b_%7Bm&space;&plus;&space;1%7D&space;%7D&space;%5Cright%5C%7D
تا [ برای مشاهده لینک ، لطفا با نام کاربری خود وارد شوید یا ثبت نام کنید ] اثبات شده حال به طریق مشابه
[ برای مشاهده لینک ، لطفا با نام کاربری خود وارد شوید یا ثبت نام کنید ] 1%7D&space;,i&space;%5Cin&space;%5Cleft%5C%7B&space;%7B1,2,...,m%7D&space;%5Crig ht%5C%7D
در غیر این صورت دو مجموعه متمایز نمیشوند همچنین
[ برای مشاهده لینک ، لطفا با نام کاربری خود وارد شوید یا ثبت نام کنید ] 1%7D&space;,i&space;%5Cin&space;%5Cleft%5C%7B&space;%7B1,2,...,m%7D&space;%5Crig ht%5C%7D
چون بازهم در غیر این صورت دو مجموعه متمایز نمیشوند
پس [ برای مشاهده لینک ، لطفا با نام کاربری خود وارد شوید یا ثبت نام کنید ] در A هستند و A به دلایل مشابه شامل عضو دیگری نیست پس
[ برای مشاهده لینک ، لطفا با نام کاربری خود وارد شوید یا ثبت نام کنید ] &space;&plus;&space;1,b_2&space;&plus;&space;2,...,b_m&space;&plus;&space;1,b_m&space;&plus;&space;2,b_%7Bm&space;&plus;&space;1%7D&space;&plus;&space;1 ,b_%7Bm&space;&plus;&space;1%7D&space;&plus;&space;2%7D&space;%5Cright%5C%7D
که این نشان میدهد A دوبرابر B عضو دارد همچنین ما رابطه اعضای دو مجموعه را نشان دادیم

CppBuilder2006
30-07-2009, 08:01
برای [ برای مشاهده لینک ، لطفا با نام کاربری خود وارد شوید یا ثبت نام کنید ] b_%7Bm&space;&plus;&space;1%7D&space;%7D&space;%5Cright%5C%7D
تا [ برای مشاهده لینک ، لطفا با نام کاربری خود وارد شوید یا ثبت نام کنید ] اثبات شده حال به طریق مشابه

تا [ برای مشاهده لینک ، لطفا با نام کاربری خود وارد شوید یا ثبت نام کنید ] چی اثبات شده ؟!:blink:

mofidy1
30-07-2009, 10:23
با سلام

دو عبارت زیر را برای هر عدد حقیقی مثبت a و b و c ثابت کنید:


[ برای مشاهده لینک ، لطفا با نام کاربری خود وارد شوید یا ثبت نام کنید ]{align}&space;4(a^3&plus;b^3)&\geq&space;(a&plus;b)^3&space;\nonumber\\&space;9(a^3&plus;b^3&plus;c^3)&space;&\geq&space;(a&plus;b&plus;c)^3&space;\nonumber\end{align}سی و دومین المپیاد ریاضی انگلستان 1996

(از کلمه ی المپیاد نترسید. مساله ی ساده اما زیبایی است!!!)

موفق باشید.

1 مرداد 1388

با سلام

از chessmathter که در پست 28 ([ برای مشاهده لینک ، لطفا با نام کاربری خود وارد شوید یا ثبت نام کنید ]) هر دو قسمت را به خوبی و در سطح خواسته شده (اول و دوم دبیرستان) حل کردند، تشکر می کنم.

هم چنین از saber57 که قسمت الف را در پست 26 ([ برای مشاهده لینک ، لطفا با نام کاربری خود وارد شوید یا ثبت نام کنید ])به درستی و به روش مشابه chessmathter حل کردند، متشکرم .(البته راه حل ایشان کمی ویرایش شد.)

راه حل CppBuilder2006 در پست 25 ([ برای مشاهده لینک ، لطفا با نام کاربری خود وارد شوید یا ثبت نام کنید ]) و پست 27 ([ برای مشاهده لینک ، لطفا با نام کاربری خود وارد شوید یا ثبت نام کنید ])بر اساس حساب دیفرانسیل است. اگر مشتق تابعی نسبت به یک متغیرش نامنفی باشد،این تابع نسبت به همان متغیر صعودی است. با تشکر از ایشان. (در پست 25 با تعریف ایشان خواهیم داشت 3=(f(0 که ظاهراً یک اشتباه محاسباتی در راه حل ایشان اتفاق افتاده است. در پست 27 نیز ظاهراً در ابتدای حل مساله فرض شده است که c>=b>=a)

آموزش حل مساله:

روشی را که chessmathter برای حل قسمت ب به کار بردند، به روش استفاده از مساله ی کمکی یا معاون معروف است. مساله ی کمکی مساله ای است که آن را برای خودش حل نمی کنیم، بلکه امید داریم با توجه به حل آن بتوانیم مساله ی اصلی را حل کنیم. در این جا قسمت الف، مساله ی کمکی برای قسمت ب بود.

موفق باشید.

8 مرداد 1388

mofidy1
30-07-2009, 10:38
با سلام

با در دست داشتن ارتفاع خارج شده از یک رأس مثلث و زاویه ی مربوط به این رأس و محیط مثلث، آن را رسم کنید.

موفق باشید.

8 مرداد 1388

saber57
31-07-2009, 03:52
با سلام

با در دست داشتن ارتفاع خارج شده از یک رأس مثلث و زاویه ی مربوط به این رأس و محیط مثلث، آن را رسم کنید.

موفق باشید.

8 مرداد 1388

مثلث ABC با زوایای A ,B ,C و اضلاع مقابل متناظر با هر زاویه را به ترتیب با a,b,c در نظر میگیریم . فرض کنیم ارتفاع وارد شده بر ضلع BC و خارج شده از راس A را مثلا AH با حرف d نشان دهیم و همچنین زاویه A و محیط مثلث یا همان مجموع اضلاع جزء معلومات مساله اند . بیاییم معلومات را مرور کنیم :



[ برای مشاهده لینک ، لطفا با نام کاربری خود وارد شوید یا ثبت نام کنید ]
[ برای مشاهده لینک ، لطفا با نام کاربری خود وارد شوید یا ثبت نام کنید ]
[ برای مشاهده لینک ، لطفا با نام کاربری خود وارد شوید یا ثبت نام کنید ]


ارتفاع AH با اندازه d مثلث ABC را به دو مثلث قائم الزاویه ABH و ACH تقسیم نموده است . در این دو مثلث :


[ برای مشاهده لینک ، لطفا با نام کاربری خود وارد شوید یا ثبت نام کنید ](B)=c.sin(B)\rightarr ow&space;sin(B)=\frac{d}{c}
[ برای مشاهده لینک ، لطفا با نام کاربری خود وارد شوید یا ثبت نام کنید ](C)=b.sin(C)\rightarr ow&space;sin(C)=\frac{d}{b}

رابطه عمومی مثلثها (بین اضلاع و زوایا :

[ برای مشاهده لینک ، لطفا با نام کاربری خود وارد شوید یا ثبت نام کنید ]{a}{sin(A)}=\frac{b}{sin(B)}=\frac{ c}{sin(C)}


با جایگذاری سینوسهای زوایای B و C در فرمول فوق :


[ برای مشاهده لینک ، لطفا با نام کاربری خود وارد شوید یا ثبت نام کنید ]{a}{sin(A)}=\frac{b}{\left&space;(&space;\frac{ d}{c}&space;\right&space;)}=\frac{c}{\left&space;(&space;\frac{d}{b}&space;\righ t&space;)}\rightarrow&space;\frac{a}{sin(A)}=\frac{bc}{d}\righ tarrow&space;bc=\left&space;(&space;\frac{d}{sin(A)}&space;\right&space;)a


در مثلث ABC :

[ برای مشاهده لینک ، لطفا با نام کاربری خود وارد شوید یا ثبت نام کنید ]^2=b^2&plus;c^2-2bc.cos(A)=\left&space;(&space;b&plus;c&space;\right&space;)^{2}-2bc-2bc.cos(A)=\left&space;(&space;b&plus;c&space;\right&space;)^{2}-2bc\left&space;(&space;1&plus;cos(A)&space;\right&space;)=\left&space;(&space;b&plus;c&space;\right&space;)^ {2}-4bc.cos^{2}\left&space;(&space;\frac{A}{2}&space;\right&space;)





بیاییم روابط را مرتب کنیم :


[ برای مشاهده لینک ، لطفا با نام کاربری خود وارد شوید یا ثبت نام کنید ]^2=\left&space;(&space;b&plus;c&space;\right&space;)^{2}-4bc.cos^{2}\left&space;(&space;\frac{A}{2}&space;\right&space;)
[ برای مشاهده لینک ، لطفا با نام کاربری خود وارد شوید یا ثبت نام کنید ](&space;\frac{d}{sin(A)}&space;\right&space;)a
[ برای مشاهده لینک ، لطفا با نام کاربری خود وارد شوید یا ثبت نام کنید ]


حالا بجای مجموع و حاصلضرب b و c در رابطه اولی جایگذاری میکنیم :


[ برای مشاهده لینک ، لطفا با نام کاربری خود وارد شوید یا ثبت نام کنید ]^2=\left&space;(&space;p-a&space;\right&space;)^{2}-\frac{4ad.cos^{2}\left&space;(&space;\frac{A}{2}&space;\right&space;)}{2si n\left&space;(&space;\frac{A}{2}&space;\right&space;).cos\left&space;(&space;\frac{A}{ 2}&space;\right&space;)}=p^2&plus;a^2-2ap-2ad.cotan\left&space;(&space;\frac{A}{2}&space;\right&space;)


با ساده سازی و مرتب سازی معادله فوق a براحتی بدست می آید :


[ برای مشاهده لینک ، لطفا با نام کاربری خود وارد شوید یا ثبت نام کنید ]{p^2}{2\left&space;(&space;p&plus;d.cotan\left&space;(&space;\ frac{A}{2}&space;\right&space;)&space;\right&space;)}


با بدست آمدن a سایر پارامترها براحتی بدست می آیند جون طبق روابط فوق مجموع b+c و حاصلضرب bc را داریم پس میتوان اضلاع b و c را از دو معادله دو مجهول بدست آورد و همینطور تا آخر

8 مرداد 88

CppBuilder2006
31-07-2009, 07:20
فکر کنم وقتی میگن رسم کنید منظورشون اینه که فقط میشه از خط کش بدون درجه بندی و چند پرگار که دهانشون با اندازه های داده شده باز شده استفاده کرد!

saber57
31-07-2009, 09:36
فکر کنم وقتی میگن رسم کنید منظورشون اینه که فقط میشه از خط کش بدون درجه بندی و چند پرگار که دهانشون با اندازه های داده شده باز شده استفاده کرد!
وقتی پارامترها را بدست بیاری رسمش که کاری نداره:31:
اما اگه بخواهیم رسمش کنیم محیط هم نیاز نیست . با یک خط کش، نقاله و پرگار رسم میشه . فقط کافیه ارتفاع و زاویه رسم را داشته باشیم .
طریقه رسم بدون محاسبه پارامترها (البته منظور سوال پیدا کردن سایر پارامترهای یک مثلث با استفاده از محیط زاویه راس و ارتفاع گذرنده از راس معلوم و ضلع مقابل هست ) :

یک خط افقی رسم میکنیم و ضلعی عمود بر آن به اندازه ارتفاع AH رسم میکنیم . . در سمت راست ارتفاع AH دهانه پرگار را به گونه ای باز میکنیم که از نقاط A و H بگذرد و یک دایره رسم میکنیم تا خط افقی عمود بر ارتفاع را در نقطه مثلا C قطع کند . نقطه C ,A و مرکز دایره حتما در یک راستا خواهند بود چون زاویه H برابر 90 درجه روبرو به کمان متناظر با قطر دایره به ضلع AC هست !! . ضلع AC که رسم شد ، مرکز نقاله را روی راس A و مماس بر ضلع AC قرار داده و به اندازه زاویه معلوم A ضلعی رسم میکنیم و هر جا این ضلع خط افقی را (که یک طرفش نقطه C مشخص شده ) را قطع کند ، نقطه B مشخص خواهد شد و مثلث رسم میشود .البته این روش رسم ، مثلث یکتایی نخواهد داد بلکه مثلثها خواهیم داشت و ضمنا ممکن است شرایط a+b+c=p را فراهم نکند . بهترین کار محاسبه مستقیم پارامترهاست .

9/5/88

chessmathter
31-07-2009, 18:47
با سلام

با در دست داشتن ارتفاع خارج شده از یک رأس مثلث و زاویه ی مربوط به این رأس و محیط مثلث، آن را رسم کنید.

موفق باشید.

8 مرداد 1388
[ برای مشاهده لینک ، لطفا با نام کاربری خود وارد شوید یا ثبت نام کنید ]

َABC همان مثلثی که میخوام رسم کنیم
AH, محیط مثلث و زاویه A معلومه
دقت کنین DB=BAوAC=CE پس DE همون محیط مثلث ABC است
اگر ADE رسم شود ABC رسم میشود چون B وC به ترتیب محل برخورد عمود منصف های AD و AE است
پس تنها باید ADE رسم شه از این مثلث زاویه یه راس A و قاعده رو به رو DE و ارتفاع AH داریم
برای رسم مثلث قاعده DE و کمان زاویه A رسم میکنیم بعد خطی موازی DE به فاصله AH رسم میکنیم تا کمان رو در نقطه قطع کنه هر کدام از این نقطه ها میتونین A بگیرین
[ برای مشاهده لینک ، لطفا با نام کاربری خود وارد شوید یا ثبت نام کنید ]
طریقه رسم کمان A هم اینه که عمود منصف DE رسم بعد از E وD زاویه های A-90 درجه جدا کرده تا عمود منصف رو در O قطع کنه

chessmathter
31-07-2009, 19:04
مثلث ABC با زوایای A ,B ,C و اضلاع مقابل متناظر با هر زاویه را به ترتیب با a,b,c در نظر میگیریم . فرض کنیم ارتفاع وارد شده بر ضلع BC و خارج شده از راس A را مثلا AH با حرف d نشان دهیم و همچنین زاویه A و محیط مثلث یا همان مجموع اضلاع جزء معلومات مساله اند . بیاییم معلومات را مرور کنیم :

[ برای مشاهده لینک ، لطفا با نام کاربری خود وارد شوید یا ثبت نام کنید ]
[ برای مشاهده لینک ، لطفا با نام کاربری خود وارد شوید یا ثبت نام کنید ]
[ برای مشاهده لینک ، لطفا با نام کاربری خود وارد شوید یا ثبت نام کنید ]


ارتفاع AH با اندازه d مثلث ABC را به دو مثلث قائم الزاویه ABH و ACH تقسیم نموده است . در این دو مثلث :


[ برای مشاهده لینک ، لطفا با نام کاربری خود وارد شوید یا ثبت نام کنید ] 9%5Crightarrow&space;sin%28B%29=%5Cfrac%7Bd%7D%7Bc%7D
[ برای مشاهده لینک ، لطفا با نام کاربری خود وارد شوید یا ثبت نام کنید ] 9%5Crightarrow&space;sin%28C%29=%5Cfrac%7Bd%7D%7Bb%7D

رابطه عمومی مثلثها (بین اضلاع و زوایا :

[ برای مشاهده لینک ، لطفا با نام کاربری خود وارد شوید یا ثبت نام کنید ] Bb%7D%7Bsin%28B%29%7D=%5Cfrac%7Bc%7D%7Bsin%28C%29% 7D


با جایگذاری سینوسهای زوایای B و C در فرمول فوق :


[ برای مشاهده لینک ، لطفا با نام کاربری خود وارد شوید یا ثبت نام کنید ] Bb%7D%7B%5Cleft&space;%28&space;%5Cfrac%7Bd%7D%7Bc%7D&space;%5Cright &space;%29%7D=%5Cfrac%7Bc%7D%7B%5Cleft&space;%28&space;%5Cfrac%7Bd%7 D%7Bb%7D&space;%5Cright&space;%29%7D%5Crightarrow&space;%5Cfrac%7Ba% 7D%7Bsin%28A%29%7D=%5Cfrac%7Bbc%7D%7Bd%7D%5Crighta rrow&space;bc=%5Cleft&space;%28&space;%5Cfrac%7Bd%7D%7Bsin%28A%29%7D &space;%5Cright&space;%29a


در مثلث ABC :

[ برای مشاهده لینک ، لطفا با نام کاربری خود وارد شوید یا ثبت نام کنید ] %7D-2bc-2bc.cos%28A%29=%5Cleft&space;%28&space;b&plus;c&space;%5Cright&space;%29%5E%7B2 %7D-2bc%5Cleft&space;%28&space;1&plus;cos%28A%29&space;%5Cright&space;%29=%5Cleft&space;% 28&space;b&plus;c&space;%5Cright&space;%29%5E%7B2%7D-4bc.cos%5E%7B2%7D%5Cleft&space;%28&space;%5Cfrac%7BA%7D%7B2%7D &space;%5Cright&space;%29





بیاییم روابط را مرتب کنیم :


[ برای مشاهده لینک ، لطفا با نام کاربری خود وارد شوید یا ثبت نام کنید ] 2%7D-4bc.cos%5E%7B2%7D%5Cleft&space;%28&space;%5Cfrac%7BA%7D%7B2%7D &space;%5Cright&space;%29
[ برای مشاهده لینک ، لطفا با نام کاربری خود وارد شوید یا ثبت نام کنید ] 29%7D&space;%5Cright&space;%29a
[ برای مشاهده لینک ، لطفا با نام کاربری خود وارد شوید یا ثبت نام کنید ]


حالا بجای مجموع و حاصلضرب b و c در رابطه اولی جایگذاری میکنیم :


[ برای مشاهده لینک ، لطفا با نام کاربری خود وارد شوید یا ثبت نام کنید ] %7D%7B2%7D&space;%5Cright&space;%29%7D%7B2sin%5Cleft&space;%28&space;%5Cfr ac%7BA%7D%7B2%7D&space;%5Cright&space;%29.cos%5Cleft&space;%28&space;%5Cfr ac%7BA%7D%7B2%7D&space;%5Cright&space;%29%7D=p%5E2&plus;a%5E2-2ap-2ad.cotan%5Cleft&space;%28&space;%5Cfrac%7BA%7D%7B2%7D&space;%5Crigh t&space;%29


با ساده سازی و مرتب سازی معادله فوق a براحتی بدست می آید :


[ برای مشاهده لینک ، لطفا با نام کاربری خود وارد شوید یا ثبت نام کنید ] cotan%5Cleft&space;%28&space;%5Cfrac%7BA%7D%7B2%7D&space;%5Cright&space;%2 9&space;%5Cright&space;%29%7D


با بدست آمدن a سایر پارامترها براحتی بدست می آیند جون طبق روابط فوق مجموع b+c و حاصلضرب bc را داریم پس میتوان اضلاع b و c را از دو معادله دو مجهول بدست آورد و همینطور تا آخر

8 مرداد 88


این راه حل درسته احتمالا هم تو محاسبه b وc اون آخر که معادله درجه 2 میشه ریشه مضاعف داره یعنی حتما:31:


فکر کنم وقتی میگن رسم کنید منظورشون اینه که فقط میشه از خط کش بدون درجه بندی و چند پرگار که دهانشون با اندازه های داده شده باز شده استفاده کرد!
در رسم شما خط کشی مدرج و نقله و پرگار میتونین استفاده کنین مگه اینکه ذکر کنن مثلا
یه زاویه 60 درجه رو با پرگار و خط کش به 3 قسمت کن اگه تونستی؟!:31:


تا [ برای مشاهده لینک ، لطفا با نام کاربری خود وارد شوید یا ثبت نام کنید ] چی اثبات شده ؟!:blink:
تا اونجا طبق فرض استقرا درست در نظر گرفتیم

وقتی پارامترها را بدست بیاری رسمش که کاری نداره:31:
اما اگه بخواهیم رسمش کنیم محیط هم نیاز نیست . با یک خط کش، نقاله و پرگار رسم میشه . فقط کافیه ارتفاع و زاویه رسم را داشته باشیم .
طریقه رسم بدون محاسبه پارامترها (البته منظور سوال پیدا کردن سایر پارامترهای یک مثلث با استفاده از محیط زاویه راس و ارتفاع گذرنده از راس معلوم و ضلع مقابل هست ) :

یک خط افقی رسم میکنیم و ضلعی عمود بر آن به اندازه ارتفاع AH رسم میکنیم . . در سمت راست ارتفاع AH دهانه پرگار را به گونه ای باز میکنیم که از نقاط A و H بگذرد و یک دایره رسم میکنیم تا خط افقی عمود بر ارتفاع را در نقطه مثلا C قطع کند . نقطه C ,A و مرکز دایره حتما در یک راستا خواهند بود چون زاویه H برابر 90 درجه روبرو به کمان متناظر با قطر دایره به ضلع AC هست !! . ضلع AC که رسم شد ، مرکز نقاله را روی راس A و مماس بر ضلع AC قرار داده و به اندازه زاویه معلوم A ضلعی رسم میکنیم و هر جا این ضلع خط افقی را (که یک طرفش نقطه C مشخص شده ) را قطع کند ، نقطه B مشخص خواهد شد و مثلث رسم میشود .البته این روش رسم ، مثلث یکتایی نخواهد داد بلکه مثلثها خواهیم داشت و ضمنا ممکن است شرایط a+b+c=p را فراهم نکند . بهترین کار محاسبه مستقیم پارامترهاست .

9/5/88
همونطور که گفتی این 2000000000000 مثلث میشه ساخت:31: تو رسم حتما جواب یکتاست مگه ذکر کنن تعداد جواب ها مثلا چند تا میشه

mir@
01-08-2009, 03:45
حل مسئله شنبه اول


.
دو نفر در فاصله 20 کیلومتری هم قرار دارند و با سرعت 10 کیلومتر در ساعت به طرف هم حرکت می کنند.

هواپیمایی با سرعت 20 کیلومتر در ساعت، هم زمان با شروع حرکت دو نفر، از نفر اول شروع کرده و به سمت نفر دوم پرواز می کند. پس از رسیدن به نفر دوم بر می گردد و به سمت نفر اول می آید.

این هواپیما آن قدر بین دونفر می رود و می آید تا دو نفر به هم برسند.

این هواپیما مجموعا چند کیلومتر پرواز کرده است؟


با تشکر از chessmathter که در پست شماره 31 راه حل هوشمندانه را ارائه کردند.

سپاس بی کران (!) از CppBuilder2006 که وقت گذاشتند و راه حل دقیق را در پست شماره 32 بیان فرمودند. بنده علاقمند بودم چنین راه حلی را نیز ببینم.

mir@
01-08-2009, 03:51
.
معادله زیر را حل بفرمایید:


[ برای مشاهده لینک ، لطفا با نام کاربری خود وارد شوید یا ثبت نام کنید ] ft&space;%7C&space;x&plus;1&space;%5Cright&space;%7C&space;=&space;x&space;&plus;&space;2

zahedy2006
01-08-2009, 05:00
معادله زیر را حل بفرمایید:تعيين علامت به بازه هاي
x<-1

-x+1-x-x-1=x+2 ---------> x=-0.5
در دامنه مورد بحث يعني كوچكتر از -1 نيست

x>=1

x-1+x+x+1=x+2 --------> x=1

0<x<1

-x+1+x+x+1=x+2 -----> x=x

0>x>-1

-x+1-x+x+1=x+2 -------> x=0

جواب هاي زير را مي دهد

x=0
x=1
[0,1]j


ساعت حل : 5:57 صبح!!!

CppBuilder2006
01-08-2009, 08:29
در رسم شما خط کشی مدرج و نقله و پرگار میتونین استفاده کنین مگه اینکه ذکر کنن مثلا
یه زاویه 60 درجه رو با پرگار و خط کش به 3 قسمت کن اگه تونستی؟!:31:

راه حل شما رو که ظاهرا میشه با پرگار و خط کش غیر مدرج، انجام داد.:31:
(البته با این فرض که اندازه ها و زاویه های معلوم با پارخطها و زاویه هایی از قبل کنار صورت مساله رسم شده باشه)

مشکل خط کش و نقاله اینه که همۀ اندازه ها روش درجه بندی نشده ن. مراسم باز کردن دهانۀ پرگار به اندازه رادیکال 2 یادتون هست!؟





تا [ برای مشاهده لینک ، لطفا با نام کاربری خود وارد شوید یا ثبت نام کنید ] چی اثبات شده ؟!:blink:


تا اونجا طبق فرض استقرا درست در نظر گرفتیم



فرض استقرا که در مورد [ برای مشاهده لینک ، لطفا با نام کاربری خود وارد شوید یا ثبت نام کنید ] 7D&space;%5Cright%5C%7D هست. می گه برای این B ،
[ برای مشاهده لینک ، لطفا با نام کاربری خود وارد شوید یا ثبت نام کنید ] &space;&plus;&space;1,b_2&space;&plus;&space;2,...,b_m&space;&plus;&space;1,b_m&space;&plus;&space;2%7D&space;%5Cright% 5C%7D
اما هیچچی در مورد [ برای مشاهده لینک ، لطفا با نام کاربری خود وارد شوید یا ثبت نام کنید ] b_%7Bm&space;&plus;&space;1%7D&space;%7D&space;%5Cright%5C%7D نمیگه!:19:

حالا استفاده از فرض استقرا خیلی هم ساده به نظر نمی رسه! باید [ برای مشاهده لینک ، لطفا با نام کاربری خود وارد شوید یا ثبت نام کنید ] b_%7Bm&space;&plus;&space;1%7D&space;%7D&space;%5Cright%5C%7D
به دوتا مجموعه شکست، ثابت کرد در شرایط مسأله صدق میکنه اووووووووو:13:!

yugioh
03-08-2009, 11:34
دفعه قبل جواب ندادید. همون تکرار میشه. اگه این بار هم کسی جواب نداد جدید سوال می دم.
درون یک کیسه 25 گوی یک شکل هست، شامل 6 قرمز 9 سبز 10 سفید. دو نفر بازی زیر رو انجام می دهند،
اولی یک گوی بیرون میاره. اگر سبز باشه برنده است اگر قرمز یا سفید باشه بازنده.
دومی یک گوی رو بیرون میاره، اگر قرمز باشه برنده است. اگر سبز باشه بازنده است. اگر سفید باشه اون گوی رو خارج می کنه و از گویهای باقی مانده یکی دیگه رو خارج می کنه. ( اگر 10 بار هم پست سر هم گوی سفید بیرون میاد.
اگر هر دو تا برنده یا بازنده شوند بازی برابر حساب شده و مجددا بازی می کنند، تا نهایتا یک نفر برندده بشه.
اگر شما جایگزین این دو نفر بودید به جای کدام یکی بازی می کردید؟

حالا یه سوال دیگه از این گوی ها، 7 تا از گویها روخارج می کنیم،اگر بدونیم توی انتخابیها، سفید وجود داره با چه احتمالی حداقل 1 قرمز و 2 سبز داره؟
راه حل davy jones برای بخش دوم درست وبرای بخش اول درسته ولی حوصله اش حتی از من هم بیشتره:
احتمال برد یکی: 36 درصده.
و دومی هم :
6/25+ (9/25)(6/24)+(9/25)(8/24)(6/23)+....(9/25)(8/24).... (1/16) * (9/15(
هر کی حوصله داره بدون ماشین حساب وکد زدن حساب کنه کدوم بیشتره.:21:
و در مورد سری توانی هم چون احتمال برد هر کس در یک دور اگر از دیگری بیشتر باشه پس در هر حال با احتمال بیشتری می بره. نیازی به سری توانی نیست. البته اشتباه هم نیست.

yugioh
03-08-2009, 14:30
این دفعه یه سوال از گراف: (بابت سوال دفعه قبل از همه به خصوص cpp builder , davy jones که زمان گذاشتند عذر می خوام و اشتباهم رو در طرح سوال مسخره قبلی قبول می کنم، به هر حال سوال مسخره ای بود:27:)
یک گراف n-بخشی است اگر بتوان آنرا به n بخش تقسیم کرد به شکلی که تمامی اعضای هر بخش به رئوس بخشهای دیگر یال داشته باشد. یعنی نسبت به بخش خود بدون یال باشد.
برای توضیح بیشتر سرچ کنید: r-partite graphs.
نوع خاص این گرافها دو بخشی اند.
ثابت کنید یک گراف دوبخشی است اگر و تنها اگر دور فرد نداشته باشد.
توضیح بدم مثلا یک گراف بدون یال هم دوبخشی است یه بخش با n راس و دیگری بدون راس.
یا مثلا k2 هم دوبخشی با هر راس در یک بخش.
این سوال رو درست حسابی پرسیدم انصافا اگر جواب بدید شاید رفتیم با هم به سمت سوالات سختتر.
توضیح: شرح اولیه من در مورد این گرافها گمراه کننده بود اینکه بگیم نسبت به بقیه گراف کامل نه یک k5 مثلا یعنی می تونه یال باشه یا نباشه.
در واقع منظورم این بود که نسبت به بخش خودش نباید یال داشته باشه، ولی نسبت به بقیه مثل گراف عادیه یال مل تونه باشه یا نه. انصافا توضیح اولیه ام خیلی گمراه کننده بوده. ببخشید. هر چند دوستان خودشون واردند.
در ضمن بگم اگر وتنها اگره یعنی باید بگید یک گراف بدون دور فرد دوبخشیه.

CppBuilder2006
03-08-2009, 15:46
نه بابا این چه حرفیه!



یک گراف n-بخشی است اگر بتوان آنرا به n بخش تقسیم کرد به شکلی که تمامی اعضای هر بخش به تمامی رئوس بخشهای دیگر یال داشته باشد. یعنی نسبت به بخش خود بدون یال باشد ولی به بخشهای دیگر ملنند گراف کامل باشد.
برای توضیح بیشتر سرچ کنید: r-partite graphs.
نوع خاص این گرافها دو بخشی اند.
ثابت کنید یک گراف دوبخشی است اگر و تنها اگر دور فرد نداشته باشد.
توضیح بدم مثلا یک گراف بدون یال هم دوبخشی است یه بخش با n راس و دیگری بدون راس.
یا مثلا k2 هم دوبخشی با هر راس در یک بخش.
این سوال رو درست حسابی پرسیدم انصافا اگر جواب بدید شاید رفتیم با هم به سمت سوالات سختتر.
خب فرض کنید A رئوس یک بخش و B رئوس یه بخش دیگه باشه. اگر A حداکثر یک عضو داشته باشه حکم برقراره چون هر دور درست دو یال داره.
فرض کنید حکم وقتی A حداکثر k عضو داره، درست باشه. نشون می­دیم وقتی A ، k+1 عضو داره هم درسسه.

پس فرض کنیدA|=k+1|. اگه دوری از همۀ رئوس A عبور نکنه، رأس­های آزاد و یال­های متصل به اونا رو حذف می­کنیم. طبق فرض استقرا دور زوجه. فرض کنید یه دور از همۀ رئوس A بگذره، یکی از یال­های این دور رو حذف می کنیم. یه مسیر باقی می­مونه که در هر کدوم از دو سرش یه یال داریم که به رأسی متصل نیست. این دو یال رو به هم وصل میکنیم و یکی میکنیم تا مسیر، تبدیل به دور بشه. این دور جدید دو یال کم تر از دور قبلی داره و طبق فرض استقرا زوجه. پس دور الیه هم زوجه.
پس هر دور زوجه و حکم برایA|=k+1 | ثابت میشه.
چه اثبات پیچ در پیچی شد[ برای مشاهده لینک ، لطفا با نام کاربری خود وارد شوید یا ثبت نام کنید ]!

yugioh
03-08-2009, 23:01
دوستان گویا پستم خیلی مفهوم نبوده ویرایش شد. از همتون عذر می خوام.

eh_mn
05-08-2009, 05:23
فرض كنيم A و B دو زيرمجموعه‏ي متناهي و مجزا از اعداد حقيقي باشند به طوري كه به ازاي هر [ برای مشاهده لینک ، لطفا با نام کاربری خود وارد شوید یا ثبت نام کنید ] داشته باشيم

[ برای مشاهده لینک ، لطفا با نام کاربری خود وارد شوید یا ثبت نام کنید ] يا [ برای مشاهده لینک ، لطفا با نام کاربری خود وارد شوید یا ثبت نام کنید ]

ثابت كنيد تعداد اعضاي A دو برابر تعداد اعضاي B است.


ـــــــــــــــــ
7 / 05 / 88


راه‏حل CppBuilder2006 در پست


برای مشاهده محتوا ، لطفا وارد شوید یا ثبت نام کنید

يك اشكال كوچيك داشت اينكه چرا نگاشت f، تابع هست؟ به عبارت ديگر چرا f خوشتعريف است؟

اما در پست


برای مشاهده محتوا ، لطفا وارد شوید یا ثبت نام کنید

نه تنها اين اشكال برطرف شد بلكه راه‏حل سرراست‏تري نسبت به قبلي ارائه شد.

حدس دوستمون zahedy2006 كاملا درسته.




اعضاي اجتماع بايد به صورت دسته هايي از 3 عدد ترتيبي باشند
مثلا بايد 3 4 5 و 8 9 10 باشند

از اين دسته اعداد كه به صورت n , n+1 , n+2 ووو m , m+1 , m+2 هستند براي

برقراري شرط بايد عضو اول در هر دسته در B و 2 عضو ديگر در A باشند



براي اثبات اين حدس chessmathter از استقرا استفاده كردن كه احتياج به كمي اصلاحات داره!

فكر مي‏كنم بايد اجزاي استقرا (پايه، فرض و حكم) رو دقيق‏تر بنويسيم.

پايه‏ي استقرا:

اگر [ برای مشاهده لینک ، لطفا با نام کاربری خود وارد شوید یا ثبت نام کنید ]{b_1\} آنگاه [ برای مشاهده لینک ، لطفا با نام کاربری خود وارد شوید یا ثبت نام کنید ]{b_1&plus;1,b_1&plus;2\}.
.
اين قسمت را chessmathter بدرستي اثبات كردند.


با اسقرار حل میکنیم بر روی اعضای B اثبات میکنیم
اعضای مجموعه به طور صعودی مرتب اند
فرض کنیدB یک عضو داشته باشه [ برای مشاهده لینک ، لطفا با نام کاربری خود وارد شوید یا ثبت نام کنید ] %7D
[ برای مشاهده لینک ، لطفا با نام کاربری خود وارد شوید یا ثبت نام کنید ] که در B نیست پس [ برای مشاهده لینک ، لطفا با نام کاربری خود وارد شوید یا ثبت نام کنید ] در A هست
از طرفی [ برای مشاهده لینک ، لطفا با نام کاربری خود وارد شوید یا ثبت نام کنید ] در B نیست پس باید [ برای مشاهده لینک ، لطفا با نام کاربری خود وارد شوید یا ثبت نام کنید ] در A باشد
حال اگه A شامل عضو دیگری مانند x باشد چون b یک عضو دارد پس x+1 وx+2 ,...در A وA شامل بینهات عضو میشود و متناهی نیست که تناقض است پس [ برای مشاهده لینک ، لطفا با نام کاربری خود وارد شوید یا ثبت نام کنید ] %5Cright%5C%7D



فرض استقرا:

اگر B داراي n عضو به صورت [ برای مشاهده لینک ، لطفا با نام کاربری خود وارد شوید یا ثبت نام کنید ]{&space;b_1,b_2,\cdots,b_n&space;\} باشد آنگاه A داراي 2n عضو به صورت [ برای مشاهده لینک ، لطفا با نام کاربری خود وارد شوید یا ثبت نام کنید ]{&space;b_1&plus;1,&space;b_1&plus;2,&space;b_2&plus;1,&space;b_2&plus;2&space;,&space;\cdots,&space; b_n&plus;1,b_n&plus;2&space;\}
خواهد بود.

ناگفته نمونه كه ما هم فرض مي‏كنيم اعضاي مجموعه‏ي B بطور صعودي مرتب شدن.

براي اثبات حكم استقرا، فرض كنيم [ برای مشاهده لینک ، لطفا با نام کاربری خود وارد شوید یا ثبت نام کنید ]{d_1,d_2,\cdots,d_{n&plus;1}&space;\}. مشابه آنچه chessmathter براي اثبات پايه‏ي استقرا
نوشته است مي‏توان نشان داد كه [ برای مشاهده لینک ، لطفا با نام کاربری خود وارد شوید یا ثبت نام کنید ]
براي اينكه بتوانيم از فرض استقرا استفاده كنيم همونطور كه
CppBuilder2006 تذكر دادن بايد يك مجموعه‏ي n عضوي داشته باشيم. قرار مي‏دهيم [ برای مشاهده لینک ، لطفا با نام کاربری خود وارد شوید یا ثبت نام کنید ]'=B\setminus&space;\{d_1\} و [ برای مشاهده لینک ، لطفا با نام کاربری خود وارد شوید یا ثبت نام کنید ]'=A\setminus&space;\{d_1&plus;1,d_1&plus;2\}. مجموعه‏ي [ برای مشاهده لینک ، لطفا با نام کاربری خود وارد شوید یا ثبت نام کنید ]'، n
عضو دارد. كافيست اثبات كنيم كه مجموعه‏هاي [ برای مشاهده لینک ، لطفا با نام کاربری خود وارد شوید یا ثبت نام کنید ]'و [ برای مشاهده لینک ، لطفا با نام کاربری خود وارد شوید یا ثبت نام کنید ]' در فرض استقرا صدق مي‏كنند.

بدين منظور فرض كنيم [ برای مشاهده لینک ، لطفا با نام کاربری خود وارد شوید یا ثبت نام کنید ]'\cup&space;B'. چون [ برای مشاهده لینک ، لطفا با نام کاربری خود وارد شوید یا ثبت نام کنید ]'\cup&space;B'&space;\subseteq&space;A\cup&space;B پس [ برای مشاهده لینک ، لطفا با نام کاربری خود وارد شوید یا ثبت نام کنید ] يا [ برای مشاهده لینک ، لطفا با نام کاربری خود وارد شوید یا ثبت نام کنید ]

به راحتي ميشه نشون داد كه
اگر [ برای مشاهده لینک ، لطفا با نام کاربری خود وارد شوید یا ثبت نام کنید ] آنگاه [ برای مشاهده لینک ، لطفا با نام کاربری خود وارد شوید یا ثبت نام کنید ]'و اگه [ برای مشاهده لینک ، لطفا با نام کاربری خود وارد شوید یا ثبت نام کنید ] آنگاه [ برای مشاهده لینک ، لطفا با نام کاربری خود وارد شوید یا ثبت نام کنید ]' و اين اثبات مي‏كنه كه [ برای مشاهده لینک ، لطفا با نام کاربری خود وارد شوید یا ثبت نام کنید ]'و [ برای مشاهده لینک ، لطفا با نام کاربری خود وارد شوید یا ثبت نام کنید ]' در فرض استقرا صدق مي‏كنن و تمام.


ـــــــــــــــــ
14/ 05 / 88

eh_mn
05-08-2009, 06:53
يك nتايي از اعداد طبيعي مانند [ برای مشاهده لینک ، لطفا با نام کاربری خود وارد شوید یا ثبت نام کنید ](a_1,a_2,\dots,a_n) را «خوب» گوييم هرگاه [ برای مشاهده لینک ، لطفا با نام کاربری خود وارد شوید یا ثبت نام کنید ] و مجموع هيچ تعدادي از [ برای مشاهده لینک ، لطفا با نام کاربری خود وارد شوید یا ثبت نام کنید ]ها برابر n نشود. تعداد همه‏ي nتايي‏هاي خوب را بيابيد.

(المپياد سال ----- كشور ------)


ـــــــــــــــــ
14 / 05 / 88

sherlockholmz
05-08-2009, 11:53
با سلام
باعنايت به استقبال گسترده دوستان:46:، مسئله زير براي يك هفته ديگر تمديد خواهد شد:
دريك كيسه 9 مهره سبز و6 مهره سفيد و در كيسه ديگر 7 مهره سبز و 7 مهره سفيد داريم.يكي از كيسه ها را به تصادف انتخاب كرده از داخل آن 5 مهره به تصادف بيرون آورده و بدون نگاه كردن در كيسه دوم مي ريزيم.حال اينبار از كيسه ديگر 5 مهره انتخاب كرده و باز بدون نگاه كردن در كيسه اولي مي ريزيم.اين عمل را 3 بار انجام مي دهيم.
حالا از هر كيسه يك مهره بيرون مي آوريم، احتمال غير همرنگ بودن آنها چيست؟
موفق باشيد.

CppBuilder2006
05-08-2009, 13:49
CppBuilder2006 در پست

مساله رو خيلي جالب حل كردن. مشروط بر اينكه بتونن يك نكته رو توضيح بدن.
اينكه چرا نگاشت f، تابع هست؟ به عبارت ديگر چرا f خوشتعريف است؟




اصلاح اثبات: برای این که نیازی به اثبات خوشتعریفی [ برای مشاهده لینک ، لطفا با نام کاربری خود وارد شوید یا ثبت نام کنید ] نباشد، تعریف [ برای مشاهده لینک ، لطفا با نام کاربری خود وارد شوید یا ثبت نام کنید ] را به این صورت تغییر می­دهیم:

[

[ برای مشاهده لینک ، لطفا با نام کاربری خود وارد شوید یا ثبت نام کنید ]



طبق فرض مسأله برای هر [ برای مشاهده لینک ، لطفا با نام کاربری خود وارد شوید یا ثبت نام کنید ] یا [ برای مشاهده لینک ، لطفا با نام کاربری خود وارد شوید یا ثبت نام کنید ] یا [ برای مشاهده لینک ، لطفا با نام کاربری خود وارد شوید یا ثبت نام کنید ] پس [ برای مشاهده لینک ، لطفا با نام کاربری خود وارد شوید یا ثبت نام کنید ] روی کل [ برای مشاهده لینک ، لطفا با نام کاربری خود وارد شوید یا ثبت نام کنید ] تعریف شده. خوشتعریفی روشن است. برای اثبات یک به یک بودن فرض کنید [ برای مشاهده لینک ، لطفا با نام کاربری خود وارد شوید یا ثبت نام کنید ]،



الف) اگر [ برای مشاهده لینک ، لطفا با نام کاربری خود وارد شوید یا ثبت نام کنید ]، آن وقت

[ برای مشاهده لینک ، لطفا با نام کاربری خود وارد شوید یا ثبت نام کنید ]

ب) اگر حالت (الف) اتفاق نیفتد و [ برای مشاهده لینک ، لطفا با نام کاربری خود وارد شوید یا ثبت نام کنید ]،


[ برای مشاهده لینک ، لطفا با نام کاربری خود وارد شوید یا ثبت نام کنید ]








ج) اگر هیچ یک از دو حالت قبل اتفاق نیفتد، مثلا این طور می­شود:
[ برای مشاهده لینک ، لطفا با نام کاربری خود وارد شوید یا ثبت نام کنید ]









در این صورت[ برای مشاهده لینک ، لطفا با نام کاربری خود وارد شوید یا ثبت نام کنید ] و [ برای مشاهده لینک ، لطفا با نام کاربری خود وارد شوید یا ثبت نام کنید ] و [ برای مشاهده لینک ، لطفا با نام کاربری خود وارد شوید یا ثبت نام کنید ] که امکان ندارد چون [ برای مشاهده لینک ، لطفا با نام کاربری خود وارد شوید یا ثبت نام کنید ]





حالا داریم:
[ برای مشاهده لینک ، لطفا با نام کاربری خود وارد شوید یا ثبت نام کنید ]
[ برای مشاهده لینک ، لطفا با نام کاربری خود وارد شوید یا ثبت نام کنید ]


در نتیجه:
[ برای مشاهده لینک ، لطفا با نام کاربری خود وارد شوید یا ثبت نام کنید ]

[ برای مشاهده لینک ، لطفا با نام کاربری خود وارد شوید یا ثبت نام کنید ]
[ برای مشاهده لینک ، لطفا با نام کاربری خود وارد شوید یا ثبت نام کنید ]
[ برای مشاهده لینک ، لطفا با نام کاربری خود وارد شوید یا ثبت نام کنید ]

حالا اگر [ برای مشاهده لینک ، لطفا با نام کاربری خود وارد شوید یا ثبت نام کنید ]، [ برای مشاهده لینک ، لطفا با نام کاربری خود وارد شوید یا ثبت نام کنید ] پس [ برای مشاهده لینک ، لطفا با نام کاربری خود وارد شوید یا ثبت نام کنید ] و در نتیجه [ برای مشاهده لینک ، لطفا با نام کاربری خود وارد شوید یا ثبت نام کنید ] پس


[ برای مشاهده لینک ، لطفا با نام کاربری خود وارد شوید یا ثبت نام کنید ]


و در نتیجه:
[ برای مشاهده لینک ، لطفا با نام کاربری خود وارد شوید یا ثبت نام کنید ]


(رفتم خوش تعریفیو ثابت کنم کلا اثبات عوض شد!)

eh_mn
05-08-2009, 17:32
اين «يا» خيلي مهمه!!

شما يك جايي گفتين طبق فرض داريم يا x+1 در A يا x-2 در B.


وقتي مي‏نويسيم گزاره‏ي P يا Q درست است يعني ، P درست است يا Q درست است يا هر دو درستن؛ اما
وقتي مي‏نويسيم يا گزاره‏ي P درست است يا Q درست است، يعني حداكثر يكي از P يا Q درستن.

در صورت سوال آمده است: x+1 در A يا x-2 در B.

بنابراين براي يك x در اجتماع ممكن است x+1 در A باشد و همزمان x-2 در B باشد در اين صورت تابع f براي اين عضو تعريف نشده است.

گمان مي‏كنم خوشتعريفي f روي اجتماع A و B همچنان واضح نيست!! (يعني ممكن است f روي تمام اجتماع A و B تعريف نشده باشد)

CppBuilder2006
05-08-2009, 17:40
سلام
پست آخر من بعد از پست شما ویرایش شده!
من معمولا وقتی میگم «یا p یا q» منظورم اینه که هر دو هم ممکنه اتفاق بیفته!
فکر میکنم اثباتی که پست قبلی نوشته م دیگه اشکالی نداشته باشه. یعنی امیدوارم!
ممنون برا ی تذکر !

خورشید زمستان
05-08-2009, 18:57
يك nتايي از اعداد طبيعي مانند [ برای مشاهده لینک ، لطفا با نام کاربری خود وارد شوید یا ثبت نام کنید ](a_1,a_2,\dots,a_n) را «خوب» گوييم هرگاه [ برای مشاهده لینک ، لطفا با نام کاربری خود وارد شوید یا ثبت نام کنید ] و مجموع هيچ تعدادي از [ برای مشاهده لینک ، لطفا با نام کاربری خود وارد شوید یا ثبت نام کنید ]ها برابر n نشود. تعداد همه‏ي nتايي‏هاي خوب را بيابيد.

(المپياد سال ----- كشور ------)


ـــــــــــــــــ
14 / 05 / 88
این اعضا متمایزند یا نه؟

CppBuilder2006
05-08-2009, 21:23
يك nتايي از اعداد طبيعي مانند [ برای مشاهده لینک ، لطفا با نام کاربری خود وارد شوید یا ثبت نام کنید ](a_1,a_2,\dots,a_n) را «خوب» گوييم هرگاه [ برای مشاهده لینک ، لطفا با نام کاربری خود وارد شوید یا ثبت نام کنید ] و مجموع هيچ تعدادي از [ برای مشاهده لینک ، لطفا با نام کاربری خود وارد شوید یا ثبت نام کنید ]ها برابر n نشود. تعداد همه‏ي nتايي‏هاي خوب را بيابيد.

(المپياد سال ----- كشور ------)


ـــــــــــــــــ
14 / 05 / 88



تعداد جواب­های
[ برای مشاهده لینک ، لطفا با نام کاربری خود وارد شوید یا ثبت نام کنید ]



برابر است با
[ برای مشاهده لینک ، لطفا با نام کاربری خود وارد شوید یا ثبت نام کنید ]


تعریف: nتایی [ برای مشاهده لینک ، لطفا با نام کاربری خود وارد شوید یا ثبت نام کنید ] را عالی می­گوییم اگر


[ برای مشاهده لینک ، لطفا با نام کاربری خود وارد شوید یا ثبت نام کنید ]


و حداقل یک [ برای مشاهده لینک ، لطفا با نام کاربری خود وارد شوید یا ثبت نام کنید ] باشد که


[ برای مشاهده لینک ، لطفا با نام کاربری خود وارد شوید یا ثبت نام کنید ]

[ برای مشاهده لینک ، لطفا با نام کاربری خود وارد شوید یا ثبت نام کنید ]


اگر E تعداد مجموعه­های عالی باشد و G تعداد مجموعه­های خوب داریم:

[ برای مشاهده لینک ، لطفا با نام کاربری خود وارد شوید یا ثبت نام کنید ]


برای [ برای مشاهده لینک ، لطفا با نام کاربری خود وارد شوید یا ثبت نام کنید ]، هر جواب

[ برای مشاهده لینک ، لطفا با نام کاربری خود وارد شوید یا ثبت نام کنید ]

را به [ برای مشاهده لینک ، لطفا با نام کاربری خود وارد شوید یا ثبت نام کنید ] شکل به یک جواب از


[ برای مشاهده لینک ، لطفا با نام کاربری خود وارد شوید یا ثبت نام کنید ]

گسترش داد که در آن [ برای مشاهده لینک ، لطفا با نام کاربری خود وارد شوید یا ثبت نام کنید ] تعداد جواب­های
[ برای مشاهده لینک ، لطفا با نام کاربری خود وارد شوید یا ثبت نام کنید ]
است و [ برای مشاهده لینک ، لطفا با نام کاربری خود وارد شوید یا ثبت نام کنید ] تعداد حالات درج [ برای مشاهده لینک ، لطفا با نام کاربری خود وارد شوید یا ثبت نام کنید ] ها بین [ برای مشاهده لینک ، لطفا با نام کاربری خود وارد شوید یا ثبت نام کنید ]ها.


پس تعداد مجموعه­های عالی برابر است با

[ برای مشاهده لینک ، لطفا با نام کاربری خود وارد شوید یا ثبت نام کنید ]


و تعداد مجموعه­های خوب برابر است با

[ برای مشاهده لینک ، لطفا با نام کاربری خود وارد شوید یا ثبت نام کنید ]

(داخل مجموع به جای !n!/k اشتباهی یه چیز دیگه نوشته م)

اصلا نمی­دونم درست حل کرده م یا نه!

eh_mn
06-08-2009, 10:21
سلام
پست آخر من بعد از پست شما ویرایش شده!
من معمولا وقتی میگم «یا p یا q» منظورم اینه که هر دو هم ممکنه اتفاق بیفته!
فکر میکنم اثباتی که پست قبلی نوشته م دیگه اشکالی نداشته باشه. یعنی امیدوارم!
ممنون برا ی تذکر !


سلام


اشكال قبلي هنوز وارده.

شما مي‏گين وقتي از يا p يا q استفاده مي‏كنين منظورتون اينه كه هر دوي p و q ممكنه اتفاق بيفته. ولي اينطور استفاده نكردين!

اگه ممكنه به اين سوال جواب بدين: آيا ممكنه xاي در اجتماع A و B باشه بطوري كه x-2 در B و x+1 در A باشه؟ چرا؟


ــــــــــــــــــــــ

eh_mn
06-08-2009, 10:30
این اعضا متمایزند یا نه؟

كدوم اعضا مورد نظرتونه؟

اگه منظورتون ai-ها هستن، چون در صورت سوال بيان نشده كه ai-ها متمايز باشن پس ما هم اين فرض رو نداريم.

CppBuilder2006
06-08-2009, 14:19
اگه ممكنه به اين سوال جواب بدين: آيا ممكنه xاي در اجتماع A و B باشه بطوري كه x-2 در B و x+1 در A باشه؟ چرا؟



با اطلاعات اولیۀ ما ممکنه در هر دو باشه. ولی ممکنه بعدا ثابت کنیم که این طور نیست. اما اگر ثابت هم نکنیم،
[ برای مشاهده لینک ، لطفا با نام کاربری خود وارد شوید یا ثبت نام کنید ]
خوش تعریفه چون برای هر x در اجتماع A و B، یه مقدار منحصر به فرد برای f تعیین شده که در همین اجتماع هست. پس خوش تعریفی مشکلی نداره. یک به یک بودن رو هم ثابت کرده م که پوشا بودن رو نتیجه میده. بعد از همین f که ثابت شده یه جایگشته در اثبات کمک گرفته م. مشکلی هست؟!

eh_mn
06-08-2009, 16:11
با اطلاعات اولیۀ ما ممکنه در هر دو باشه. ولی ممکنه بعدا ثابت کنیم که این طور نیست. اما اگر ثابت هم نکنیم،
[ برای مشاهده لینک ، لطفا با نام کاربری خود وارد شوید یا ثبت نام کنید ]
خوش تعریفه چون برای هر x در اجتماع A و B، یه مقدار منحصر به فرد برای f تعیین شده که در همین اجتماع هست. پس خوش تعریفی مشکلی نداره. یک به یک بودن رو هم ثابت کرده م که پوشا بودن رو نتیجه میده. بعد از همین f که ثابت شده یه جایگشته در اثبات کمک گرفته م. مشکلی هست؟!

حق با شماست، اگه ثابت كنيم كه براي هر x در اجتماع A و B،‏ يا x+1 در A و يا x-2 در B، بعدش مي‏تونيم تابعي كه شما معرفي كردين رو روي اجتماع A و B تعريف كنيم. شما اين كار رو كجا انجام دادين؟ و گرنه مقدار f رو براي اون‏هايي كه

با اطلاعات اولیۀ ما ممکنه در هر دو باشه
بايد چي بگيريم؟

mofidy1
06-08-2009, 17:40
با سلام

با در دست داشتن ارتفاع خارج شده از یک رأس مثلث و زاویه ی مربوط به این رأس و محیط مثلث، آن را رسم کنید.

موفق باشید.

8 مرداد 1388

با سلام

روش درست حل این گونه مسائل را chessmathter در پست 69 ([ برای مشاهده لینک ، لطفا با نام کاربری خود وارد شوید یا ثبت نام کنید ]) ارائه کردند. از ایشان تشکر می کنم. البته روشsaber57 در پست 66 ([ برای مشاهده لینک ، لطفا با نام کاربری خود وارد شوید یا ثبت نام کنید ])غلط نیست اما همان طور که CppBuilder2006 توضیح دادند، هدف از حل این گونه مسائل استفاده ی صرف از ابزار هندسی اقلیدسی (خط کش و پرگار) است نه ابزار جبری. بد نیست بدانید که این محدودیت اجباری باعث تولد یکی از مدرن ترین شاخه های جبر به نام نظریه ی گالوا شد که یکی از بهترین ابزار برای حل معادلات در میدان های مختلف است.(ببخشید که بیش از حد تخصصی شد!!!)

آموزش حل مساله:

تبدیل یا تحویل مساله به مساله ای ساده تر، یکی از روش های خوب حل مساله است که مساله ی این هفته نیز یکی از مثال های آن بود.

موفق باشید.

15 مرداد 1388

mofidy1
06-08-2009, 18:23
با سلام

دایره ی ثابتی به شعاع 1 و مثلث متساوی الاضلاع محاط در این دایره را در نظر بگیرید. طول ضلع این مثلث را L فرض می کنیم. فرض کنید وتری از این دایره به طور تصادفی انتخاب شده باشد. احتمال این که طول وتر - که آن را M می گیریم - از طول ضلع مثلث محاط در دایره - که L فرض کردیم - بیشتر باشد، چه قدر است؟

موفق باشید.

15 مرداد 1388

< A L I >
06-08-2009, 18:31
با سلام

دایره ی ثابتی به شعاع 1 و مثلث متساوی الاضلاع محاط در این دایره را در نظر بگیرید. طول ضلع این مثلث را L فرض می کنیم. فرض کنید وتری از این دایره به طور تصادفی انتخاب شده باشد. احتمال این که طول وتر - که آن را M می گیریم - از طول ضلع مثلث محاط در دایره - که L فرض کردیم - بیشتر باشد، چه قدر است؟

موفق باشید.

15 مرداد 1388
جواب 1/3 مي باشد.

chessmathter
06-08-2009, 19:09
با سلام

دایره ی ثابتی به شعاع 1 و مثلث متساوی الاضلاع محاط در این دایره را در نظر بگیرید. طول ضلع این مثلث را L فرض می کنیم. فرض کنید وتری از این دایره به طور تصادفی انتخاب شده باشد. احتمال این که طول وتر - که آن را M می گیریم - از طول ضلع مثلث محاط در دایره - که L فرض کردیم - بیشتر باشد، چه قدر است؟

موفق باشید.

15 مرداد 1388
هر چه وتر به مرکز نزدیک تر باشه بزرگتراست از طرفی هر ضلع مثلث متساوی الاضلاع به ضلع [ برای مشاهده لینک ، لطفا با نام کاربری خود وارد شوید یا ثبت نام کنید ] محاط در دایره از مرکز [ برای مشاهده لینک ، لطفا با نام کاربری خود وارد شوید یا ثبت نام کنید ] فاصله داره پس یه دایره به این شعاع و به مرکز دایره اصلی میزنیم اگه فاصله ی وتر کمتر از این باشد بزرگتر از ضلع است در غیر این صورت کوچکتر (یعنی باید وسط وتر داخل دایره بیفتد) نسبت مساحت این دایره به کل دایره میشه احتمال خواسته شده که هست [ برای مشاهده لینک ، لطفا با نام کاربری خود وارد شوید یا ثبت نام کنید ]

mofidy1
06-08-2009, 19:19
جواب 1/3 مي باشد.

با سلام

دقیقاً توضیح دهید. چرا جواب، یک سوم است؟!

15 مرداد 1388

CppBuilder2006
06-08-2009, 19:44
با سلام

دایره ی ثابتی به شعاع 1 و مثلث متساوی الاضلاع محاط در این دایره را در نظر بگیرید. طول ضلع این مثلث را L فرض می کنیم. فرض کنید وتری از این دایره به طور تصادفی انتخاب شده باشد. احتمال این که طول وتر - که آن را M می گیریم - از طول ضلع مثلث محاط در دایره - که L فرض کردیم - بیشتر باشد، چه قدر است؟

موفق باشید.

15 مرداد 1388

میدونید، فکر میکنم این سوال از نظر تخصصی جواب ثابتی نداره! تا دوستان نظرشون چی باشه.

< A L I >
06-08-2009, 22:00
با سلام

دقیقاً توضیح دهید. چرا جواب، یک سوم است؟!

15 مرداد 1388
سلام
مثلث ABC را مثلث محاط در دايره در نظر مي گيريم.
در ابتدا و براي مفهوم بودن اثبات، وتر مورد نظر را AO در نظر مي گيريم(A راس مثلث و O هر نقطه ي دلخواه از محيط دايره)
اگر O روي كمان BC باشد AO بزرگتر از ضلع مثلث مي شود(زيرا كمان AO بزرگتر از كمان ما بين هر ضلع مي شود)
و مي دانيم كه كمان BC يك سوم كل محيط دايره است پس احتمال مورد نظر براي نقطه ي A برابر يك سوم مي شود.
مشخص است كه اين اثبات براي نقاط ديگر محيط دايره نيز قابل ارايه است پس جواب مساله نيز يك سوم مي شود.

chessmathter
06-08-2009, 22:55
سلام
مثلث ABC را مثلث محاط در دايره در نظر مي گيريم.
در ابتدا و براي مفهوم بودن اثبات، وتر مورد نظر را AO در نظر مي گيريم(A راس مثلث و O هر نقطه ي دلخواه از محيط دايره)
اگر O روي كمان BC باشد AO بزرگتر از ضلع مثلث مي شود(زيرا كمان AO بزرگتر از كمان ما بين هر ضلع مي شود)
و مي دانيم كه كمان BC يك سوم كل محيط دايره است پس احتمال مورد نظر براي نقطه ي A برابر يك سوم مي شود.
مشخص است كه اين اثبات براي نقاط ديگر محيط دايره نيز قابل ارايه است پس جواب مساله نيز يك سوم مي شود.
مشکل اینجاست ممکنه وتر اصلا شامل راس های مثلث نشه

< A L I >
06-08-2009, 23:19
سلام
مثلث ABC را مثلث محاط در دايره در نظر مي گيريم.
در ابتدا و براي مفهوم بودن اثبات، وتر مورد نظر را AO در نظر مي گيريم(A راس مثلث و O هر نقطه ي دلخواه از محيط دايره)
اگر O روي كمان BC باشد AO بزرگتر از ضلع مثلث مي شود(زيرا كمان AO بزرگتر از كمان ما بين هر ضلع مي شود)
و مي دانيم كه كمان BC يك سوم كل محيط دايره است پس احتمال مورد نظر براي نقطه ي A برابر يك سوم مي شود.
مشخص است كه اين اثبات براي نقاط ديگر محيط دايره نيز قابل ارايه است پس جواب مساله نيز يك سوم مي شود.


مشکل اینجاست ممکنه وتر اصلا شامل راس های مثلث نشه
سلام
براي نقاط ديگر كه راس مثلث ABC نمي باشند اين مراحل به اثبات اضافه مي شود:
نام نقطه ي دلخواه مورد نظر(راس مثلث ABC نباشد) را M در نظر مي گيريم. حال مثلث متساوي الاضلاعي را به گونه اي در دايره محاط مي كنيم كه يكي از راس هاي آن M شود(مثلث جديد=MNP ) واضح است كه MNP و ABC هم نهشت مي باشند. ادامه ي اثبات همانند قبل مي شود.

chessmathter
07-08-2009, 11:28
سلام
براي نقاط ديگر كه راس مثلث ABC نمي باشند اين مراحل به اثبات اضافه مي شود:
نام نقطه ي دلخواه مورد نظر(راس مثلث ABC نباشد) را M در نظر مي گيريم. حال مثلث متساوي الاضلاعي را به گونه اي در دايره محاط مي كنيم كه يكي از راس هاي آن M شود(مثلث جديد=MNP ) واضح است كه MNP و ABC هم نهشت مي باشند. ادامه ي اثبات همانند قبل مي شود.
این طوری که نمیشه حاجی باس مثلث و ثابت نگه داری بلاخره بینهایت وتر هست که راس مثلث نیستن اونا باید تو احتمال حساب شه

zahedy2006
07-08-2009, 12:28
این طوری که نمیشه حاجی باس مثلث و ثابت نگه داری بلاخره بینهایت وتر هست که راس مثلث نیستن اونا باید تو احتمال حساب شه
سلام
چرا بايد مثلث ثابت باشه؟؟
براي ما احتمال طول مهم هست

ابتدا وتر رسم شده را مي گيريم و از يك سر آن مثلث خودمان را رسم مي كنيم
طول ضلع مثلث تغيير نمي كند
طول وتر نيز تغيير نمي كند
پس در كليت مساله - احتمال ما - تاثيري ندارد.

البته چون خودم مي خواستم اين جواب رو بنويسم ولي ديدم جواب داده شده اومدم توضيح بدهم

saber57
07-08-2009, 12:55
سوال eh_mn سوال شانزدهمین المپیاد ریاضی(مرحله دوم بهار 77) بود


با سلام

دقیقاً توضیح دهید. چرا جواب، یک سوم است؟!

15 مرداد 1388


مشکل اینجاست ممکنه وتر اصلا شامل راس های مثلث نشه

اگه مثلث متساوی الاضلاع رو طوری بچرخونیم که راس A روی یک نقطه وتر قرار بگیره ، برای اینکه وتر فرضی بزرگتر از طول یک ضلع مثلث باشه ، نقطه دوم وتر قطعا روی کمان BC قرار خواهد گرفت و در واقع کمان BC مکان هندسی نقاطی از وتر است که طول وتر آن بزرگتر از طول یک ضلع مثلث متساوی الاضلاع و نقطه طرف دیگر آن وتر روی راس مقابل کمان BC قرار گرفته باشه . مجموع این نقاط کمان BC را تشکیل میدهند که یک سوم محیط کل دایره است . احتمالش هم میشه :

[ برای مشاهده لینک ، لطفا با نام کاربری خود وارد شوید یا ثبت نام کنید ](A)=\frac{\overset{\frown&space;}{BC}}{\overs et{\frown&space;}{ABC}}=\frac{1}{3}

chessmathter
07-08-2009, 14:56
سلام
چرا بايد مثلث ثابت باشه؟؟
براي ما احتمال طول مهم هست

ابتدا وتر رسم شده را مي گيريم و از يك سر آن مثلث خودمان را رسم مي كنيم
طول ضلع مثلث تغيير نمي كند
طول وتر نيز تغيير نمي كند
پس در كليت مساله - احتمال ما - تاثيري ندارد.

البته چون خودم مي خواستم اين جواب رو بنويسم ولي ديدم جواب داده شده اومدم توضيح بدهم


سوال eh_mn سوال شانزدهمین المپیاد ریاضی(مرحله دوم بهار 77) بود





اگه مثلث متساوی الاضلاع رو طوری بچرخونیم که راس A روی یک نقطه وتر قرار بگیره ، برای اینکه وتر فرضی بزرگتر از طول یک ضلع مثلث باشه ، نقطه دوم وتر قطعا روی کمان BC قرار خواهد گرفت و در واقع کمان BC مکان هندسی نقاطی از وتر است که طول وتر آن بزرگتر از طول یک ضلع مثلث متساوی الاضلاع و نقطه طرف دیگر آن وتر روی راس مقابل کمان BC قرار گرفته باشه . مجموع این نقاط کمان BC را تشکیل میدهند که یک سوم محیط کل دایره است . احتمالش هم میشه :

[ برای مشاهده لینک ، لطفا با نام کاربری خود وارد شوید یا ثبت نام کنید ] &space;%7D%7BBC%7D%7D%7B%5Coverset%7B%5Cfrown&space;%7D%7BABC% 7D%7D=%5Cfrac%7B1%7D%7B3%7D

ببینین وقتی میگه احتمال اینکه وتر بزرگتر از ضلع باشه یعنی همه وتر ها ولی در راه حل شما فقط وتر هایی محاسبه میشه که از یه راس اونم راس مشخص میگذره محاسبه میشه نه همه وتر ها
ببینین یه مثلث دادن دایره محیطی شم دادن وتر های هست که از راس مثلث نگذره بلاخره تو که نمیتونه هی مثلث و دوران بدی.... چه جوری بگم یه مثال باس پیدا کنم:31:

saber57
07-08-2009, 16:17
شما میتونید یک مثلث فرضی قرینه با مثلث اصلی در دایره رسم کنید . اضلاع این دو مثلث دو بدو موازیند .(مثل پرچم اسراییل) هر وتر موازی با دو ضلع متناظر موازی و بین آنها بزرگتر از ضلع مثلث اصلی هست. حالا مثلث محاط در دایره رو اونقدر میچرخونیم تا موازی با وتر باشه بالطبع مثلث قرینه موازی هم یک ضلع موازی با وتر خواهد داشت . در صورتی وتر تصادفی بین دو ضلع موازی باشه مقدارش بزرگتر از ضلع مثلث و خارج کوچکتر هست و چون مجموع کمانهای واقع بین دو ضلع موازی 120 درجه هست پس بعبارتی احتمال 3/1 خواهد بود

< A L I >
07-08-2009, 16:38
ببینین وقتی میگه احتمال اینکه وتر بزرگتر از ضلع باشه یعنی همه وتر ها ولی در راه حل شما فقط وتر هایی محاسبه میشه که از یه راس اونم راس مشخص میگذره محاسبه میشه نه همه وتر ها
ببینین یه مثلث دادن دایره محیطی شم دادن وتر های هست که از راس مثلث نگذره بلاخره تو که نمیتونه هی مثلث و دوران بدی.... چه جوری بگم یه مثال باس پیدا کنم:31:
سلام
مي تونيم به جاي اين كه مثلث( ABC ) را دوران بدهيم تا وتر بر يكي از راس هاي آن منطبق شود،مثلث اوليه(مثلث ABC) را نگه داريم و مثلث متسوي الاضلاع ديگري را(مثلث MNP) در دايره محاط كنيم به صورتي كه يكي از راس هاي آن ،راس وتر فرضي باشد.(قبول داريد كه در اين صورت تمامي وترها محسبه مي شود؟)
و ادمه ي كار.....=احتمال اين كه وتر از ضلع مثلث MNP بزرگتر باشد 1/3 مي شود.
قسمت بنفش رنگ را كه دوستان به اشكال مختلف ثابت كردند.
مي دانيم كه ABC و MNP هم نهشت هستند.(دو مثلث متساوي الاضلاع كه در يك دايره محاط شده اند)
پس اگر وتر فرضي از ضلع مثلث MNP بزرگتر باشد از ضلع مثلث ABC نيز بزرگتر مي شود.پس احتمال اين كه وتر از ضلع ABC بزرگتر شود نيز برابر 1/3 مي شود.


البته من اثبات ديگر دوستان را كه از روش دوران رفته اند، كاملاً قبول دارم.:31::31::31:

chessmathter
07-08-2009, 21:23
با سلام

دایره ی ثابتی به شعاع 1 و مثلث متساوی الاضلاع محاط در این دایره را در نظر بگیرید. طول ضلع این مثلث را L فرض می کنیم. فرض کنید وتری از این دایره به طور تصادفی انتخاب شده باشد. احتمال این که طول وتر - که آن را M می گیریم - از طول ضلع مثلث محاط در دایره - که L فرض کردیم - بیشتر باشد، چه قدر است؟

موفق باشید.

15 مرداد 1388
دوستان من فکر کردم و فهمیدم اصلا مثلث وجودش لازم نیست ضلع مثلث رادیکال 3 هست
سوال میگه احتمال اینکه یه وتر بیشتر از رادیکال 3 باشه چقدره
از اونجایی که راه حل من 1/4 بود و راه حل شما 1/3 و من به جواب خودم مطمعنم:31: و یه مسله 2 جواب نداره تحقیق کردم و جالبه جوابه 1/2 هم داریم:18::31:
اینم از پارادکس برتراند

مساله:وتر، پاره خطی است که نقطه های انتهایش، دو نقطه از دایره باشند.در دایره ای به شعاع1 ,احتمال این که طول وتری بیش از[ برای مشاهده لینک ، لطفا با نام کاربری خود وارد شوید یا ثبت نام کنید ] باشد، چقدر است؟

چنین مساله ي ساده ای می تواند بسیار شگفت انگیز باشد، به این علت که می توان چندین راه حل به ظاهر منطقی برای آن ارائه داد که هر یک به پاسخی متفاوت می انجامد.

راه حل اول:
وتری مانند AB را در نظر بگیرید که در نقطه ی M بر دایره ي به شعاع[ برای مشاهده لینک ، لطفا با نام کاربری خود وارد شوید یا ثبت نام کنید ] و مرکز دایره ي نخست ، مماس باشد.( شکل1 )

[ برای مشاهده لینک ، لطفا با نام کاربری خود وارد شوید یا ثبت نام کنید ]
از آن جا که AB بر این دایره مماس است ، بر شعاع MC عمود خواهد بود.پس بنا بر قضیه ی فیثاغورث داریم:

[ برای مشاهده لینک ، لطفا با نام کاربری خود وارد شوید یا ثبت نام کنید ]
به طریق مشابه داریم: [ برای مشاهده لینک ، لطفا با نام کاربری خود وارد شوید یا ثبت نام کنید ]پس طول وتر AB برابر [ برای مشاهده لینک ، لطفا با نام کاربری خود وارد شوید یا ثبت نام کنید ] است.
برای وتر دلخواه EF ،اگر پای عمودی که ازمرکز دایره بر این وتر اخراج می شود،درون دایره ی داخلي بیفتد، آن گاه :
طول AB < EF طول و در غیر این صورت :
طول AB[ برای مشاهده لینک ، لطفا با نام کاربری خود وارد شوید یا ثبت نام کنید ] EF طول . پس این نتیجه گیری به نظر منطقی می آید که احتمال این که طول وتری از[ برای مشاهده لینک ، لطفا با نام کاربری خود وارد شوید یا ثبت نام کنید ] بیش تر باشد برابر است با احتمال این که پای عمود آن درون دایره ی داخلی واقع شود.پس احتمال مورد نظربرابر است با:

[ برای مشاهده لینک ، لطفا با نام کاربری خود وارد شوید یا ثبت نام کنید ]

[ برای مشاهده لینک ، لطفا با نام کاربری خود وارد شوید یا ثبت نام کنید ]
راه حل دوم:
همه ی وترهایی را که از نقطه ی A واقع بر دایره می گذرند، در نظر بگیرید.نقطه هاي B و C را بر دايره طوري بگيريد كه [ برای مشاهده لینک ، لطفا با نام کاربری خود وارد شوید یا ثبت نام کنید ] .شکل 2 را خواهیم داشت:

[ برای مشاهده لینک ، لطفا با نام کاربری خود وارد شوید یا ثبت نام کنید ]
با توجه به شکل،از آن جا که ABD>،زاويه محاطي روبروي نيم دايره است،قائمه خواهد بود و چون کسینوسBAD>،برابر[ برای مشاهده لینک ، لطفا با نام کاربری خود وارد شوید یا ثبت نام کنید ] است،درنتیجه: [ برای مشاهده لینک ، لطفا با نام کاربری خود وارد شوید یا ثبت نام کنید ] >.به همین ترتیب[ برای مشاهده لینک ، لطفا با نام کاربری خود وارد شوید یا ثبت نام کنید ] > ،پس[ برای مشاهده لینک ، لطفا با نام کاربری خود وارد شوید یا ثبت نام کنید ] >ولذا طول كمان BDC برابر [ برای مشاهده لینک ، لطفا با نام کاربری خود وارد شوید یا ثبت نام کنید ] محیط دایره است.هر وتری که یک سرش A و سر دیگرش (نقطه اي غير از B,C )بر کمان BDC واقع باشد،از [ برای مشاهده لینک ، لطفا با نام کاربری خود وارد شوید یا ثبت نام کنید ]بزرگ تر است و هر وتری که از A بگذرد وسرديگرش بر كمان BDC نباشد از [ برای مشاهده لینک ، لطفا با نام کاربری خود وارد شوید یا ثبت نام کنید ]کوچک تر است.پس احتمال این که طول وتری بیش از[ برای مشاهده لینک ، لطفا با نام کاربری خود وارد شوید یا ثبت نام کنید ] باشد،همان احتمال واقع شدن سرديگر وتر در كمان BDC مي باشد و این یعنی احتمال مورد نظر برابراست با:

[ برای مشاهده لینک ، لطفا با نام کاربری خود وارد شوید یا ثبت نام کنید ]

[ برای مشاهده لینک ، لطفا با نام کاربری خود وارد شوید یا ثبت نام کنید ]
راه حل سوم:
همه ی وتر هایی را در نظر بگیرید که بر شعاعی از دایره ،چون CDعمود باشند،براي وتري به طول[ برای مشاهده لینک ، لطفا با نام کاربری خود وارد شوید یا ثبت نام کنید ] ، نقطه ی E در فاصله ی [ برای مشاهده لینک ، لطفا با نام کاربری خود وارد شوید یا ثبت نام کنید ] از C قرار می گیرد(شكل3).حال با توجه به قضیه ی فیثاغورث در مورد مثلث BEC داریم:

[ برای مشاهده لینک ، لطفا با نام کاربری خود وارد شوید یا ثبت نام کنید ]

[ برای مشاهده لینک ، لطفا با نام کاربری خود وارد شوید یا ثبت نام کنید ]
هر وتر عمود برCD ،اگر به C نزدیک تر باشد تا به D،از [ برای مشاهده لینک ، لطفا با نام کاربری خود وارد شوید یا ثبت نام کنید ] بزرگ تر و در غیر این صورت از [ برای مشاهده لینک ، لطفا با نام کاربری خود وارد شوید یا ثبت نام کنید ] کوچک تر است.پس منطقی است که نتیجه بگیریم: احتمال این که طول وتری بیش از[ برای مشاهده لینک ، لطفا با نام کاربری خود وارد شوید یا ثبت نام کنید ] باشد،برابر است با احتمال این که فاصله ی نقطه ی تقاطع وتر و شعاع عمود بر آن ، بين C وE واقع شود واین یعنی احتمال مورد نظر برابر است با :

[ برای مشاهده لینک ، لطفا با نام کاربری خود وارد شوید یا ثبت نام کنید ]

[ برای مشاهده لینک ، لطفا با نام کاربری خود وارد شوید یا ثبت نام کنید ]
ما در این جا به یک پارادوکس می رسیم که چون توسط ژوزف برتراند مطرح شده است،به پارادوکس برتراند مشهور است.

mir@
08-08-2009, 01:02
حل مسئله شنبه دوم


.
معادله زیر را حل بفرمایید:


[ برای مشاهده لینک ، لطفا با نام کاربری خود وارد شوید یا ثبت نام کنید ] ft&space;%7C&space;x&plus;1&space;%5Cright&space;%7C&space;=&space;x&space;&plus;&space;2



تعيين علامت به بازه هاي
x<-1

-x+1-x-x-1=x+2 ---------> x=-0.5
در دامنه مورد بحث يعني كوچكتر از -1 نيست

x>=1

x-1+x+x+1=x+2 --------> x=1

0<x<1

-x+1+x+x+1=x+2 -----> x=x

0>x>-1

-x+1-x+x+1=x+2 -------> x=0

جواب هاي زير را مي دهد

x=0
x=1
[0,1]j


ساعت حل : 5:57 صبح!!!


با تشکر از جناب zahedy2006 که این مسئله را به درستی حل کردند.

mir@
08-08-2009, 01:29
.
تمام سه تایی های صحیح [ برای مشاهده لینک ، لطفا با نام کاربری خود وارد شوید یا ثبت نام کنید ] را پیدا کنید که در رابطه زیر صدق کنند:


[ برای مشاهده لینک ، لطفا با نام کاربری خود وارد شوید یا ثبت نام کنید ]

ali_hp
09-08-2009, 09:55
الف)بیشترین و کمترین مقدار تابع f رابدست اورید:

[ برای مشاهده لینک ، لطفا با نام کاربری خود وارد شوید یا ثبت نام کنید ](x)=2|x-1|-|x-5|&plus;|3-x|

ب)قرار دهید:

[ برای مشاهده لینک ، لطفا با نام کاربری خود وارد شوید یا ثبت نام کنید ](x)=a_1|x-b_1|&plus;a_2|x-b_2|&plus;...&plus;a_n|x-b_n|

نشان دهید بیشترین و کمترین مقدار f یکی از مقادیر زیر می باشند:

[ برای مشاهده لینک ، لطفا با نام کاربری خود وارد شوید یا ثبت نام کنید ](b_1),f(b_2),...,f(b_n),\lim_{x\to\inft y}{f(x)},\lim_{x\to-\infty}{f(x)}
حل:
الف)حالت خاصی از ب) است.
بدون کم شدن از کلیت مساله فرض کنید:
[ برای مشاهده لینک ، لطفا با نام کاربری خود وارد شوید یا ثبت نام کنید ]{b_2}\le...\le{b_n}


دامنه f به n+1 بازه [ برای مشاهده لینک ، لطفا با نام کاربری خود وارد شوید یا ثبت نام کنید ](-\infty,b_1],[b_1,b_2],...,[b_n,\infty) تقسیم کنید،دقت کنید که f در هریک از این بازه ها تابعی خطی است(چرا؟)،و هر تابع خطی بیشترین و کمترین مقدارش را در یکی از نقاط انتهایی دامنه اش می گیرد،پس f بیشترین و کمترین مقدارش را در یکی از نقاط [ برای مشاهده لینک ، لطفا با نام کاربری خود وارد شوید یا ثبت نام کنید ] می گیرد!




اولی min میشه 2- و max میشه بینهایت.

دومی از این قضیه به دست میاد:
«ماکسیمم و مینیمم هر تابع در نقاط بحرانی حاصل میشوند»
با تشکر از شما،اگه ممکنه راه حلتونو کامل کنید،نقاط بحرانی در اینجا کدوم نقاط هستند؟در کدوم نقطه ها مشتق صفر می شود؟

ali_hp
09-08-2009, 10:18
الف)آیا تابع [ برای مشاهده لینک ، لطفا با نام کاربری خود وارد شوید یا ثبت نام کنید ](0,1)\rightarrow{(200,501)}که یک به یک و پوشا باشد وجود دارد؟

ب)آیا تابع [ برای مشاهده لینک ، لطفا با نام کاربری خود وارد شوید یا ثبت نام کنید ](0,1)\rightarrow\mathbb{R} که یک به یک و پوشا باشد وجود دارد؟

CppBuilder2006
09-08-2009, 14:15
ببینین وقتی میگه احتمال اینکه وتر بزرگتر از ضلع باشه یعنی همه وتر ها ولی در راه حل شما فقط وتر هایی محاسبه میشه که از یه راس اونم راس مشخص میگذره محاسبه میشه نه همه وتر ها
ببینین یه مثلث دادن دایره محیطی شم دادن وتر های هست که از راس مثلث نگذره بلاخره تو که نمیتونه هی مثلث و دوران بدی.... چه جوری بگم یه مثال باس پیدا کنم:31:

شما فکر کنید بینهایت مثلث متساوی الاضلاع یکسان در حالتهای مختلف درون دایره محاط شده ن احتمال این که طول یک وتر انتخاب از طول ضلع یکی از این مثلثا بیش تر باشه کدومه؟!

در هر حال من نظر قبلی خودمو که هیچ جواب ثابتی برای این مسأله وجود نداره تکرار میکنم.

yugioh
10-08-2009, 20:52
از همه به خصوص جناب cpp builder که لطف کردند و پاسخ دادند تشکر می کنم یه عذر خواهی هم می کنم به خاطر اینکه متاسفانه الان سرعت اینترنتم وضع مطلوبی نداره، بحث جواب رو بدلیل نیاز به عکس می زارم برای بعد، دوتا سوال:
ثابت کنید هر درخت حداقل به تعداد بزرگترین درجه در رئوسش برگ دارد.
دوتا سوال هم برای مسائل جانبی واضحی، می زارم.
احتمال اینکه در 35 بار انتخاب از بین اعداد 1 تا 100 با جایگذاری، 17، 100 بیاد؟ این احتمال برای 84 تا از 375 تا 100 اومدن چقدره؟
این دو احتمال رو بگید به طور متوسط هر چند بار یک بار رخ می ده؟
پیشاپیش ممنونم.

eh_mn
12-08-2009, 12:53
قبل از اين كه به جواب مسأله‏ي اين هفته بپردازيم بايد بگم كه در مورد سوال دو هفته قبل مشكلي در مورد خوشتعريف بودن تابع معرفي شده در راه حل CppBuilder2006 بوجود اومده بود كه با توضيحات (خصوصي(!)) ايشون من متوجه اشتباهم شدم و پست مربوط به جواب مسأله‏ي چهارشنبه‏ي دوم يعني

برای مشاهده محتوا ، لطفا وارد شوید یا ثبت نام کنید
رو ويرايش كردم. از ايشون به خاطر توضيحات كاملشون متشكرم. و اما مسأله‏ي هفته قبل:


يك nتايي از اعداد طبيعي مانند [ برای مشاهده لینک ، لطفا با نام کاربری خود وارد شوید یا ثبت نام کنید ](a_1,a_2,\dots,a_n) را «خوب» گوييم هرگاه [ برای مشاهده لینک ، لطفا با نام کاربری خود وارد شوید یا ثبت نام کنید ] و مجموع هيچ تعدادي از [ برای مشاهده لینک ، لطفا با نام کاربری خود وارد شوید یا ثبت نام کنید ]ها برابر n نشود. تعداد همه‏ي nتايي‏هاي خوب را بيابيد.

(المپياد سال ----- كشور ------)


ـــــــــــــــــ
14 / 05 / 88


من هم مانند CppBuilder2006 در پست

برای مشاهده محتوا ، لطفا وارد شوید یا ثبت نام کنید
مسأله رو بصورت تركيبياتي حل كرده بودم. ولي متوجه شدم كه در شمارش، تعداد جواب‏هايي كه چند بار شمرده مي‏شن خيلي زياده.
از طرفي اگه در فرمول بدست اومده به جاي n عدد 2 رو قرار بديم مي‏بينيم كه جواب منفي مي‏شه.

ولي راه حل از اين قراره.
ابتدا چند نكته رو در نظر مي‏گيريم
فرض كنيد [ برای مشاهده لینک ، لطفا با نام کاربری خود وارد شوید یا ثبت نام کنید ](a_1,a_2,\dots,a_n) يك nتايي خوب باشه. از اين به بعد اين n تايي رو بصورت صعودي مرتب شده در نظر مي‏گيريم. نشون مي‏ديم كه اگر aiها مساوي نباشن آنگاه a1=1.
چون [ برای مشاهده لینک ، لطفا با نام کاربری خود وارد شوید یا ثبت نام کنید ] پس


[ برای مشاهده لینک ، لطفا با نام کاربری خود وارد شوید یا ثبت نام کنید ](n-2)a_1&space;\leq&space;a_2&plus;\dots&plus;a_{n-1}&space;=&space;2n-a_1-a_n\leq&space;2n-2a_1&space;-1

بنابراين [ برای مشاهده لینک ، لطفا با نام کاربری خود وارد شوید یا ثبت نام کنید ]{1}{n}. پس a1=1.
از طرفي اگه a_i ها مساوي باشن بايد همشون 2 باشن. حالا اگه n زوج باشه n-تايي‏اي كه همه مولفه‏هاش 2 باشن نمي‏تونه يك n-تايي خوب باشه و اگه n فرد باشه چنين n-تايي‏اي حتما خوب هست. از اين به بعد فرض مي‏كنيم كه a_iها با هم مساوي نيستن.

حالا مي‏خوايم با استفاده از استقراء ثابت كنيم كه همه n-تايي‏هايي خوب به صورت [ برای مشاهده لینک ، لطفا با نام کاربری خود وارد شوید یا ثبت نام کنید ](1,1,\dots,n&plus;1) هستن.

پايه‏ي استقراء : به راحتي بررسي مي‏شود.
فرض استقراء : n-1n-1 تايي [ برای مشاهده لینک ، لطفا با نام کاربری خود وارد شوید یا ثبت نام کنید ](1,1,\dots,n) تنها n-1-تايي خوب به طور مرتب صعودي است.

مي‏خواهيم نشان دهيم كه هر n-تايي خوب به طور صعودي مرتب شده مانند [ برای مشاهده لینک ، لطفا با نام کاربری خود وارد شوید یا ثبت نام کنید ](1,a_2,\dots,a_n) به صورت [ برای مشاهده لینک ، لطفا با نام کاربری خود وارد شوید یا ثبت نام کنید ](1,1,\dots,n&plus;1) است. ادعا مي‏كنيم كه n-1-تايي [ برای مشاهده لینک ، لطفا با نام کاربری خود وارد شوید یا ثبت نام کنید ](b_1,b_2,\dots,b_{n-1})=(a_2,\dots,a_n-1) خوب است.

واضح است كه [ برای مشاهده لینک ، لطفا با نام کاربری خود وارد شوید یا ثبت نام کنید ]{n-1}=2(n-1). فرض كنيم B زيرمجموعه‏اي از [ برای مشاهده لینک ، لطفا با نام کاربری خود وارد شوید یا ثبت نام کنید ]{1,\dots,n-1\} باشد و


[ برای مشاهده لینک ، لطفا با نام کاربری خود وارد شوید یا ثبت نام کنید ]{i\in&space;B}=n-1

اگر n-1 در B باشد آنگاه

[ برای مشاهده لینک ، لطفا با نام کاربری خود وارد شوید یا ثبت نام کنید ]{&space;i&space;\in&space;B}b_i=\sum_{&space;i&space;\in&space;B&space;~&space;i&space;\neq&space;n-1}b_i&plus;a_n-1

پس


[ برای مشاهده لینک ، لطفا با نام کاربری خود وارد شوید یا ثبت نام کنید ]{&space;i&space;\in&space;B&space;~&space;i&space;\neq&space;n-1}a_i&plus;a_n=n

كه ممكن نيست.
و اگر n-1 در B نباشد آنگاه

[ برای مشاهده لینک ، لطفا با نام کاربری خود وارد شوید یا ثبت نام کنید ]{i\in&space;B}a_i&space;&plus;&space;1&space;=&space;\sum_{i\in&space;B}b_i&space; &plus;&space;1&space;=n
كه باز هم غيرممكن است. بنابراين [ برای مشاهده لینک ، لطفا با نام کاربری خود وارد شوید یا ثبت نام کنید ](a_2,\dots,a_n-1) يك n-1-تايي خوب است. و حكم بدين ترتيب اثبات مي‏شود.

در نهايت تعداد n-تايي‏هي خوب اين طور بدست مي‏آيد كه اگر n زوج باشد برابر n و اگر n فرد باشد برابر n+1.

sherlockholmz
12-08-2009, 15:37
با سلام
در يك كيسه دو مهره آبي، 6 مهره قرمز و 4 مهره سياه وجود دارد.
يك مهره از كيسه درآورده و بدون نگاه به آن كنار مي گذاريم، مهره دوم را برمي داريم، احتمال قرمز بودن آن چقدر است؟
اين را حل كنيد تا...

eh_mn
12-08-2009, 15:47
ثابت كنيد براي همه‏ي مقدارهاي مثبت x داريم

[ برای مشاهده لینک ، لطفا با نام کاربری خود وارد شوید یا ثبت نام کنید ]^{\sqrt[12]{x}}&plus;2^{\sqrt[4]{x}}\geq&space;2\times&space;2^{\sqrt[6]{x}}



ـــــــــــــــــــــــــ
21 / 05 / 88

kachool
13-08-2009, 10:43
با سلام
در يك كيسه دو مهره آبي، 6 مهره قرمز و 4 مهره سياه وجود دارد.
يك مهره از كيسه درآورده و بدون نگاه به آن كنار مي گذاريم، مهره دوم را برمي داريم، احتمال قرمز بودن آن چقدر است؟
اين را حل كنيد تا...
رنگ ها متناسب با انتخاب مهره ها هستند !


~ (4 / 12) * (6 / 11) + (6 / 12) * (5 / 11) + (2 / 12) * (6 / 11)
= (4 * 6) / (12 * 11) + (6 * 5) / (12 * 11) + (2 * 6) / (12 * 11)
= ((4 * 6) + (6 * 5) + (2 * 6)) / (12 * 11)
= (24 + 30+ 12) / (132)
= 66 / 132
= 1/2
= 0.5

amintnt
13-08-2009, 12:00
با سلام
در يك كيسه دو مهره آبي، 6 مهره قرمز و 4 مهره سياه وجود دارد.
يك مهره از كيسه درآورده و بدون نگاه به آن كنار مي گذاريم، مهره دوم را برمي داريم، احتمال قرمز بودن آن چقدر است؟
اين را حل كنيد تا...
چون از رنگ مهره اطلاعی نداریم میتونیم فرض کنیم مهره همچنان توی کیسه است... و احتمال قرمز بودن میشه 6/12 یعنی پنج دهم.

mir@
15-08-2009, 03:56
حل مسئله شنبه سوم


.
تمام سه تایی های صحیح [ برای مشاهده لینک ، لطفا با نام کاربری خود وارد شوید یا ثبت نام کنید ] را پیدا کنید که در رابطه زیر صدق کنند:


[ برای مشاهده لینک ، لطفا با نام کاربری خود وارد شوید یا ثبت نام کنید ]



معادله داده شده معادل است با:


[ برای مشاهده لینک ، لطفا با نام کاربری خود وارد شوید یا ثبت نام کنید ]


می دانیم gcd یعنی ب.م.م. (بزرگترین مخرج مشترک)

بنابراین


[ برای مشاهده لینک ، لطفا با نام کاربری خود وارد شوید یا ثبت نام کنید ] ases%7D&space;x=da&space;%5C%5C&space;y=db&space;%5Cend%7Bcases%7D&space;%5Cqqua d&space;gcd%28a,b%29=1%5C%5C&space;%5CRightarrow&space;%5Cbegin%7Bca ses%7D&space;y-x=ka&space;%5C%5C&space;x-z=kb&space;%5Cend%7Bcases%7D&space;%5C&&space;%5Cqquad&space;gcd%28a,b-a%29=gcd%28a,b%29=1%5C%5C&space;%5CRightarrow&space;%28b-a%29%7Ck%5CRightarrow&space;k=m%28b-a%29%5CRightarrow&space;d=ma%5C%5C&space;%5CRightarrow&space;%5Cbegi n%7Bcases%7D&space;x=ma%5E2&space;%5C%5C&space;y=mab&space;%5C%5Cz=m%28a%5 E2&plus;ab-b%5E2%29&space;%5Cend%7Bcases%7D

برای مقادیر a,b,m دلخواه

mir@
15-08-2009, 04:07
عبارت زیر را تجزیه کنید:


[ برای مشاهده لینک ، لطفا با نام کاربری خود وارد شوید یا ثبت نام کنید ]^3&plus;b^3&plus;c^3-3abc

راهنمایی: از معادله زیر کمک بگیرید:


[ برای مشاهده لینک ، لطفا با نام کاربری خود وارد شوید یا ثبت نام کنید ](X)=X^3-(a&plus;b&plus;c)X^2&plus;(ab&plus;bc&plus;ca)X-abc


سپس عبارت زیر را تجزیه کنید:


[ برای مشاهده لینک ، لطفا با نام کاربری خود وارد شوید یا ثبت نام کنید ](x-y)^3&plus;(y-z)^3&plus;(z-x)^3

ali_hp
16-08-2009, 09:40
الف)آیا تابع [ برای مشاهده لینک ، لطفا با نام کاربری خود وارد شوید یا ثبت نام کنید ](0,1)\rightarrow{(200,501)}که یک به یک و پوشا باشد وجود دارد؟

ب)آیا تابع [ برای مشاهده لینک ، لطفا با نام کاربری خود وارد شوید یا ثبت نام کنید ](0,1)\rightarrow\mathbb{R} که یک به یک و پوشا باشد وجود دارد؟
حل:
الف)محور اعداد حقیقی را در نظر بگیرید،مساله معادل این است که تناظری یک به یک بین نقاط بازه [ برای مشاهده لینک ، لطفا با نام کاربری خود وارد شوید یا ثبت نام کنید ](0,1)
و بازه [ برای مشاهده لینک ، لطفا با نام کاربری خود وارد شوید یا ثبت نام کنید ](200,501) وجود داشته باشد،حال این دو بازه را به صورتی در صفحه قرار دهید که در یک امتداد نباشند،نقطه 0 را به 200 و 1 را به 501 وصل کنید،و محل تقاطع این دو خط را P بنامید،حال برای هر نقطه دلخواه مثل q از بازه[ برای مشاهده لینک ، لطفا با نام کاربری خود وارد شوید یا ثبت نام کنید ](0,1)،محل تقاطع خط pq باپاره خط متناظر با بازه [ برای مشاهده لینک ، لطفا با نام کاربری خود وارد شوید یا ثبت نام کنید ](200,501) را به عنوان متناظر نقطه q در نظر بگیرید.
ب)پاره خط متناظر با بازه [ برای مشاهده لینک ، لطفا با نام کاربری خود وارد شوید یا ثبت نام کنید ](0,1) را به یک نیم دایره تبدیل کنید،حال نیم دایره را طوری در صفحه قرار دهید که
قطر آن موازی محور اعداد حقیقی باشد،مرکز نیمدایره را P بنامید،و به هر نقطه نیمدایره مثل q نفطه ای از محور اعداد حقیقی را متناظر کنید که محا تقاطع pq و محور است.
آیا ارتباطی بین این مساله و مساله ی پنج شنبه ی چهارم (پست 92) وجود دارد؟

ali_hp
16-08-2009, 10:07
زیرمجموعه ی A از اعداد طبیعی را ضعیف می نامیم، هر گاه مجموع هیچ دو عضوی از A بر عضوی از A بخش پذیر نباشد. در ضمن، یک مجموعه را بزرگ می نامیم، هر گاه هیچ زیرمجموعه ی ضعیف دیگری شامل آن نباشد. درستی و نادرستی هریک از گزینه های زیر را بررسی کنید.
الف) نامتناهی مجموعه ی ضعیف نامتناهی داریم.
ب) مجموعه ی ضعیف نامتناهی وجود دارد که بزرگ است.
ج) نامتناهی مجموعه ی ضعیف متناهی داریم که بزرگ هستند.
د) مجموعه ی ضعیف نامتناهی وجود دارد که همه ی اعضای آن بر 1381 بخش پذیر اند.
ﻫ) مجموعه ای ضعیف وجود دارد که از هر سه عدد متوالی حداقل یکی را دارا است.

CppBuilder2006
17-08-2009, 02:51
[ برای مشاهده لینک ، لطفا با نام کاربری خود وارد شوید یا ثبت نام کنید ] مساله یکشنبه پنجم
زیرمجموعه ی A از اعداد طبیعی را ضعیف می نامیم، هر گاه مجموع هیچ دو عضوی از A بر عضوی از A بخش پذیر نباشد. در ضمن، یک مجموعه را بزرگ می نامیم، هر گاه هیچ زیرمجموعه ی ضعیف دیگری شامل آن نباشد. درستی و نادرستی هریک از گزینه های زیر را بررسی کنید.
الف) نامتناهی مجموعه ی ضعیف نامتناهی داریم.
ب) مجموعه ی ضعیف نامتناهی وجود دارد که بزرگ است.
ج) نامتناهی مجموعه ی ضعیف متناهی داریم که بزرگ هستند.
د) مجموعه ی ضعیف نامتناهی وجود دارد که همه ی اعضای آن بر 1381 بخش پذیر اند.
ﻫ) مجموعه ای ضعیف وجود دارد که از هر سه عدد متوالی حداقل یکی را دارا است.


فک کنم توی تعریف مجموعۀ بزرگ یه چیزی جا افتاده!

الف) درست است. مجموعۀ توان های 3 و همۀ زیرمجوعه های نامتناهی آن.
ب) درست است. اجتماع همۀ ضعیف ها را بگیرید.
ج) نادرست هر ضعیف متناهی قابل گسرش است پس بزرگ نیست.
د) درست، یک ضعیف نا متناهی بگیرید همۀ اعضاش رو در 1381 ضرب کنید.
ه) درست است. 1+2k را بگیرید.

ali_hp
17-08-2009, 12:55
فک کنم توی تعریف مجموعۀ بزرگ یه چیزی جا افتاده!

الف) درست است. مجموعۀ توان های 3 و همۀ زیرمجوعه های نامتناهی آن.
ب) درست است. اجتماع همۀ ضعیف ها را بگیرید.
ج) نادرست هر ضعیف متناهی قابل گسرش است پس بزرگ نیست.
د) درست، یک ضعیف نا متناهی بگیرید همۀ اعضاش رو در 1381 ضرب کنید.
ه) درست است. 1+2k را بگیرید.
سلام،با تشکر از CppBuilder عزیز،پاسخهایشان را بررسی می کنیم:
صورت سوال درسته،و چیزی جا نیفتاده
الف)این مجموعه ضعیف نیست،چون جمع هر دو عضوش بر 3 که عضوی از مجموعه است بخش پذیر می باشد.
ب)چرا اجتماع ضعیفها،ضعیف است؟اجتماع ضعیفها می شود مجموعه اعداد طبیعی که ضعیف نیست!
ج)چرا هر ضعیف متناهی قابل گسترش است؟
د)بله،در صورتی که حداقل یک مجموعه ضعیف نامتناهی معرفی کنید،راه حلتان تکمیل می شود.
ه)این مجموعه ضعیف نیست!9+11 بر 5 بخش پذیر است.

yugioh
18-08-2009, 00:13
از اوننجا که کسی جواب نداد سوال می مونه ( اصولا سوال طرح کردن راحت جواب دادن تایپ زیاد می خواد اگه خودتون جواب بدین می گم درسته یا نه این مورد هم همین جوره):31:
اون سوال احتمال رو که دیگه می تونستید جواب بدید، ببینید چقدر احتمالش هست ( دلایل :21:)

اما سوال این هفته چون هیچ کس جز من سوال گراف نمیده من سوال می پرسم تا مهجور نمونه.
ثابت کنید هر گراف همیند مسیری حداقل به طول مینیموم {(1-n (n:درجه ( تعداد رئوس) , (2* دلتاکوچک )( کمترین درجه در رئوس ( تعداد یال وارده به راس درجه ی راس است، دلتا بزرگ بالاترین درجه و دلتا کوچک کمترین درجه است ))} min{n-1 , 2 delta}
عجب ضایع تایپ کردم دستم درد نکنه:21:

برای اینکه حوصله تون سر نره یه سوال ساده ی محاسباتی معروف به مساله نارگیل می زارم، همه می تونند حل کنند ولی با توجه به اینکه افراد کم سن هم هستند گفتم یه سوال برای اونه هم باشه. البته من از هر کسی که جواب بده تشکر می کنم و استقبال می کنم.:
سه دریانورد به جزیره ای میروند و N نارگیل و یک میمون میبینند. نارگیل ها رو جمع می کنند.
شب یکی از اونها مخفیانه 1/3 رو بر میداره ولی یک نارگیل میمونه ( واضحه که اول کل رو تقسیم به 3 کرده یکی مونده) از ره خیر خواهی اون یکی رو به میمون می ده و می خوابه.
بعد از اون که اولی به خواب می ره دومی بیدار میشه به خیال اینکه کل نارگیل ها همینه که مونده 3 قسمت می کنه 1/3 برمیداره و یکی به میمون میده. اونهم می خوابه.
بعدش سومی بیدار میشه واین کار رو تکرار می کنه. باز هم یکی به میمون میرسه.
صبح همه بیدار میشن و به دروغ می گن نارگیل ها همینه بیاید کلش رو تقسیم کنیم. همه رو سه قسمت می کنند هر کدوم 1/3 برمی داره و باز یکی به میمون میرسه.
حداقل تعداد نارگیل ها چندتاست؟

eh_mn
19-08-2009, 11:28
ثابت كنيد براي همه‏ي مقدارهاي مثبت x داريم

[ برای مشاهده لینک ، لطفا با نام کاربری خود وارد شوید یا ثبت نام کنید ]^{\sqrt[12]{x}}&plus;2^{\sqrt[4]{x}}\geq&space;2\times&space;2^{\sqrt[6]{x}}



ـــــــــــــــــــــــــ
21 / 05 / 88

يك نامساوي مهم كه بعضي مواقع در اثبات ساير نامساوي‏ها به كار مي‏ره، نامساوي كشي براي ميانگين حسابي و هندسي است. كه بيان مي‏كند اگر xiها اعداد نامنفي باشند آنگاه


[ برای مشاهده لینک ، لطفا با نام کاربری خود وارد شوید یا ثبت نام کنید ]{x_1&plus;\dots&plus;x_n}{n}\geq&space;\sqrt[n]{x_1\dots&space;x_n}

بعضي از نامساوي‏هاي معروف رو مي‏تونيد در

برای مشاهده محتوا ، لطفا وارد شوید یا ثبت نام کنید
پيدا كنيد.

در حل اين مسأله هم دو بار از نامساوي ميانگين حسابي و هندسي استفاده مي‏شه. داريم


[ برای مشاهده لینک ، لطفا با نام کاربری خود وارد شوید یا ثبت نام کنید ]^{\frac{1}{12}}&plus;x^{\frac{1}{4}}\geq&space;2\s qrt{x^{\frac{1}{4}&plus;\frac{1}{12}}}&space;=&space;2x^{\frac{1}{6 }}

بنابراين


[ برای مشاهده لینک ، لطفا با نام کاربری خود وارد شوید یا ثبت نام کنید ]{align}\nonumber&space;2^{x^{\frac{1}{12 }}}&plus;2^{x^{\frac{1}{4}}}&space;&&space;\geq&space;2\sqrt{2^{x^{\frac{1}{12}}&plus;x^{\frac{1}{14}}} }&space;\cr&space;&&space;\geq&space;2&space;\sqrt{2^{2x^{\frac{1}{6}}}}&space;\cr&space;&&space;=&space;2\times&space;2^{\sqrt[6]{x}}&space;\end{align}



ـــــــــــــــــــــــــ
28 / 05 / 88

eh_mn
19-08-2009, 13:38
فرض كنيد a، ‏b و c اعداد نامنفي با مجموع برابر يك باشند ثابت كنيد


[ برای مشاهده لینک ، لطفا با نام کاربری خود وارد شوید یا ثبت نام کنید ]^2b&plus;b^2c&plus;c^2a\leq\frac{4}{27}



ـــــــــــــــــ
28 / 05 / 88

CppBuilder2006
20-08-2009, 03:50
سلام،با تشکر از CppBuilder عزیز،پاسخهایشان را بررسی می کنیم:
صورت سوال درسته،و چیزی جا نیفتاده
الف)این مجموعه ضعیف نیست،چون جمع هر دو عضوش بر 3 که عضوی از مجموعه است بخش پذیر می باشد.
ب)چرا اجتماع ضعیفها،ضعیف است؟اجتماع ضعیفها می شود مجموعه اعداد طبیعی که ضعیف نیست!
ج)چرا هر ضعیف متناهی قابل گسترش است؟
د)بله،در صورتی که حداقل یک مجموعه ضعیف نامتناهی معرفی کنید،راه حلتان تکمیل می شود.
ه)این مجموعه ضعیف نیست!9+11 بر 5 بخش پذیر است.

:31:
آره با عجله جواب دادم! 4 صبح بهتر از این نمی شد فکر کرد!

CppBuilder2006
20-08-2009, 07:20
فرض كنيد a، ‏b و c اعداد نامنفي با مجموع برابر يك باشند ثابت كنيد


[ برای مشاهده لینک ، لطفا با نام کاربری خود وارد شوید یا ثبت نام کنید ]^2b&plus;b^2c&plus;c^2a\leq\frac{4}{27}



ـــــــــــــــــ
28 / 05 / 88

[ برای مشاهده لینک ، لطفا با نام کاربری خود وارد شوید یا ثبت نام کنید ]
البته فک کنم راه حل قشنگ تری هم باشه!

mofidy1
20-08-2009, 15:19
با سلام

دایره ی ثابتی به شعاع 1 و مثلث متساوی الاضلاع محاط در این دایره را در نظر بگیرید. طول ضلع این مثلث را L فرض می کنیم. فرض کنید وتری از این دایره به طور تصادفی انتخاب شده باشد. احتمال این که طول وتر - که آن را M می گیریم - از طول ضلع مثلث محاط در دایره - که L فرض کردیم - بیشتر باشد، چه قدر است؟

موفق باشید.

15 مرداد 1388

با سلام

با تشکر از دوستان که درباره ی این مساله به خوبی با یکدیگر بحث کردند.همان طور که chessmathter در پست 106 ([ برای مشاهده لینک ، لطفا با نام کاربری خود وارد شوید یا ثبت نام کنید ]) به طور کامل توضیح دادند، این مساله «پارادکس برتراند» نام دارد. البته ای کاش ایشان منبع خود را هم ذکر می کردند. این مقاله در سایت انجمن ریاضیدانان جوان anjoman.ir که نویسنده ی آن «پانيذ نوري اسكوئي
» است، منتشر شده است. البته این مقاله در اصل ترجمه ی ناقصی از مقاله ای است که آقای Melvin Lax در UMAPJornal سال 1986 به چاپ رسانده ند. ایشان در این مقاله با چهار «راه حل» به جواب های مختلف می رسند که سه جواب آن را در پست 106 ([ برای مشاهده لینک ، لطفا با نام کاربری خود وارد شوید یا ثبت نام کنید ]) می بینید و جواب چهارم آن هم «دو یازدهم» است. بد نیست بدانید که همه ی این راه حل ها برفرض هایی در زمینه ی نحوه ی انتخاب وتر مبتنی هستند. البته هر کدام از این راه حل ها اشکالاتی دارد که به آن ها نمی پردازیم. از CppBuilder2006 و < A L I > که روی مساله بجث کردند ممنونم.

روش حل مساله:

از بحث بالا می توان نتیجه گرفت که گاهی مساله ای به ظاهر کامل و دقیق است اما در واقع چنین نیست. برای مساله ی بالا هیچ فرض درستی وجود ندارد مگر این که اطلاعات بیشتری در مورد مساله داده شود. می توان برای هر یک از راه حل های بالا مدلی فیزیکی با فرض های ویژه ی خودش ارائه کرد. این ارتباط حیاتی بین ریاضیات و کاربردهایش مهم ترین درس پارادکس برتراند است.

منبع فارسی برای مطالعه ی بیشتر: جنگ ریاضی دانشجو، دانشکده ی علوم دانشگاه تهران، جلد پنجم، بهمن 1368، صفحه ی 135

موفق باشید.

29 مرداد 1388

mofidy1
20-08-2009, 15:33
با سلام

احتمال این که در نفس بعدی، یک مولکول از هوای بازدم بوعلی سینا را به شش هایتان وارد کنید، چه قدر است؟

اطلاعات مورد نیاز: می توان گفت که جو زمین «10 به توان 44» مولکول دارد و تعداد مولکول ها در هر بازدم «10 به توان 22».

موفق باشید.

29 مرداد 1388

mir@
22-08-2009, 03:36
پاسخ مسئله شنبه چهارم


عبارت زیر را تجزیه کنید:


[ برای مشاهده لینک ، لطفا با نام کاربری خود وارد شوید یا ثبت نام کنید ]

راهنمایی: از معادله زیر کمک بگیرید:


[ برای مشاهده لینک ، لطفا با نام کاربری خود وارد شوید یا ثبت نام کنید ]


سپس عبارت زیر را تجزیه کنید:


[ برای مشاهده لینک ، لطفا با نام کاربری خود وارد شوید یا ثبت نام کنید ]


ملاحظه می شود که a و b و c ریشه های چندجمله ای P هستند. لذا با قردادن آنها در P جمع هر سه معادله داریم:


[ برای مشاهده لینک ، لطفا با نام کاربری خود وارد شوید یا ثبت نام کنید ] &space;%5Cqquad&space;%5Cqquad&space;%28ab&plus;bc&plus;ca%29%28a&plus;b&plus;c%29-3abc=0&space;%5CRightarrow&space;a%5E3&plus;b%5E3&plus;c%5E3-3abc=%28a&plus;b&plus;c%29%28a%5E2&plus;b%5E2&plus;c%5E2-ab-ac-bc%29%5Cqquad&space;%281%29%5C%5C&space;%281%29&space;%5Crightarrow&space; a&plus;b&plus;c=0&space;%5CRightarrow&space;a%5E3&plus;b%5E3&plus;c%5E3=3abc



با توجه به تجزیه بالا داریم


[ برای مشاهده لینک ، لطفا با نام کاربری خود وارد شوید یا ثبت نام کنید ]

mir@
22-08-2009, 03:40
.
فرض کنید M نقطه ای است داخل مثلث ABC . ثابت کنید داریم:


[ برای مشاهده لینک ، لطفا با نام کاربری خود وارد شوید یا ثبت نام کنید ]

amintnt
22-08-2009, 11:48
[ برای مشاهده لینک ، لطفا با نام کاربری خود وارد شوید یا ثبت نام کنید ]


[ برای مشاهده لینک ، لطفا با نام کاربری خود وارد شوید یا ثبت نام کنید ]

[ برای مشاهده لینک ، لطفا با نام کاربری خود وارد شوید یا ثبت نام کنید ]

از جمع دو نامساوی بالا و با توجه به اینکه [ برای مشاهده لینک ، لطفا با نام کاربری خود وارد شوید یا ثبت نام کنید ] داریم:

[ برای مشاهده لینک ، لطفا با نام کاربری خود وارد شوید یا ثبت نام کنید ]

و بعد از ساده شدن:

[ برای مشاهده لینک ، لطفا با نام کاربری خود وارد شوید یا ثبت نام کنید ]

CppBuilder2006
22-08-2009, 17:37
با سلام

احتمال این که در نفس بعدی، یک مولکول از هوای بازدم بوعلی سینا را به شش هایتان وارد کنید، چه قدر است؟

اطلاعات مورد نیاز: می توان گفت که جو زمین «10 به توان 44» مولکول دارد و تعداد مولکول ها در هر بازدم «10 به توان 22».

موفق باشید.

29 مرداد 1388

اگه ملوکول ها را بی نهایت کوچک باشند یه ملکول از بازدم جولیوس سزار با احتمال 1 در دم ماست.
اگه یه قطره رنگ توی دریا بریزیم به همه جاش می رسه!

sherlockholmz
25-08-2009, 11:09
بدون استفاده از هوپيتال و هم ارزي حد زير را محاسبه نمائيد:


[ برای مشاهده لینک ، لطفا با نام کاربری خود وارد شوید یا ثبت نام کنید ] 1%29%7D%7Bx%7D


موفق باشيد.

eh_mn
26-08-2009, 23:01
فرض كنيد a، ‏b و c اعداد نامنفي با مجموع برابر يك باشند ثابت كنيد


[ برای مشاهده لینک ، لطفا با نام کاربری خود وارد شوید یا ثبت نام کنید ]^2b&plus;b^2c&plus;c^2a\leq\frac{4}{27}



ـــــــــــــــــ
28 / 05 / 88

ضمن عذر خواهي بابت اينكه دير شد!

اين جا قبلا خالي نبود!! راه حلي نوشته شده بود كه بعد فهميدم غلط هست.

اما جناب CppBuilder2006 لطف كردن و در


برای مشاهده محتوا ، لطفا وارد شوید یا ثبت نام کنید

راه حلي رو نوشتن كه فكر مي‏كنم احتياج به كمي توضيح داره.

مثلا در مورد اينكه تابع متقارن هست. منظور چيه؟





ـــــــــــــــــــ
04 / 06 / 88

eh_mn
26-08-2009, 23:05
ثابت كنيد براي هر سه عدد مثبت a، ‏b و c داريم


[ برای مشاهده لینک ، لطفا با نام کاربری خود وارد شوید یا ثبت نام کنید ](a^2b&plus;b^2c&plus;c^2a)(ab^2&plus;bc^2&plus;ca^2)\geq&space;9a^ 2b^2c^2



ــــــــــــــــــــ
04 / 06 / 88

mofidy1
27-08-2009, 10:58
با سلام

احتمال این که در نفس بعدی، یک مولکول از هوای بازدم بوعلی سینا را به شش هایتان وارد کنید، چه قدر است؟

اطلاعات مورد نیاز: می توان گفت که جو زمین «10 به توان 44» مولکول دارد و تعداد مولکول ها در هر بازدم «10 به توان 22».

موفق باشید.

29 مرداد 1388

با سلام

همان طور که CppBuilder2006 گفتند، اگر بخواهیم مساله را به طور شهودی حل کنیم، در هر نفس یک مولکول از این بازدم هست. اما دقت کنید که سطح سوال، پیش دانش گاهی ذکر شده و ترفند مساله دقیق تر کردن این وضعیت است. واضح است که احتمال این که یکی از این مولکول ها «بوعلی سینایی!!» نباشد،عبارت است از:


[ برای مشاهده لینک ، لطفا با نام کاربری خود وارد شوید یا ثبت نام کنید ]{10^{44}-10^{22}}{10^{44}}=1-10^{-22}

بنابر این احتمال این که نفس بعدی شما کاملاً «غیربوعلی سینایی» باشد، عبارت است از:


[ برای مشاهده لینک ، لطفا با نام کاربری خود وارد شوید یا ثبت نام کنید ](1-10^{-22}&space;\right&space;)^{10^{22}}=\left&space;(1-\frac{1}{10^{22}}&space;\right&space;)^{10^{22}}

این عدد را نمی توان با ماشین حساب یا حتی با بعضی از رایانه ها محاسبه کرد، چرا که نتیجه 1 خواهد بود. اما با توجه به این که


[ برای مشاهده لینک ، لطفا با نام کاربری خود وارد شوید یا ثبت نام کنید ]{n\to&space;\infty}\left&space;(&space;1-\frac{1}{n}&space;\right&space;)^n=\frac{1}{e}\approx&space;./368

پس احتمال این که نفس بعدی شما حاوی مولکولی از بازدم «این بزرگ مرد ایرانی» باشد، تقریباً 63 درصد است.


آموزش حل مساله:

ماشین حساب و رایانه جایگزین ریاضیات نظری نیستند و برای بعضی از مسائل -احتمالاً- تنها راه، استفاده از ابزار ریاضیات نظری است.

موفق باشید.

5 شهریور 1388

mofidy1
27-08-2009, 11:35
با سلام

سعی کنید شکل زیر را با حرکت پیوسته ی قلم، بدون این که از یک مسیر، دوبار عبور نمایید، رسم کنید:



[ برای مشاهده لینک ، لطفا با نام کاربری خود وارد شوید یا ثبت نام کنید ]


موفق باشید.

5 شهریور 1388

CppBuilder2006
28-08-2009, 08:19
ضمن عذر خواهي بابت اينكه دير شد!

اين جا قبلا خالي نبود!! راه حلي نوشته شده بود كه بعد فهميدم غلط هست.

اما جناب CppBuilder2006 لطف كردن و در


برای مشاهده محتوا ، لطفا وارد شوید یا ثبت نام کنید

راه حلي رو نوشتن كه فكر مي‏كنم احتياج به كمي توضيح داره.

مثلا در مورد اينكه تابع متقارن هست. منظور چيه؟





ـــــــــــــــــــ
04 / 06 / 88

سلام
منظورم از متقارن اصطلاح تخصصی نبود. منظورم این بود که همۀ متغیرها نقش یکسانی دارند. و هر چیز رو در مورد یک متغیر ثابت کنیم به خاطر تقارن برای دو تای دیگه هم ثابت میشه.
البته اون چیزی که نوشتم یه خرده ابهام داره.
پستای این thread به طور خودکار برای من ایمیل میشن! چون درش اشتراک دارم. برای همین من پست شما رو قبل از edit دارم:20:.

CppBuilder2006
28-08-2009, 08:49
با سلام

سعی کنید شکل زیر را با حرکت پیوسته ی قلم، بدون این که از یک مسیر، دوبار عبور نمایید، رسم کنید:



[ برای مشاهده لینک ، لطفا با نام کاربری خود وارد شوید یا ثبت نام کنید ]


موفق باشید.

5 شهریور 1388

یه بار با این جور شکلا بازی می کردم دیدم همۀ شکلایی رو که با حرکت پیوسته ی قلم، بدون این که مسیری دوبار طی بشه، رسم میکنیم و به نقطۀ اول بر میگردیم با دو تا رنگ میشه رنگ کرد. مثل قضیۀ چهار رنگ. حالا اگه این حدس من درست باشه و ثابت بشه، چون گوَشه شکل (مربع بزرگ) بالا رو با دو رنگ نمیشه رنگ کرد پس رسم پیوسته ی بدون تکرار با بازگشت به نقطۀ اول امکان نداره.

maziar92142
28-08-2009, 12:56
تایید میشه
برای اتفاق افتادن باید مرتبه ی 2 راس فرد وبقیه زوج باشه که اینجا 4 راس فرده

mir@
28-08-2009, 22:32
حل مسئله شنبه پنجم


.
فرض کنید M نقطه ای است داخل مثلث ABC . ثابت کنید داریم:


[ برای مشاهده لینک ، لطفا با نام کاربری خود وارد شوید یا ثبت نام کنید ]




[ برای مشاهده لینک ، لطفا با نام کاربری خود وارد شوید یا ثبت نام کنید ]


[ برای مشاهده لینک ، لطفا با نام کاربری خود وارد شوید یا ثبت نام کنید ]

[ برای مشاهده لینک ، لطفا با نام کاربری خود وارد شوید یا ثبت نام کنید ]

از جمع دو نامساوی بالا و با توجه به اینکه [ برای مشاهده لینک ، لطفا با نام کاربری خود وارد شوید یا ثبت نام کنید ] داریم:

[ برای مشاهده لینک ، لطفا با نام کاربری خود وارد شوید یا ثبت نام کنید ]

و بعد از ساده شدن:

[ برای مشاهده لینک ، لطفا با نام کاربری خود وارد شوید یا ثبت نام کنید ]

جناب آقای amintnt -مدظله العالی-
آفرین بر نفست باد که خوش بردی بوی

mir@
28-08-2009, 22:40
.
مقدار مجموع زیر را برای تمامی n های مثبت بیابید:


[ برای مشاهده لینک ، لطفا با نام کاربری خود وارد شوید یا ثبت نام کنید ] m%7Bn%7D%7Bk%7D%7D%7B%5Cbinom%7B2n-1%7D%7Bk%7D%7D


راهنمایی: عبارت زیر را ساده کرده و از قاعده تلسکوپی استفاده کنید:


[ برای مشاهده لینک ، لطفا با نام کاربری خود وارد شوید یا ثبت نام کنید ] Cbinom%7B2n%7D%7Bk%7D%7D-%5Cfrac%7B%5Cbinom%7Bn%7D%7Bk&plus;1%7D%7D%7B%5Cbinom%7 B2n%7D%7Bk&plus;1%7D%7D

ali_hp
30-08-2009, 08:55
زیرمجموعه ی A از اعداد طبیعی را ضعیف می نامیم، هر گاه مجموع هیچ دو عضوی از A بر عضوی از A بخش پذیر نباشد. در ضمن، یک مجموعه را بزرگ می نامیم، هر گاه هیچ زیرمجموعه ی ضعیف دیگری شامل آن نباشد. درستی و نادرستی هریک از گزینه های زیر را بررسی کنید.
الف) نامتناهی مجموعه ی ضعیف نامتناهی داریم.
ب) مجموعه ی ضعیف نامتناهی وجود دارد که بزرگ است.
ج) نامتناهی مجموعه ی ضعیف متناهی داریم که بزرگ هستند.
د) مجموعه ی ضعیف نامتناهی وجود دارد که همه ی اعضای آن بر 1381 بخش پذیر اند.
ﻫ) مجموعه ای ضعیف وجود دارد که از هر سه عدد متوالی حداقل یکی را دارا است.
الف)درست است.با استفاده ازقضیه باقیمانده چینی و به صورت استقرایی ،با شروع از هر عدد بزرگتر از دو،می توان یک مجموعه ضعیف نامتناهی صعودی درست کر د!
ب)درست است.مجموعه ضعیفی را در نظر بگیرید که با شروع از سه ساخته می شود،و در هر مرحله کوچکترین عدد ممکن به آن اضافه می شود!
ج)درست است.فرض کنید فقط تعدادی متناهی مجموعه ضعیف نامتناهی بزرگ داریم،حاصلضرب کوچکترین عضو این مجموعه ها را به عنوان عضو شروع یک مجموعه جدید در نظر بگیرید!
د)درست است .یک مجموعه ضعیف نامتناهی در نظر بگیرید و اعضای آن را در 1381 ضرب کنید!
ه)درست نیست.اگر چنین مجموعه ای وجود داشته باشد پس 3 اولین عضو آن است،حال اگر عضو بعدی مثلا به صورت 3k+1 باشد،همه اعضا باید به همین صورت باشند،پس همه 3k+1 ها عضو این مجموعه هستند!(چرا؟)
که تناقض است!(چرا؟)

ali_hp
30-08-2009, 09:05
27) در شکل مقابل مثلث ABC قائم الزاویه است (B=90o) و AB=10-π و BC=6. نیم استوانه ای با شعاع واحد و محور عمود بر AB، بین نقاط A وC مانع شده است.
[ برای مشاهده لینک ، لطفا با نام کاربری خود وارد شوید یا ثبت نام کنید ]
مورچه بنا به دلایلی (!) باید هر چه سریع تر از نقطه ی A به لانه اش در نقطه ی C برود. طول کوتاه ترین مسیر ممکن چقدر است؟
(مساله مرحله اول بیست یکمین المپیاد ریاضی ایران،برای دیدن کلیه مساِیل به
[ برای مشاهده لینک ، لطفا با نام کاربری خود وارد شوید یا ثبت نام کنید ]
بروید.)

amintnt
30-08-2009, 12:55
.
مقدار مجموع زیر را برای تمامی n های مثبت بیابید:


[ برای مشاهده لینک ، لطفا با نام کاربری خود وارد شوید یا ثبت نام کنید ] m%7Bn%7D%7Bk%7D%7D%7B%5Cbinom%7B2n-1%7D%7Bk%7D%7D


راهنمایی: عبارت زیر را ساده کرده و از قاعده تلسکوپی استفاده کنید:


[ برای مشاهده لینک ، لطفا با نام کاربری خود وارد شوید یا ثبت نام کنید ] Cbinom%7B2n%7D%7Bk%7D%7D-%5Cfrac%7B%5Cbinom%7Bn%7D%7Bk&plus;1%7D%7D%7B%5Cbinom%7 B2n%7D%7Bk&plus;1%7D%7D






وقتی ساده میکنیم و از قاعده ی تلسکوپی استفاده میکنیم در مخرج کسر دوم به !(1-) میرسیم که تعریف نشده... چی کارش کنیم؟ :eh:
بعد این دومیه معادل بالاییه؟

CppBuilder2006
30-08-2009, 14:21
۲۷) در شکل مقابل مثلث ABC قائم الزاویه است (B=90o) و AB=10-π و BC=6. نیم استوانه ای با شعاع واحد و محور عمود بر AB، بین نقاط A وC مانع شده است.


[ برای مشاهده لینک ، لطفا با نام کاربری خود وارد شوید یا ثبت نام کنید ]

مورچه بنا به دلایلی (!) باید هر چه سریع تر از نقطه ی A به لانه اش در نقطه ی C برود. طول کوتاه ترین مسیر ممکن برابر است با

الف) [ برای مشاهده لینک ، لطفا با نام کاربری خود وارد شوید یا ثبت نام کنید ] ب) [ برای مشاهده لینک ، لطفا با نام کاربری خود وارد شوید یا ثبت نام کنید ] ج) 10

د) 7+π ﻫ) 11

فک کنم جواب بشه رادیکال 136 یعنی الف
البته منظور از AB=10-pio رو بدون احتساب تکۀ زیر استوانه می گیریم.

mir@
31-08-2009, 21:16
وقتی ساده میکنیم و از قاعده ی تلسکوپی استفاده میکنیم در مخرج کسر دوم به !(1-) میرسیم که تعریف نشده... چی کارش کنیم؟ :eh:
بعد این دومیه معادل بالاییه؟

من که حلشو میبینم به نظرم این طوری نیست. شما یه کم بیشتر دقت کن. دومیه تقریبا معادل بالاییه.

saber57
01-09-2009, 16:12
27) در شکل مقابل مثلث ABC قائم الزاویه است (B=90o) و AB=10-π و BC=6. نیم استوانه ای با شعاع واحد و محور عمود بر AB، بین نقاط A وC مانع شده است.



[ برای مشاهده لینک ، لطفا با نام کاربری خود وارد شوید یا ثبت نام کنید ]
مورچه بنا به دلایلی (!) باید هر چه سریع تر از نقطه ی A به لانه اش در نقطه ی C برود. طول کوتاه ترین مسیر ممکن چقدر است؟
(مساله مرحله اول بیست یکمین المپیاد ریاضی ایران،برای دیدن کلیه مساِیل به
[ برای مشاهده لینک ، لطفا با نام کاربری خود وارد شوید یا ثبت نام کنید ]


بروید.)

کوتاهترین مسیر همون AC هست پس کافیه مقدار وتر بدست بیاد . محیط نیم دایره برابر pi هست . با توجه به شکل مشخصه که اختلاف طول وترAC و ضلع مقابلش AB برابر محیط نیم استوانه یا pi هست !!بنابراین:

AC-AB=pi ---->AC=AB+pi----->AC=10-pi+pi----->AC=10

بنابراین گزینه ج صحیح میباشد

eh_mn
02-09-2009, 11:21
ثابت كنيد براي هر سه عدد مثبت a، ‏b و c داريم


[ برای مشاهده لینک ، لطفا با نام کاربری خود وارد شوید یا ثبت نام کنید ](a^2b&plus;b^2c&plus;c^2a)(ab^2&plus;bc^2&plus;ca^2)\geq&space;9a^ 2b^2c^2



ــــــــــــــــــــ
04 / 06 / 88

با استفاده از نامساوي ميانگين حسابي-ميانگين هندسي داريم


[ برای مشاهده لینک ، لطفا با نام کاربری خود وارد شوید یا ثبت نام کنید ]{align}&space;{\nonumber&space;\frac{a^2b&plus;b^2c &plus;c^2a}{3}&space;&&space;\geq&space;\sqrt[3]{a^3b^3c^3}\cr&space;\frac{ab^2&plus;bc^2&plus;ca^2}{3}&space;&&space;\geq&space;\sqrt[3]{a^3b^3c^3}\cr&space;}\end{align}

با تغييري كوچك در اين نامساوي‌ها و ضرب طرفين، حكم به راحتي به دست مي‌آيد.




ــــــــــــــــــــــ
11 / 06 / 88

eh_mn
02-09-2009, 11:30
يكي ديگر از نامساوي‌هاي مهم، نامساوي كوشي-شوارتز است:
اگر a1 و a2 و ... و an و b1 و b2 و ... و bn اعدادي نامنفي باشند آنگاه


[ برای مشاهده لینک ، لطفا با نام کاربری خود وارد شوید یا ثبت نام کنید ](a_1b_1&plus;a_2b_2&plus;\dots&plus;a_nb_n)^2\leq&space;(a_1^ 2&plus;a_2^2&plus;\dots&plus;a_n^2)(b_1^2&plus;b_2^2&plus;\dots&plus;b_n^2)

با استفاده از نامساوي كوشي-شوارتز مسأله‌ي زير را حل كنيد.

فرض كنيد [ برای مشاهده لینک ، لطفا با نام کاربری خود وارد شوید یا ثبت نام کنید ](x) يك چندجمله‌اي با ضرايب مثبت باشد. ثابت كنيد كه اگر رابطه‌ي


[ برای مشاهده لینک ، لطفا با نام کاربری خود وارد شوید یا ثبت نام کنید ](\frac{1}{x}&space;\right&space;)\geq&space;\frac{1} {P(x)}
براي x=1 برقرار باشد آنگاه براي هر x>0 برقرار است.



ــــــــــــــــــ
11 / 06 / 88

mir@
05-09-2009, 13:09
حل مسئله شنبه ششم


.
مقدار مجموع زیر را برای تمامی n های مثبت بیابید:


[ برای مشاهده لینک ، لطفا با نام کاربری خود وارد شوید یا ثبت نام کنید ] m%7Bn%7D%7Bk%7D%7D%7B%5Cbinom%7B2n-1%7D%7Bk%7D%7D



راهنمایی: عبارت زیر را ساده کرده و از قاعده تلسکوپی استفاده کنید:





[ برای مشاهده لینک ، لطفا با نام کاربری خود وارد شوید یا ثبت نام کنید ] Cbinom%7B2n%7D%7Bk%7D%7D-%5Cfrac%7B%5Cbinom%7Bn%7D%7Bk&plus;1%7D%7D%7B%5Cbinom%7 B2n%7D%7Bk&plus;1%7D%7D











با تشکر از استاد amintnt که برای حل این مسئله سعی بلیغ نمودند.


[ برای مشاهده لینک ، لطفا با نام کاربری خود وارد شوید یا ثبت نام کنید ]{\binom{n}{k}}{\binom{2n}{k}}-\frac{\binom{n}{k&plus;1}}{\binom{2n}{k&plus;1}}=\\&space;\frac{n! (2n-k)!}{(n-k)!2n!}-\frac{n!(2n-k-1)!}{(n-k-1)!2n!}\\&space;=\frac{n!(2n-1-k)!}{(n-k)!(2n-1)!}\left(&space;\frac{2n-k}{2n}-\frac{n-k}{2n}&space;\right&space;)\\&space;=\frac{1}{2}\frac{\binom{n}{k}}{ \binom{2n-1}{k}}\\&space;\Rightarrow&space;S=2&space;\sum_{k=0}^n&space;\left(&space;\frac {\binom{n}{k}}{\binom{2n}{k}}-\frac{\binom{n}{k&plus;1}}{\binom{2n}{k&plus;1}}&space;\right&space;)\\&space; =2\left(&space;\frac{\binom{n}{0}}{\binom{2n}{0}}-\frac{\binom{n}{n&plus;1}}{\binom{2n}{n&plus;1}}&space;\right&space;)=2

mir@
05-09-2009, 13:17
.
لطفا حاصل مجموع زیر را بیابید:


[ برای مشاهده لینک ، لطفا با نام کاربری خود وارد شوید یا ثبت نام کنید ]{n=1}^\infty&space;\tan^{-1}\frac{2}{n^2}


راهنمایی: از اتحاد زیر استفاده کنید:


[ برای مشاهده لینک ، لطفا با نام کاربری خود وارد شوید یا ثبت نام کنید ]^{-1}\frac{1}{n-1}-\tan^{-1}\frac{1}{n&plus;1}=\tan^{-1}\frac{2}{n^2}


توجه: [ برای مشاهده لینک ، لطفا با نام کاربری خود وارد شوید یا ثبت نام کنید ]^{-1}(x)\leftrightarrow&space;\arctan(x)

saber57
05-09-2009, 15:05
.
لطفا حاصل مجموع زیر را بیابید:


[ برای مشاهده لینک ، لطفا با نام کاربری خود وارد شوید یا ثبت نام کنید ]{n=1}^\infty&space;\tan^{-1}\frac{2}{n^2}


راهنمایی: از اتحاد زیر استفاده کنید:


[ برای مشاهده لینک ، لطفا با نام کاربری خود وارد شوید یا ثبت نام کنید ]^{-1}\frac{1}{n-1}-\tan^{-1}\frac{1}{n&plus;1}=\tan^{-1}\frac{2}{n^2}


توجه: [ برای مشاهده لینک ، لطفا با نام کاربری خود وارد شوید یا ثبت نام کنید ]^{-1}(x)\leftrightarrow&space;\arctan(x)


در این مساله هم از قاعده تلسکوپی استفاده میکنیم فقط یک نکته جالب در حل مساله نهفته هست و اونم یک عبارت اضافه (بجز عبارات اول و آخر)هست .طبق اتحادی که خودتون فرمودید و براحتی با استفاده از اتحاد تانژانت تفاضل قابل اثبات هست و داریم:

[ برای مشاهده لینک ، لطفا با نام کاربری خود وارد شوید یا ثبت نام کنید ]{n=1}^{\infty&space;}tan^{-1}\left&space;(&space;\frac{1}{n-1}&space;\right&space;)-&space;tan^{-1}\left&space;(&space;\frac{1}{n&plus;1}&space;\right&space;)=tan^{-1}\infty&space;-tan^{-1}\frac{1}{2}&plus;tan^{-1}1-tan^{-1}\frac{1}{3}&plus;tan^{-1}\frac{1}{2}-tan^{-1}\frac{1}{4}&plus;....-tan^{-1}\frac{1}{\infty&space;}=tan^{-1}\infty&space;&plus;tan^{-1}1-tan^{-1}0=\frac{\pi&space;}{2}&plus;\frac{\pi&space;}{4}=\frac{3\pi&space;}{4}

فقط عبارت [ برای مشاهده لینک ، لطفا با نام کاربری خود وارد شوید یا ثبت نام کنید ]^{-1}1 وجود دارد که قرینه اش در کل مجموع وجود ندارد ولی تفاضل عبارات اول و آخر را داریم


البته به فرم دیگر :


[ برای مشاهده لینک ، لطفا با نام کاربری خود وارد شوید یا ثبت نام کنید ]{n=1}^{\infty&space;}tan^{-1}\left&space;(&space;\frac{2}{n^2}&space;\right&space;)=\sum_{n=1}^{\inft y&space;}\left&space;(tan^{-1}\left&space;(&space;\frac{1}{n-1}&space;\right&space;)-&space;tan^{-1}\left&space;(&space;\frac{1}{n&plus;1}&space;\right&space;)&space;\right&space;)


[ برای مشاهده لینک ، لطفا با نام کاربری خود وارد شوید یا ثبت نام کنید ]{n=1}^{\infty&space;}\left&space;(tan^{-1}\left&space;(&space;\frac{1}{n-1}&space;\right&space;)-&space;tan^{-1}\left&space;(&space;\frac{1}{n}&space;\right&space;)&space;\right&space;)&plus;\sum_{n=1} ^{\infty&space;}\left&space;(tan^{-1}\left&space;(&space;\frac{1}{n}&space;\right&space;)-&space;tan^{-1}\left&space;(&space;\frac{1}{n&plus;1}&space;\right&space;)&space;\right&space;)


[ برای مشاهده لینک ، لطفا با نام کاربری خود وارد شوید یا ثبت نام کنید ](tan^{-1}\infty&space;-tan^{-1}0&space;\right&space;)&plus;\left&space;(&space;tan^{-1}1-tan^{-1}0&space;\right&space;)=\frac{\pi&space;}{2}&plus;\frac{\pi&space;}{4}=\frac{3 \pi&space;}{4}




برای مشاهده محتوا ، لطفا وارد شوید یا ثبت نام کنید

Parser
06-09-2009, 10:49
به نام معشوق ازلي
حل مسئله ي چهارشنبه‌ي هفتم

يكي ديگر از نامساوي‌هاي مهم، نامساوي كوشي-شوارتز است:
اگر a1 و a2 و ... و an و b1 و b2 و ... و bn اعدادي نامنفي باشند آنگاه


[ برای مشاهده لینک ، لطفا با نام کاربری خود وارد شوید یا ثبت نام کنید ](a_1b_1&plus;a_2b_2&plus;\dots&plus;a_nb_n)^2\leq&space;(a_1^ 2&plus;a_2^2&plus;\dots&plus;a_n^2)(b_1^2&plus;b_2^2&plus;\dots&plus;b_n^2)

با استفاده از نامساوي كوشي-شوارتز مسأله‌ي زير را حل كنيد.

فرض كنيد [ برای مشاهده لینک ، لطفا با نام کاربری خود وارد شوید یا ثبت نام کنید ](x) يك چندجمله‌اي با ضرايب مثبت باشد. ثابت كنيد كه اگر رابطه‌ي


[ برای مشاهده لینک ، لطفا با نام کاربری خود وارد شوید یا ثبت نام کنید ](\frac{1}{x}&space;\right&space;)\geq&space;\frac{1} {P(x)}
براي x=1 برقرار باشد آنگاه براي هر x>0 برقرار است.



[ برای مشاهده لینک ، لطفا با نام کاربری خود وارد شوید یا ثبت نام کنید ](P(1))^{2}%5Cgeq%201
[ برای مشاهده لینک ، لطفا با نام کاربری خود وارد شوید یا ثبت نام کنید ](x)=A_{n}x^n+A_{n-1}x^{n-1}+...+A_{0}
[ برای مشاهده لینک ، لطفا با نام کاربری خود وارد شوید یا ثبت نام کنید ](A_0+A_1+...+A_n)^2%5Cge q%201
[ برای مشاهده لینک ، لطفا با نام کاربری خود وارد شوید یا ثبت نام کنید ](x)P(%5Cfrac{1}{x})=(A_{n}{(%5Cfrac{1}{ x})}^n+A_{n-1}{(%5Cfrac{1}{x})}^{n-1}+...+A_{0})(A_{n}x^n+A_{n-1}x^{n-1}+...+A_{0})

[ برای مشاهده لینک ، لطفا با نام کاربری خود وارد شوید یا ثبت نام کنید ](a_0b_0+a_1b_1+...+a_nb_n)^2%5Cleq%20(a_ 0^2+a_1^2+...+a_n^2)(b_0^2+b_1^2+...+b_n^2)
[ برای مشاهده لینک ، لطفا با نام کاربری خود وارد شوید یا ثبت نام کنید ]{A_{n}%5Cfrac{1}{x^n}}=a_0
[ برای مشاهده لینک ، لطفا با نام کاربری خود وارد شوید یا ثبت نام کنید ]{A_{n}x^n}=b_0
[ برای مشاهده لینک ، لطفا با نام کاربری خود وارد شوید یا ثبت نام کنید ]{A_{n-1}%5Cfrac{1}{x^{n-1}}}=a_1
[ برای مشاهده لینک ، لطفا با نام کاربری خود وارد شوید یا ثبت نام کنید ]{A_{n-1}x^{n-1}}=b_1

...

[ برای مشاهده لینک ، لطفا با نام کاربری خود وارد شوید یا ثبت نام کنید ](A_0+A_1+...+A _n)^2%5Cleq%20(A_n%5Ctfrac{1}{x^n}+...+A_0)(A_nx^n +...+A_0)
[ برای مشاهده لینک ، لطفا با نام کاربری خود وارد شوید یا ثبت نام کنید ](%5Cfrac{1}{x })P(x)
[ برای مشاهده لینک ، لطفا با نام کاربری خود وارد شوید یا ثبت نام کنید ](x)%3E0
[ برای مشاهده لینک ، لطفا با نام کاربری خود وارد شوید یا ثبت نام کنید ](%5Cfrac{1}{x})%5Cgeq%2 0%5Cfrac{1}{P(x)}

ali_hp
06-09-2009, 12:51
27) در شکل مقابل مثلث ABC قائم الزاویه است (B=90o) و AB=10-π و BC=6. نیم استوانه ای با شعاع واحد و محور عمود بر AB، بین نقاط A وC مانع شده است.

[ برای مشاهده لینک ، لطفا با نام کاربری خود وارد شوید یا ثبت نام کنید ]
مورچه بنا به دلایلی (!) باید هر چه سریع تر از نقطه ی A به لانه اش در نقطه ی C برود. طول کوتاه ترین مسیر ممکن چقدر است؟
(مساله مرحله اول بیست یکمین المپیاد ریاضی ایران،برای دیدن کلیه مساِیل به
[ برای مشاهده لینک ، لطفا با نام کاربری خود وارد شوید یا ثبت نام کنید ]

بروید.)

سلام

با تشکر ازCppBuilder2006 عزیز،جوابشونو بررسی می کنیم:

فک کنم جواب بشه رادیکال 136 یعنی الف
البته منظور از AB=10-pio رو بدون احتساب تکۀ زیر استوانه می گیریم.
با تشکر،راه حل شما درسته،ولی منظور سوال با احتساب تیکه زیر استوانه است.

با تشکر از saber57 عزیز،جوابشونو بررسی می کنیم:

کوتاهترین مسیر همون AC هست پس کافیه مقدار وتر بدست بیاد . محیط نیم دایره برابر pi هست . با توجه به شکل مشخصه که اختلاف طول وترAC و ضلع مقابلش AB برابر محیط نیم استوانه یا pi هست !!بنابراین:

AC-AB=pi ---->AC=AB+pi----->AC=10-pi+pi----->AC=10


بنابراین گزینه ج صحیح میباشد

راه حلتون درست نیست،چرا کوتاهترین مسیر AC میشه؟
اختلاف طول وترAC و ضلع مقابلش AB برابر محیط نیم استوانه یا pi نیست.

حل:
صفحه را مسطح کنید!به وضوح کوناهترین مسیر مسیری است که پس از مسطح کردن صفحه A را به C وصل می کند.بعد از مسطح کردن صفحه طول AB برابر 8 می شود.(چرا؟)پس طول مسیر AC برابر 10 می شود.(طبق قضیه فیثاغورث!)
دقت کنید که کوتاهترین مسیر AC نیست!بلکه مسیری است که پس از مسطح کردن صفحه A را به C وصل می کند،که لزوما منطبق بر AC قبل از مسطح کردن صفحه نیست!

ali_hp
06-09-2009, 13:02
قضیه هرون:
در مثلث دلخواه ABC فرض کنید BC=a , AB=c , AB=c و S مساحت مثلث باشدو P برابر نصف محیط مثلث باشند ثابت کنید:


[ برای مشاهده لینک ، لطفا با نام کاربری خود وارد شوید یا ثبت نام کنید ]{p(p-a)(p-b)(p-c)}

Parser
07-09-2009, 16:55
به نام معشوق ازلي
حل مسئله‌ي یکشنبه هفتم

قضیه هرون:
در مثلث دلخواه ABC فرض کنید BC=a , AB=c , AB=c و S مساحت مثلث باشدو P برابر نصف محیط مثلث باشند ثابت کنید:


[ برای مشاهده لینک ، لطفا با نام کاربری خود وارد شوید یا ثبت نام کنید ]{p(p-a)(p-b)(p-c)}


[ برای مشاهده لینک ، لطفا با نام کاربری خود وارد شوید یا ثبت نام کنید ]{1}{2}b.c.%5Csin%20A=%5Cfrac{1} {2}b.c.%5Csqrt{1-%5Ccos^2%20A}
[ برای مشاهده لینک ، لطفا با نام کاربری خود وارد شوید یا ثبت نام کنید ]^2=b^2+c^2+2b.c.%5Ccos%20A%20%5CRightar row%20%5Ccos%20A=%5Cfrac{b^2+c^2-a^2}{2b.c}

[ برای مشاهده لینک ، لطفا با نام کاربری خود وارد شوید یا ثبت نام کنید ]{1}{2}b.c%5Csqr t{1-%5Cleft%20(%5Cfrac{b^2+c^2-a^2}{2b.c}%5Cright%20)^2}

[ برای مشاهده لینک ، لطفا با نام کاربری خود وارد شوید یا ثبت نام کنید ]{1}{2}b.c%5Csqr t{%5Cfrac{4b^2c^2-(b^2+c^2-a^2)^2}{4b^2c^2}}=%5Cfrac{1}{4}{}%5Csqrt{2a^2b^2+2 b^2c^2+2a^2c^2-a^4-b^4-c^4}

[ برای مشاهده لینک ، لطفا با نام کاربری خود وارد شوید یا ثبت نام کنید ]{2a^2b^2+2b^2c^2+2a^2c^2-a^4-b^4-c^4}{(a+b-c)(b+c-a)(a+c-b)}=a+b+c

[ برای مشاهده لینک ، لطفا با نام کاربری خود وارد شوید یا ثبت نام کنید ]{%5Cfrac{(a+b+c )(b+c-a)(a+c-b)(a+b-c)}{16}}

[ برای مشاهده لینک ، لطفا با نام کاربری خود وارد شوید یا ثبت نام کنید ]{%5Cfrac{(a+b+c )}{2}(%5Cfrac{a+b+c}{2}-a)(%5Cfrac{a+b+c}{2}-b)(%5Cfrac{a+b+c}{2}-c)}

[ برای مشاهده لینک ، لطفا با نام کاربری خود وارد شوید یا ثبت نام کنید ]{p(p-a)(p-b)(p-c)}

eh_mn
09-09-2009, 13:47
يكي ديگر از نامساوي‌هاي مهم، نامساوي كوشي-شوارتز است:
اگر a1 و a2 و ... و an و b1 و b2 و ... و bn اعدادي نامنفي باشند آنگاه


[ برای مشاهده لینک ، لطفا با نام کاربری خود وارد شوید یا ثبت نام کنید ](a_1b_1&plus;a_2b_2&plus;\dots&plus;a_nb_n)^2\leq&space;(a_1^ 2&plus;a_2^2&plus;\dots&plus;a_n^2)(b_1^2&plus;b_2^2&plus;\dots&plus;b_n^2)

با استفاده از نامساوي كوشي-شوارتز مسأله‌ي زير را حل كنيد.

فرض كنيد [ برای مشاهده لینک ، لطفا با نام کاربری خود وارد شوید یا ثبت نام کنید ](x) يك چندجمله‌اي با ضرايب مثبت باشد. ثابت كنيد كه اگر رابطه‌ي


[ برای مشاهده لینک ، لطفا با نام کاربری خود وارد شوید یا ثبت نام کنید ](\frac{1}{x}&space;\right&space;)\geq&space;\frac{1} {P(x)}
براي x=1 برقرار باشد آنگاه براي هر x>0 برقرار است.



ــــــــــــــــــ
11 / 06 / 88

راه حل Parser هر چند بدون شرحه ولي كاملا درسته. اين راه حل رو در

برای مشاهده محتوا ، لطفا وارد شوید یا ثبت نام کنید
ملاحظه كنيد. با تشكر فراوان از ايشان.






ــــــــــــــــــــــ
18 / 06 / 88

eh_mn
09-09-2009, 13:51
براي هر چهار عدد مثبت مانند a و b و c و d ثابت كنيد


[ برای مشاهده لینک ، لطفا با نام کاربری خود وارد شوید یا ثبت نام کنید ]{1}{a}+\frac{1}{b}+\frac{4}{c}+\fra c{16}{d}\geq \frac{64}{a+b+c+d}


ـــــــــــــــــــــــ
18 / 06 / 88

mofidy1
10-09-2009, 11:33
با سلام

سعی کنید شکل زیر را با حرکت پیوسته ی قلم، بدون این که از یک مسیر، دوبار عبور نمایید، رسم کنید:



[ برای مشاهده لینک ، لطفا با نام کاربری خود وارد شوید یا ثبت نام کنید ]


موفق باشید.

5 شهریور 1388

با سلام

همان طور که CppBuilder2006 در پست 141 ([ برای مشاهده لینک ، لطفا با نام کاربری خود وارد شوید یا ثبت نام کنید ])و maziar92142 در پست 142 ([ برای مشاهده لینک ، لطفا با نام کاربری خود وارد شوید یا ثبت نام کنید ]) توضیح دادند، چنین کاری ممکن نیست زیرا چهار رأس داریم که درجه ی آن ها فرد است. با تشکر از این دو بزرگوار.

آموزش حل مساله:

اگر کمی دقت کنید، این مساله بسیار شبیه «مساله ی پل های کونیگسبرگ» است و راه حلی مشابه با آن دارد؛ بنابراین حل هر مساله ایده های جدیدی به دست می دهد که این ایده ها ممکن است برای حل مسائل دیگر به کار رود. شاید بتوان گفت که این روش یکی از مهم ترین روش های حل مساله است. پس «برای حل مساله، مساله حل کنید!!»

موفق باشید.

19 شهریور 1388

mofidy1
10-09-2009, 12:00
با سلام

با استفاده از محاسبات مربوط به اعداد مختلط (یا روش دیگر) ثابت کنید



[ برای مشاهده لینک ، لطفا با نام کاربری خود وارد شوید یا ثبت نام کنید ](72^\circ&space;)=cos(\frac{2\pi}{5})=\frac {\sqrt{5}-1}{4}.

موفق باشید.

18 شهریور 1388

Parser
11-09-2009, 16:44
به نام معشوق ازلي
حل مسئله‌ي شنبه ی هفتم

[ برای مشاهده لینک ، لطفا با نام کاربری خود وارد شوید یا ثبت نام کنید ](72^{%5Ccirc})=%5Ccos%20(90^{%5 Ccirc}-18^{%5Ccirc})=%5Csin%20(18^{%5Ccirc})

[ برای مشاهده لینک ، لطفا با نام کاربری خود وارد شوید یا ثبت نام کنید ](2%5Ctheta%20)=1-2%5Csin%20^2(%5Ctheta%20)%20%5C,%5C,%5C,%5C,,%5C,% 5C,%5C,%5C,%20%5Csin%20(2%5Ctheta%20)=2%5Csin%20(% 5Ctheta%20)%5Ccos%20(%5Ctheta%20)

[ برای مشاهده لینک ، لطفا با نام کاربری خود وارد شوید یا ثبت نام کنید ](72^{%5Ccirc})=1-2%5Csin%20^2(36^{%5Ccirc})=1-2(2%5Csin%20(18^{%5Ccirc})%5Ccos%20(18^{%5Ccirc})) ^2

[ برای مشاهده لینک ، لطفا با نام کاربری خود وارد شوید یا ثبت نام کنید ]{%5Ccolor{red}%5Csin%20( 18^{%5Ccirc})=1-8%5Csin%20^2(18^{%5Ccirc})+8%5Csin%20^4(18^{%5Ccir c})}

[ برای مشاهده لینک ، لطفا با نام کاربری خود وارد شوید یا ثبت نام کنید ]^4(18^{%5Ccirc })-8%5Csin%20^2(18^{%5Ccirc})-%5Csin%20(18^{%5Ccirc})+1=0

[ برای مشاهده لینک ، لطفا با نام کاربری خود وارد شوید یا ثبت نام کنید ](%5Csin%20(18^{%5Ccirc})-1)(%5Csin%20(18^{%5Ccirc})+%5Cfrac{1}{2})(8%5Csin% 20^2(18^{%5Ccirc})+4%5Csin%20(18^{%5Ccirc})-2)=0

[ برای مشاهده لینک ، لطفا با نام کاربری خود وارد شوید یا ثبت نام کنید ](18^{%5Ccirc})%5Cneq%201%20%5C, %5C,%5C,%5C,,%5C,%5C,%5C,%5C,%20%5Csin%20(18^{%5Cc irc})%5Cneq%20-%5Cfrac{1}{2}

[ برای مشاهده لینک ، لطفا با نام کاربری خود وارد شوید یا ثبت نام کنید ]^2(18^{%5Ccirc })+2%5Csin%20(18^{%5Ccirc})-1=0

[ برای مشاهده لینک ، لطفا با نام کاربری خود وارد شوید یا ثبت نام کنید ](18^{%5Ccirc})%3E%200%20%5C,%5C ,%5C,%5C,%20%5CRightarrow%20%5C,%5C,%5C,%5C,%5Csin %20(18^{%5Ccirc})=%5Cfrac{%5Csqrt{5}-1}{4}

[ برای مشاهده لینک ، لطفا با نام کاربری خود وارد شوید یا ثبت نام کنید ](18^{%5Ccirc})=%5Ccos%20(90^{%5 Ccirc}-18^{%5Ccirc})=%5Ccos%20(72^{%5Ccirc})

[ برای مشاهده لینک ، لطفا با نام کاربری خود وارد شوید یا ثبت نام کنید ](72^{%5Ccirc})= %5Cfrac{%5Csqrt{5}-1}{4}

saber57
11-09-2009, 19:29
به نام معشوق ازلي
حل مسئله‌ي شنبه ی هفتم

[ برای مشاهده لینک ، لطفا با نام کاربری خود وارد شوید یا ثبت نام کنید ](72^{%5Ccirc})=%5Ccos%20(90^{%5 Ccirc}-18^{%5Ccirc})=%5Csin%20(18^{%5Ccirc})


[ برای مشاهده لینک ، لطفا با نام کاربری خود وارد شوید یا ثبت نام کنید ](2%5Ctheta%20)=1-2%5Csin%20^2(%5Ctheta%20)%20%5C,%5C,%5C,%5C,,%5C,% 5C,%5C,%5C,%20%5Csin%20(2%5Ctheta%20)=2%5Csin%20(% 5Ctheta%20)%5Ccos%20(%5Ctheta%20)


[ برای مشاهده لینک ، لطفا با نام کاربری خود وارد شوید یا ثبت نام کنید ](72^{%5Ccirc})=1-2%5Csin%20^2(36^{%5Ccirc})=1-2(2%5Csin%20(18^{%5Ccirc})%5Ccos%20(18^{%5Ccirc})) ^2


[ برای مشاهده لینک ، لطفا با نام کاربری خود وارد شوید یا ثبت نام کنید ]{%5Ccolor{red}%5Csin%20( 18^{%5Ccirc})=1-8%5Csin%20^2(18^{%5Ccirc})+8%5Csin%20^4(18^{%5Ccir c})}


[ برای مشاهده لینک ، لطفا با نام کاربری خود وارد شوید یا ثبت نام کنید ]^4(18^{%5Ccirc })-8%5Csin%20^2(18^{%5Ccirc})-%5Csin%20(18^{%5Ccirc})+1=0


[ برای مشاهده لینک ، لطفا با نام کاربری خود وارد شوید یا ثبت نام کنید ](%5Csin%20(18^{%5Ccirc})-1)(%5Csin%20(18^{%5Ccirc})+%5Cfrac{1}{2})(8%5Csin% 20^2(18^{%5Ccirc})+4%5Csin%20(18^{%5Ccirc})-2)=0


[ برای مشاهده لینک ، لطفا با نام کاربری خود وارد شوید یا ثبت نام کنید ](18^{%5Ccirc})%5Cneq%201%20%5C, %5C,%5C,%5C,,%5C,%5C,%5C,%5C,%20%5Csin%20(18^{%5Cc irc})%5Cneq%20-%5Cfrac{1}{2}


[ برای مشاهده لینک ، لطفا با نام کاربری خود وارد شوید یا ثبت نام کنید ]^2(18^{%5Ccirc })+2%5Csin%20(18^{%5Ccirc})-1=0


[ برای مشاهده لینک ، لطفا با نام کاربری خود وارد شوید یا ثبت نام کنید ](18^{%5Ccirc})%3E%200%20%5C,%5C ,%5C,%5C,%20%5CRightarrow%20%5C,%5C,%5C,%5C,%5Csin %20(18^{%5Ccirc})=%5Cfrac{%5Csqrt{5}-1}{4}


[ برای مشاهده لینک ، لطفا با نام کاربری خود وارد شوید یا ثبت نام کنید ](18^{%5Ccirc})=%5Ccos%20(90^{%5 Ccirc}-18^{%5Ccirc})=%5Ccos%20(72^{%5Ccirc})



[ برای مشاهده لینک ، لطفا با نام کاربری خود وارد شوید یا ثبت نام کنید ](72^{%5Ccirc})= %5Cfrac{%5Csqrt{5}-1}{4}


محاسباتتون درست هست .. با کمی دقت و فرض [ برای مشاهده لینک ، لطفا با نام کاربری خود وارد شوید یا ثبت نام کنید ](x)=t به یک معادله درجه سوم [ برای مشاهده لینک ، لطفا با نام کاربری خود وارد شوید یا ثبت نام کنید ]^3&plus;8t^2-1=0 میرسیم.بعد راه حل مختلط چطور حل میشه؟
ویرایش 20:39

mir@
12-09-2009, 06:31
حل مسئله شنبه هفتم


.
لطفا حاصل مجموع زیر را بیابید:


[ برای مشاهده لینک ، لطفا با نام کاربری خود وارد شوید یا ثبت نام کنید ]{n=1}^\infty&space;\tan^{-1}\frac{2}{n^2}


راهنمایی: از اتحاد زیر استفاده کنید:


[ برای مشاهده لینک ، لطفا با نام کاربری خود وارد شوید یا ثبت نام کنید ]^{-1}\frac{1}{n-1}-\tan^{-1}\frac{1}{n&plus;1}=\tan^{-1}\frac{2}{n^2}


توجه: [ برای مشاهده لینک ، لطفا با نام کاربری خود وارد شوید یا ثبت نام کنید ]^{-1}(x)\leftrightarrow&space;\arctan(x)



در این مساله هم از قاعده تلسکوپی استفاده میکنیم فقط یک نکته جالب در حل مساله نهفته هست و اونم یک عبارت اضافه (بجز عبارات اول و آخر)هست .طبق اتحادی که خودتون فرمودید و براحتی با استفاده از اتحاد تانژانت تفاضل قابل اثبات هست و داریم:

[ برای مشاهده لینک ، لطفا با نام کاربری خود وارد شوید یا ثبت نام کنید ]{n=1}^{\infty&space;}tan^{-1}\left&space;(&space;\frac{1}{n-1}&space;\right&space;)-&space;tan^{-1}\left&space;(&space;\frac{1}{n&plus;1}&space;\right&space;)=tan^{-1}\infty&space;-tan^{-1}\frac{1}{2}&plus;tan^{-1}1-tan^{-1}\frac{1}{3}&plus;tan^{-1}\frac{1}{2}-tan^{-1}\frac{1}{4}&plus;....-tan^{-1}\frac{1}{\infty&space;}=tan^{-1}\infty&space;&plus;tan^{-1}1-tan^{-1}0=\frac{\pi&space;}{2}&plus;\frac{\pi&space;}{4}=\frac{3\pi&space;}{4}

فقط عبارت [ برای مشاهده لینک ، لطفا با نام کاربری خود وارد شوید یا ثبت نام کنید ]^{-1}1 وجود دارد که قرینه اش در کل مجموع وجود ندارد ولی تفاضل عبارات اول و آخر را داریم


البته به فرم دیگر :


[ برای مشاهده لینک ، لطفا با نام کاربری خود وارد شوید یا ثبت نام کنید ]{n=1}^{\infty&space;}tan^{-1}\left&space;(&space;\frac{2}{n^2}&space;\right&space;)=\sum_{n=1}^{\inft y&space;}\left&space;(tan^{-1}\left&space;(&space;\frac{1}{n-1}&space;\right&space;)-&space;tan^{-1}\left&space;(&space;\frac{1}{n&plus;1}&space;\right&space;)&space;\right&space;)


[ برای مشاهده لینک ، لطفا با نام کاربری خود وارد شوید یا ثبت نام کنید ]{n=1}^{\infty&space;}\left&space;(tan^{-1}\left&space;(&space;\frac{1}{n-1}&space;\right&space;)-&space;tan^{-1}\left&space;(&space;\frac{1}{n}&space;\right&space;)&space;\right&space;)&plus;\sum_{n=1} ^{\infty&space;}\left&space;(tan^{-1}\left&space;(&space;\frac{1}{n}&space;\right&space;)-&space;tan^{-1}\left&space;(&space;\frac{1}{n&plus;1}&space;\right&space;)&space;\right&space;)


[ برای مشاهده لینک ، لطفا با نام کاربری خود وارد شوید یا ثبت نام کنید ](tan^{-1}\infty&space;-tan^{-1}0&space;\right&space;)&plus;\left&space;(&space;tan^{-1}1-tan^{-1}0&space;\right&space;)=\frac{\pi&space;}{2}&plus;\frac{\pi&space;}{4}=\frac{3 \pi&space;}{4}




برای مشاهده محتوا ، لطفا وارد شوید یا ثبت نام کنید


حضرت استاد saber57 به درستی قفل مسئله را گشوده اند. خدایشان جزای خیر دهاد.

mir@
12-09-2009, 06:53
.
تمام توابع f ِهمه جا مشتق پذیر را بیابید که در رابطه زیر صدق کنند:


[ برای مشاهده لینک ، لطفا با نام کاربری خود وارد شوید یا ثبت نام کنید ](x)+f(y)=f\left(\frac{x+y}{1-xy}\right)


برای تمام x و y هایی که [ برای مشاهده لینک ، لطفا با نام کاربری خود وارد شوید یا ثبت نام کنید ]

راهنمایی: نسبت به x و y مشتق گرفته و ساده کنید.

CppBuilder2006
12-09-2009, 20:56
خدایشان جزای خیر دهاد.
آممین
...................

ali_hp
14-09-2009, 19:33
قضیه هرون:
در مثلث دلخواه ABC فرض کنید BC=a , AB=c , AB=c و S مساحت مثلث باشدو P برابر نصف محیط مثلث باشند ثابت کنید:

دوست عزیزمون parser در پست 159 مساله را به درستی حل کردند،راه حل من هم تقریبا همونه.

ali_hp
14-09-2009, 19:44
شازده کوچولو، روی سیاره کوچکی زندگی می کند که شعاعش 60 متر است. روزی با شروع از روی خط استوا، 30pi متر به شرق ، 20pi متر به شمال، 30pi متر به غرب و سرانجام 20pi متر به جنوب می رود. شازده کوچولو تا مکان اولش چند متر فاصله دارد؟

zahedy2006
15-09-2009, 04:20
شازده کوچولو، روی سیاره کوچکی زندگی می کند که شعاعش 60 متر است. روزی با شروع از روی خط استوا، 30pi متر به شرق ، 20pi متر به شمال، 30pi متر به غرب و سرانجام 20pi متر به جنوب می رود. شازده کوچولو تا مکان اولش چند متر فاصله دارد؟


فاصله روي زمين 30 pi
فاصله مستقيم 60*(1.44) - 60 ضربدر راديكال 2

Parser
15-09-2009, 13:57
به نام معــشوق ازلی


براي هر چهار عدد مثبت مانند a و b و c و d ثابت كنيد


[ برای مشاهده لینک ، لطفا با نام کاربری خود وارد شوید یا ثبت نام کنید ]{1}{a}+\frac{1}{b}+\frac{4}{c}+\fra c{16}{d}\geq \frac{64}{a+b+c+d}


ـــــــــــــــــــــــ
18 / 06 / 88

میانگین دو عدد حقیقی مثبت، بزرگتر یا مساوی است از واسطه ی هندسی آن دو، به عبارتی

[ برای مشاهده لینک ، لطفا با نام کاربری خود وارد شوید یا ثبت نام کنید ]>0 \,,\, B>0 \Rightarrow A+B\geq 2\sqrt{AB}

پس می توان اظهار داشت

[ برای مشاهده لینک ، لطفا با نام کاربری خود وارد شوید یا ثبت نام کنید ]{a}{b}> 0\,,\,\frac{b}{a}> 0\Rightarrow \frac{a}{b}+\frac{b}{a}\geq 2\sqrt{\frac{a}{b}\frac{b}{a}}\Rightarrow \frac{a}{b}+\frac{b}{a}\geq 2

به همین ترتیب
برای چهار عدد مثبت a و b و c و d

[ برای مشاهده لینک ، لطفا با نام کاربری خود وارد شوید یا ثبت نام کنید ]{c}{a}+\frac{4a}{c}\geq 2\sqrt{\frac{c}{a}\frac{4a}{c}}\Rightarrow \frac{c}{a}+\frac{4a}{c}\geq 4

[ برای مشاهده لینک ، لطفا با نام کاربری خود وارد شوید یا ثبت نام کنید ]{d}{a}+\frac{16a}{d}\geq 8 \,\,,\,\, \frac{c}{b}+\frac{4b}{c}\geq 4 \,\,,\,\, \frac{d}{b}+\frac{16b}{d}\geq 8 \,\,,\,\, \frac{4d}{c}+\frac{16c}{d}\geq 16

با جمع شش نامساوی فوق می توان نوشت

[ برای مشاهده لینک ، لطفا با نام کاربری خود وارد شوید یا ثبت نام کنید ]{b}{a}+\frac{c}{a}+\frac{d}{a}+\fra c{a}{b}+\frac{c}{b}+\frac{d}{b}+\frac{4a}{c}+\frac {4b}{c}+\frac{4d}{c}+\frac{16a}{d}+\frac{16b}{d}+\ frac{16c}{d}\geq 42

اگر به طرفین، مقادیر [ برای مشاهده لینک ، لطفا با نام کاربری خود وارد شوید یا ثبت نام کنید ]{a}{a} و [ برای مشاهده لینک ، لطفا با نام کاربری خود وارد شوید یا ثبت نام کنید ]{b}{b} و [ برای مشاهده لینک ، لطفا با نام کاربری خود وارد شوید یا ثبت نام کنید ]{4c}{c} و [ برای مشاهده لینک ، لطفا با نام کاربری خود وارد شوید یا ثبت نام کنید ]{16d}{d}را اضافه گردند، عبارت زیر بدست می آید


[ برای مشاهده لینک ، لطفا با نام کاربری خود وارد شوید یا ثبت نام کنید ] \frac{1}{a}(a+b+c+d)+\frac{1}{b}(a+b+c+d)+\frac{4} {c}(a+b+c+d)+\frac{16}{d}(a+b+c+d)\geq 64

[ برای مشاهده لینک ، لطفا با نام کاربری خود وارد شوید یا ثبت نام کنید ] \left ( \frac{1}{a}+\frac{1}{b}+\frac{4}{c}+\frac{16}{d} \right )(a+b+c+d)\geq 64

[ برای مشاهده لینک ، لطفا با نام کاربری خود وارد شوید یا ثبت نام کنید ]> 0 \Rightarrow \frac{1}{a}+\frac{1}{b}+\frac{4}{c}+\frac{16}{d}\g eq \frac{64}{a+b+c+d}


از عبارت فوق می توان به دو عبارت قبل از آن، که بینشان علامت [ برای مشاهده لینک ، لطفا با نام کاربری خود وارد شوید یا ثبت نام کنید ] است، بازگشت. درستی هریک درستی دیگری را نیز نتیجه می دهد.

eh_mn
16-09-2009, 22:33
براي هر چهار عدد مثبت مانند a و b و c و d ثابت كنيد


[ برای مشاهده لینک ، لطفا با نام کاربری خود وارد شوید یا ثبت نام کنید ]{1}{a}+\frac{1}{b}+\frac{4}{c}+\fra c{16}{d}\geq \frac{64}{a+b+c+d}


ـــــــــــــــــــــــ
18 / 06 / 88

با تشكر از Parser برای حل این مسأله. برای مشاهده‌ی راه حل ایشان به

برای مشاهده محتوا ، لطفا وارد شوید یا ثبت نام کنید
مراجعه كنید.

راه حل دیگری با استفاده از نامساوی كشی-شوارتز كه در

برای مشاهده محتوا ، لطفا وارد شوید یا ثبت نام کنید
معرفی شده است، وچود دارد.


[ برای مشاهده لینک ، لطفا با نام کاربری خود وارد شوید یا ثبت نام کنید ](a+b+c+d)(\frac{1}{a}+\frac{1}{b}+\fra c{4}{c}+\frac{16}{d}) \geq ( 1 + 1 + 2 + 4 )^2 = 64

با توجه به مثبت بودن a+b+c+d حكم به راحتی نتیجه می‌شود.



ـــــــــــــــــــــ
25 / 06 / 88

eh_mn
16-09-2009, 22:44
الف. نشان دهيد براي هر سه عدد مثبت x و y و z داريم


[ برای مشاهده لینک ، لطفا با نام کاربری خود وارد شوید یا ثبت نام کنید ](x+y+z)^2\geq 3(xy+yz+zx)




ـــــــــــــــــــــــ
25 / 06 / 88

mofidy1
17-09-2009, 11:58
با سلام

با استفاده از محاسبات مربوط به اعداد مختلط (یا روش دیگر) ثابت کنید



[ برای مشاهده لینک ، لطفا با نام کاربری خود وارد شوید یا ثبت نام کنید ](72^\circ&space;)=cos(\frac{2\pi}{5})=\frac {\sqrt{5}-1}{4}.

موفق باشید.

18 شهریور 1388

با سلام و آرزوی قبولی طاعات

برای دیدن راه حلی مقدماتی از این مساله به پست 164 ([ برای مشاهده لینک ، لطفا با نام کاربری خود وارد شوید یا ثبت نام کنید ])که Parser ارسال کردند، مراجعه کنید. از ایشان تشکر می کنم. هدف بنده استفاده از صفحه ی مختلط برای حل این مساله بود که با کاربردهای بی نظیر و زیبای این صفحه آشنایی بیشتری پیدا کنیم.

قرار دهید


[ برای مشاهده لینک ، لطفا با نام کاربری خود وارد شوید یا ثبت نام کنید ](\frac{2\pi}{5})+i\,sin(\frac{2\p i}{5})

بنابر این z یک ریشه ی پنجم واحد است و لذا


[ برای مشاهده لینک ، لطفا با نام کاربری خود وارد شوید یا ثبت نام کنید ]^4+z^3+z^2+z+1=0

در نتیجه


[ برای مشاهده لینک ، لطفا با نام کاربری خود وارد شوید یا ثبت نام کنید ](z+\frac{1}{z} \right)^2+\left(z+\frac{1}{z} \right)-1=0

اما به راحتی می توان دید که


[ برای مشاهده لینک ، لطفا با نام کاربری خود وارد شوید یا ثبت نام کنید ]{1}{z}=2cos(\frac{2\pi}{5})

حال با جای گذاری و حل معادله ی درجه ی 2 به دست آمده بر حسب کسینوس، مساله حل خواهد شد.

نکته:

بد نیست بدانید که این مساله نشان می دهد که یک پنج ضلعی منتظم را می توان به وسیله ی خط کش و پرگار رسم کرد. این مباحث از جمله مباحث بسیار زیبای جبر و جزیی از «نظریه گالوا» است. آیا می توانید مساله ی مشکل زیر را هم حل کنید؟

ساختن یک هفت ضلعی منتظم به وسیله ی خط کش و پرگار امکان پذیر نیست!!

آموزش حل مساله:

مساله ی بالا، امکان اتصال ایده های به ظاهر بی ربط به یکدیگر را برای حل یک مساله، نشان می دهد. بنابر این هر چه اطلاعات شما در زمینه های مختلف ریاضی، بیشتر باشد امکان حل مساله (یا حداقل حل ساده تر مساله) بیشتر است.

موفق باشید.

26 شهریور 1388

mofidy1
17-09-2009, 14:35
انتگرال زیر را محاسبه کنید:


[ برای مشاهده لینک ، لطفا با نام کاربری خود وارد شوید یا ثبت نام کنید ]{0}^{\infty}e^{-t^2}\,dt

راهنمایی: از تابع گاما استفاده کنید.

موفق باشید.

26 شهریور 1388

amintnt
17-09-2009, 14:51
خواهشی داشتم از عزیزانی که لطف میکنن و سوال ها رو طرح میکنن...

لطفا مثل جناب مفیدی ذکر کنید که سوال در چه سطحی هست...

ممنونم

Parser
18-09-2009, 11:56
به نام معــشوق ازلی


انتگرال زیر را محاسبه کنید:


[ برای مشاهده لینک ، لطفا با نام کاربری خود وارد شوید یا ثبت نام کنید ]{0}^{%5Cinfty%20}e^{-u^2}du



[ برای مشاهده لینک ، لطفا با نام کاربری خود وارد شوید یا ثبت نام کنید ](x)=%5Cint_{0}^{%5Cinfty%20}t ^{x-1}e^{-t}dt

با توجه به حدود انتگرال٬ t هاي بزرگتر يا برابر با صفر توسط مقاديري از u طبق رابطه ي [ برای مشاهده لینک ، لطفا با نام کاربری خود وارد شوید یا ثبت نام کنید ]^2 توليد مي گردند.

تابع [ برای مشاهده لینک ، لطفا با نام کاربری خود وارد شوید یا ثبت نام کنید ]^2 به ازاي u هاي مثبت٬ اعداد مثبت را پوشش مي دهد.

همچنين اگر [ برای مشاهده لینک ، لطفا با نام کاربری خود وارد شوید یا ثبت نام کنید ]٬ [ برای مشاهده لینک ، لطفا با نام کاربری خود وارد شوید یا ثبت نام کنید ]^2 يك به يك است.

با در نظر گرفتن شرايط لازم براي تعويض متغيّر٬[ برای مشاهده لینک ، لطفا با نام کاربری خود وارد شوید یا ثبت نام کنید ]^2 را با t مي توان جايگزين نمود و حدود جديد انتگرال را بدست آورد.

[ برای مشاهده لینک ، لطفا با نام کاربری خود وارد شوید یا ثبت نام کنید ]

ملاحظه بفرماييد كه كران بالاي انتگرال٬ پس از تعويض متغيّر [ برای مشاهده لینک ، لطفا با نام کاربری خود وارد شوید یا ثبت نام کنید ]است.

[ برای مشاهده لینک ، لطفا با نام کاربری خود وارد شوید یا ثبت نام کنید ]{0}^{%5Cinfty%20}t^{x-1}e^{-t}dt=%5Cint_{0}^{%5Cinfty%20}(u^2)^{x-1}e^{-u^2}(2u)du=2%5Cint_{0}^{%5Cinfty%20}u^{2x-1}e^{-u^2}du

[ برای مشاهده لینک ، لطفا با نام کاربری خود وارد شوید یا ثبت نام کنید ](x)=2%5Cint_{ 0}^{%5Cinfty%20}u^{2x-1}e^{-u^2}du

ثابت مي شود كه [ برای مشاهده لینک ، لطفا با نام کاربری خود وارد شوید یا ثبت نام کنید ](%5Cfrac{1}{2})=%5Csqrt{%5Cpi %20}

[ برای مشاهده لینک ، لطفا با نام کاربری خود وارد شوید یا ثبت نام کنید ](%5Cfrac{1}{2 })=2%5Cint_{0}^{%5Cinfty%20}e^{-u^2}du

[ برای مشاهده لینک ، لطفا با نام کاربری خود وارد شوید یا ثبت نام کنید ]{0}^{%5Cinfty%20} e^{-u^2}du=%5Cfrac{%5Csqrt{%5Cpi%20}}{2}

mir@
19-09-2009, 01:23
پاسخ مسئله شنبه هشتم


.
تمام توابع f ِهمه جا مشتق پذیر را بیابید که در رابطه زیر صدق کنند:


[ برای مشاهده لینک ، لطفا با نام کاربری خود وارد شوید یا ثبت نام کنید ] +y%7D%7B1-xy%7D%5Cright%29


برای تمام x و y هایی که [ برای مشاهده لینک ، لطفا با نام کاربری خود وارد شوید یا ثبت نام کنید ]

راهنمایی: نسبت به x و y مشتق گرفته و ساده کنید.

با گرفتن مشتق جزئی نسبت به دو متغیر داریم:


[ برای مشاهده لینک ، لطفا با نام کاربری خود وارد شوید یا ثبت نام کنید ]



[ برای مشاهده لینک ، لطفا با نام کاربری خود وارد شوید یا ثبت نام کنید ]



با ساده کردن این عبارات، مثلا تقسیم یکی بر دیگری خواهیم داشت:


[ برای مشاهده لینک ، لطفا با نام کاربری خود وارد شوید یا ثبت نام کنید ] 27%28x%29



از آنجا که یک طرف کاملا وابسته به y و یکطرف کاملا وابسته به x است، بنابراین هریک از طرفین باید مساوی یک مقدار ثابت باشد.

[ برای مشاهده لینک ، لطفا با نام کاربری خود وارد شوید یا ثبت نام کنید ]


بنابراین تابع در حالت کلی به صورت زیر خواهد بود:


[ برای مشاهده لینک ، لطفا با نام کاربری خود وارد شوید یا ثبت نام کنید ]


اما اگر در رابطه اصلی قرار دهیم : [ برای مشاهده لینک ، لطفا با نام کاربری خود وارد شوید یا ثبت نام کنید ]

خواهیم داشت:


[ برای مشاهده لینک ، لطفا با نام کاربری خود وارد شوید یا ثبت نام کنید ]


بنابراین [ برای مشاهده لینک ، لطفا با نام کاربری خود وارد شوید یا ثبت نام کنید ]

و نتیجتاً تابع برابر خواهد بود با:


[ برای مشاهده لینک ، لطفا با نام کاربری خود وارد شوید یا ثبت نام کنید ]


خدا به من جزای خیر بدهد!

mir@
19-09-2009, 01:33
برای [ برای مشاهده لینک ، لطفا با نام کاربری خود وارد شوید یا ثبت نام کنید ] حاصل مجموع بی نهایت زیر را بیابید:


[ برای مشاهده لینک ، لطفا با نام کاربری خود وارد شوید یا ثبت نام کنید ] ac%7Bx%5E2%7D%7B%28x+1%29%28x%5E2+1%29%7D+%5Cfrac% 7Bx%5E4%7D%7B%28x+1%29%28x%5E2+1%29%28x%5E4+1%29%7 D+%5Cldots

راهنمایی: در عبارت [ برای مشاهده لینک ، لطفا با نام کاربری خود وارد شوید یا ثبت نام کنید ] ضرب کنید طرفین را و بعد ساده کنید. بعدش به کسرهای جزئی تجزیه کنید، بعد بعدش ...

Parser
19-09-2009, 15:09
به نام معــشوق ازلی

پاسخ مسئله ی چهارشنبه‌ی نهم




[ برای مشاهده لینک ، لطفا با نام کاربری خود وارد شوید یا ثبت نام کنید ](x+y+z)^2\geq 3(xy+yz+zx)


بديهي است كه براي سه عبارت حقيقي [ برای مشاهده لینک ، لطفا با نام کاربری خود وارد شوید یا ثبت نام کنید ](x-y)^2 و [ برای مشاهده لینک ، لطفا با نام کاربری خود وارد شوید یا ثبت نام کنید ](y-z)^2 و [ برای مشاهده لینک ، لطفا با نام کاربری خود وارد شوید یا ثبت نام کنید ](z-x)^2 مي توان نوشت


[ برای مشاهده لینک ، لطفا با نام کاربری خود وارد شوید یا ثبت نام کنید ](x-y)^2+(y-z)^2+(z-x)^2%5Cgeq%200

[ برای مشاهده لینک ، لطفا با نام کاربری خود وارد شوید یا ثبت نام کنید ](x^2-2xy+y^2)+(y^2-2yz+z^2)+(z^2-2zx+x^2)%5Cgeq%200

[ برای مشاهده لینک ، لطفا با نام کاربری خود وارد شوید یا ثبت نام کنید ]^2+2y^2+2z^2-2(xy+yz+zx)%5Cgeq%200

[ برای مشاهده لینک ، لطفا با نام کاربری خود وارد شوید یا ثبت نام کنید ]^2+y^2+z^2-(xy+yz+zx)%5Cgeq%200

[ برای مشاهده لینک ، لطفا با نام کاربری خود وارد شوید یا ثبت نام کنید ]^2+y^2+z^2+2(xy+yz+ zx)%5Cgeq%203(xy+yz+zx)

[ برای مشاهده لینک ، لطفا با نام کاربری خود وارد شوید یا ثبت نام کنید ](x+y+z)^2%5Cgeq%203( xy+yz+zx)

به علامات [ برای مشاهده لینک ، لطفا با نام کاربری خود وارد شوید یا ثبت نام کنید ] توجّه كنيد.
همچنين دقّت نماييد كه در هر دو عمليّات رفت و برگشت از فرض مثبت بودن x و y و z استفاده نشده است. و اين نا مساوي به ازاي هر سه عدد حقيقي دلخواه براي سه متغيّر ياد شده٬ برقرار است.

Parser
20-09-2009, 20:51
به نام معــشوق ازلی
پاسخ مسئله ی شنبه ی نهم

مجموع جزيي سري به صورت زير است


[ برای مشاهده لینک ، لطفا با نام کاربری خود وارد شوید یا ثبت نام کنید ]{n}=%5Csum_{k=0}^{n}%5Cfrac{x^{2^k}}{( x+1)(x^2+1)(x^4+1)(x^8+1)...(x^{2^k}+1)}

به استقرا مي توان نشان داد كه


[ برای مشاهده لینک ، لطفا با نام کاربری خود وارد شوید یا ثبت نام کنید ]{n}=%5Cfrac{x^{2^{n+1}}-x}{x^{2^{n+1}}-1}


عمليّات زير٬ قسمتي از برهان استقرا را نشان مي دهد. بديهي است كه تساوي مذكور به ازاي n=0 همچنين n=1 صحيح است. دقّت بفرماييد كه x>1


[ برای مشاهده لینک ، لطفا با نام کاربری خود وارد شوید یا ثبت نام کنید ]{n}=%5Cfrac{x^{2^{n+1}}-x}{x^{2^{n+1}}-1}%20%5C,%5C,,%5C,%5C,%20S_{n+1}=S_{n}+%5Cfrac{x^{ 2^{n+1}}}{(x+1)(x^2+1)(x^4+1)(x^8+1)...(x^{2^{n+1} }+1)}

[ برای مشاهده لینک ، لطفا با نام کاربری خود وارد شوید یا ثبت نام کنید ]{x^{2^{n+1}}}{(x+1)(x^2+1)(x^4+1) (x^8+1)...(x^{2^{n+1}}+1)}=%5Cfrac{(x-1)x^{2^{n+1}}}{x^{2^{n+2}}-1}

[ برای مشاهده لینک ، لطفا با نام کاربری خود وارد شوید یا ثبت نام کنید ]{n+1}=%5Cfrac{x^{2^{n+ 1}}-x}{x^{2^{n+1}}-1}+%5Cfrac{(x-1)x^{2^{n+1}}}{x^{2^{n+2}}-1}=%5Cfrac{(x^{2^{n+1}}-x)(x^{2^{n+1}}+1)+(x-1)x^{2^{n+1}}}{x^{2^{n+2}}-1}

[ برای مشاهده لینک ، لطفا با نام کاربری خود وارد شوید یا ثبت نام کنید ]{n+1}=%5Cfrac{x^{2^{n+ 2}}-x^{2^{n+1}+1}+x^{2^{n+1}}-x+x^{2^{n+1}+1}-x^{2^{n+1}}}{x^{2^{n+2}}-1}

[ برای مشاهده لینک ، لطفا با نام کاربری خود وارد شوید یا ثبت نام کنید ]{n+1}=%5Cfrac{x^{2^{n+ 2}}-x}{x^{2^{n+2}}-1}

اكنون مي توان به راحتي دريافت كه دنباله ي [ برای مشاهده لینک ، لطفا با نام کاربری خود وارد شوید یا ثبت نام کنید ]{n}به چه عددي همگراست. اين مقدار را S بناميد



[ برای مشاهده لینک ، لطفا با نام کاربری خود وارد شوید یا ثبت نام کنید ]{n%5Cto%20%5Cinfty%20}S_{n}=%5C lim_{n%5Cto%20%5Cinfty%20}%5Cfrac{x^{2^{n+1}}-x}{x^{2^{n+1}}-1}=1

eh_mn
23-09-2009, 23:14
الف. نشان دهيد براي هر سه عدد مثبت x و y و z داريم


[ برای مشاهده لینک ، لطفا با نام کاربری خود وارد شوید یا ثبت نام کنید ](x+y+z)^2\geq 3(xy+yz+zx)


ـــــــــــــــــــــــ
25 / 06 / 88

قسمت الف رو جناب Parser به درستی (و زیبایی) در اینجا

برای مشاهده محتوا ، لطفا وارد شوید یا ثبت نام کنید
حل كردند.

برای قسمت ب از همه دوستان به ویژه از عزیزانی كه وقت گذاشتن عذر خواهی می‌كنم. صورت سوال رو درست ننوشته بودم. نامساوی صحیح را كه با استفاده از نامساوی قسمت الف حل می‌شود، به عنوان قسمت الف سوال جدید در نظر گرفتم كه می‌تونید در پست بعدی اون رو ملاحظه كنید.


ــــــــــــــــــــــــ
01 / 07 / 88

eh_mn
23-09-2009, 23:18
الف. (با استفاده از مسأله‌ی هفته‌ی قبل) ثابت كنید اگر a و b و x و y و z همگی مثبت باشند آنگاه

[ برای مشاهده لینک ، لطفا با نام کاربری خود وارد شوید یا ثبت نام کنید ]{x}{ay+bz} + \frac{y}{az+bx} + \frac{z}{ax+by}\geq \frac{3}{a+b}

ب. برای هر a و b و c مثبت، ثابت كنید

[ برای مشاهده لینک ، لطفا با نام کاربری خود وارد شوید یا ثبت نام کنید ]{a}{b+c}+\frac{b}{a+c}+\frac{c}{a +b} \geq \frac{3}{2}
(راهنمایی: برای شروع مي‌توانید صورت همه‌ی عبارت‌های سمت چپ را طوری تغییر دهید كه به شكل [ برای مشاهده لینک ، لطفا با نام کاربری خود وارد شوید یا ثبت نام کنید ] تبدیل شود)



ـــــــــــــــــــــــــ
01 / 07 / 88

mofidy1
24-09-2009, 16:38
انتگرال زیر را محاسبه کنید:


[ برای مشاهده لینک ، لطفا با نام کاربری خود وارد شوید یا ثبت نام کنید ]{0}^{\infty}e^{-t^2}\,dt

راهنمایی: از تابع گاما استفاده کنید.

موفق باشید.

26 شهریور 1388

با سلام

از Parser که در پست 178 ([ برای مشاهده لینک ، لطفا با نام کاربری خود وارد شوید یا ثبت نام کنید ]) مساله را حل کردند، سپاس گزارم. البته نوشته ی ایشان مقداری تصحیح شد.

آموزش حل مساله:

باز هم ایده ی اتصال داده ها.

موفق باشید.

2 مهر 1388

mofidy1
24-09-2009, 16:43
با سلام

چگونه می توان با دو ظرف آب، یکی به گنجایش نه لیتر و دیگری به گنجایش چهار لیتر، شش لیتر آب از یک نهر برداشت؟!!

موفق باشید.

2 مهر 1388

amintnt
24-09-2009, 17:32
با سلام

چگونه می توان با دو ظرف آب، یکی به گنجایش نه لیتر و دیگری به گنجایش چهار لیتر، شش لیتر آب از یک نهر برداشت؟!!

موفق باشید.

2 مهر 1388
دوتا 4 لیتری پر میکنیم میریزیم به 9 لیتری... حالا 9 لیتری 1 لیتر جا داره... دوباره 4 لیتری رو پر میکنیم میریزیم به 9 لیتری تا پر بشه... حالا 3 لیتر داریم توی 4 لیتری... 9 لیتری رو خالی میکنیم... 3 لیتر رو میریزیم به 9 لیتری... 4 لیتری رو پر میکنیم و میریزیم روی 3 لیتری که توی ظرف 9 لیتری قرار داره... حالا 7 لیتر توی ظرف 9 لیتری داریم... 4 لیتری رو پر میکنیم میریزم به ظرف 9 لیتری که 2 لیتر جا داره... حالا ظرف 4 لیتری 2 لیتر توش هست... ظرف 9 لیتری رو خالی میکنیم و اون 2 لیتر رو به ظرف 9 لیتری انتقال میدیم... 4 لیتری رو پر میکنیم و میریزیم به 9 لیتری... در مجموع 6 لیتر توی 9 لیتری هست هم اکنون....

نوش جان!

Iron
26-09-2009, 10:31
سلام

البته ساده تره که 9 لیتری رو پر کنیم و دوتا 4 لیتر ازش برداریم و بریزیم تو نهر. 1 لیتر میمونه که اونو میریزیم تو 4 لیتری. دوباره 9 لیتری رو پر می کنیم و باهاش 3 لیتر باقی مونده چهار لیتری رو پر می کنیم. حالا توی 9 لیتری 6 لیتر آبه.:46:

Parser
26-09-2009, 14:23
به نام معشوق ازلی

جواب مسئله ی چهار شنبه ی دهم قسمت ب



برای هر a و b و c مثبت، ثابت كنید

[ برای مشاهده لینک ، لطفا با نام کاربری خود وارد شوید یا ثبت نام کنید ] %7Ba+c%7D+%5Cfrac%7Bc%7D%7Ba+b%7D%20%5Cgeq%20%5Cfr ac%7B3%7D%7B2%7D


ابتدا توجّه فرمایید که


[ برای مشاهده لینک ، لطفا با نام کاربری خود وارد شوید یا ثبت نام کنید ] 0%5C,%5C,%5Cfrac%7Ba%7D%7Bb+c%7D+%5Cfrac%7Bb%7D%7B c+a%7D+%5Cfrac%7Bc%7D%7Ba+b%7D%5Cgeq%20%5Cfrac%7B3 %7D%7B2%7D

[ برای مشاهده لینک ، لطفا با نام کاربری خود وارد شوید یا ثبت نام کنید ] Bb+c%7D-%5Cfrac%7Bb+c%7D%7Bb+c%7D+%5Cfrac%7Ba+b+c%7D%7Bc+a %7D-%5Cfrac%7Bc+a%7D%7Bc+a%7D+%5Cfrac%7Ba+b+c%7D%7Ba+b %7D-%5Cfrac%7Ba+b%7D%7Ba+b%7D%5Cgeq%20%5Cfrac%7B3%7D%7 B2%7D

[ برای مشاهده لینک ، لطفا با نام کاربری خود وارد شوید یا ثبت نام کنید ] Bb+c%7D+%5Cfrac%7Ba+b+c%7D%7Bc+a%7D+%5Cfrac%7Ba+b+ c%7D%7Ba+b%7D%5Cgeq%20%5Cfrac%7B3%7D%7B2%7D+3

[ برای مشاهده لینک ، لطفا با نام کاربری خود وارد شوید یا ثبت نام کنید ] 20%28%20%5Cfrac%7B1%7D%7Bb+c%7D+%5Cfrac%7B1%7D%7Bc +a%7D+%5Cfrac%7B1%7D%7Ba+b%7D%20%5Cright%20%29%5Cg eq%209

[ برای مشاهده لینک ، لطفا با نام کاربری خود وارد شوید یا ثبت نام کنید ] 29+%28a+b%29%29%5Cleft%20%28%20%5Cfrac%7B1%7D%7Bb+ c%7D+%5Cfrac%7B1%7D%7Bc+a%7D+%5Cfrac%7B1%7D%7Ba+b% 7D%20%5Cright%20%29%5Cgeq%209


حال، فرض کنید که [ برای مشاهده لینک ، لطفا با نام کاربری خود وارد شوید یا ثبت نام کنید ] , [ برای مشاهده لینک ، لطفا با نام کاربری خود وارد شوید یا ثبت نام کنید ] , [ برای مشاهده لینک ، لطفا با نام کاربری خود وارد شوید یا ثبت نام کنید ] ، بدیهی است که A,B,C هر سه مثبتند.

اکنون اگر نامساوی زیر اثبات گردد، مسئله حل گشته است.


[ برای مشاهده لینک ، لطفا با نام کاربری خود وارد شوید یا ثبت نام کنید ] 0%5C,%5C,%28A+B+C%29%5Cleft%20%28%20%5Cfrac%7B1%7D %7BA%7D+%5Cfrac%7B1%7D%7BB%7D+%5Cfrac%7B1%7D%7BC%7 D%20%5Cright%20%29%5Cgeq%209


همان طور که پیش تر گفته شد:

[ برای مشاهده لینک ، لطفا با نام کاربری خود وارد شوید یا ثبت نام کنید ] D%7BB%7D+%5Cfrac%7BB%7D%7BA%7D%5Cgeq%202

و به همین تزتیب

[ برای مشاهده لینک ، لطفا با نام کاربری خود وارد شوید یا ثبت نام کنید ] D%7BC%7D+%5Cfrac%7BC%7D%7BB%7D%5Cgeq%202

[ برای مشاهده لینک ، لطفا با نام کاربری خود وارد شوید یا ثبت نام کنید ] D%7BA%7D+%5Cfrac%7BA%7D%7BC%7D%5Cgeq%202

پس با برقرار بودن همه ی شرایط می توان اظهار داشت که :


[ برای مشاهده لینک ، لطفا با نام کاربری خود وارد شوید یا ثبت نام کنید ] %7BA%7D%7BA%7D+%5Cfrac%7BA%7D%7BB%7D+%5Cfrac%7BA%7 D%7BC%7D+%5Cfrac%7BB%7D%7BB%7D+%5Cfrac%7BB%7D%7BA% 7D+%5Cfrac%7BB%7D%7BC%7D+%5Cfrac%7BC%7D%7BC%7D+%5C frac%7BC%7D%7BA%7D+%5Cfrac%7BC%7D%7BB%7D%5Cgeq%201 +1+1+6

[ برای مشاهده لینک ، لطفا با نام کاربری خود وارد شوید یا ثبت نام کنید ] 0%28%20%5Cfrac%7B1%7D%7BA%7D+%5Cfrac%7B1%7D%7BB%7D +%5Cfrac%7B1%7D%7BC%7D%20%5Cright%20%29%5Cgeq%209


به علامات [ برای مشاهده لینک ، لطفا با نام کاربری خود وارد شوید یا ثبت نام کنید ] توجّه کنید.


[ برای مشاهده لینک ، لطفا با نام کاربری خود وارد شوید یا ثبت نام کنید ] 28A+B+C%29%5Cleft%20%28%20%5Cfrac%7B1%7D%7BA%7D+%5 Cfrac%7B1%7D%7BB%7D+%5Cfrac%7B1%7D%7BC%7D%20%5Crig ht%20%29%5Cgeq%209

mir@
27-09-2009, 05:47
حل مسئله شنبه نهم

پاسخ Parser در پست 182 صحیح است.

mir@
27-09-2009, 06:02
.
نشان دهید که هر عدد صحیح را می توان به صورت [ برای مشاهده لینک ، لطفا با نام کاربری خود وارد شوید یا ثبت نام کنید ]^2+y^2-5z^2 نوشت که در آن x و y و z اعداد صحیح هستند.

Parser
27-09-2009, 11:31
به نام معشوق ازلی

با عرض پوزش، راه حل یاد شده برای مسئله ی چهارشنبه ی دهم، قسمت الف، نادرست بود.

حلال کنید! جدّی می گم.

ali_hp
27-09-2009, 18:36
شازده کوچولو، روی سیاره کوچکی زندگی می کند که شعاعش 60 متر است. روزی با شروع از روی خط استوا، 30pi متر به شرق ، 20pi متر به شمال، 30pi متر به غرب و سرانجام 20pi متر به جنوب می رود. شازده کوچولو تا مکان اولش چند متر فاصله دارد؟
سلام،بابت غیبت هفته گذشته از همه عزیزان عذرخواهی می کنم.
پاسخ دوست عزیزمون،zahedy2006 درسته،منظور سوال فاصله زمین بوده.
حل:نصف النهار گذرنده از نقطه شروع حرکت را نصف النهار مبدا در نظر بگیرید،در مرحله اول حرکت،شازده یک چهارم مدار استوا را می پیماید،پس روی نصف النهار 90 درجه قرار می گیرد،در مرحله بعد یک ششم دایره عظیمه را می پیماید،یعنی کمان 60 درجه،و روی مداری(موازی استوا)به شعاع (30pi )قرار می گیرد(چرا؟) در مرحله سوم نصف این مدار را می پیماید،و روی نصف النهار منفی نود درجه قرار می گیرد،و در مرحله آخر روی نصف النهار منفی نود و روی خط استوا قرار می گیرد،که به وضوح فاصله آن تا نقطه شروع 30pi است.

ali_hp
27-09-2009, 19:03
احکام زیر را ثابت یا رد کنید:
الف)برای هر عدد طبیعی n،دایره ای در صفحه وجود دارد که دقیقا شامل n نقطه با مختصات صحیح باشد.
ب)برای هر عدد طبیعی n،کره ای در فضا وجود دارد که دقیقا شامل n نقطه با مختصات صحیح باشد.

eh_mn
30-09-2009, 23:57
الف. (با استفاده از مسأله‌ی هفته‌ی قبل) ثابت كنید اگر a و b و x و y و z همگی مثبت باشند آنگاه

[ برای مشاهده لینک ، لطفا با نام کاربری خود وارد شوید یا ثبت نام کنید ]{x}{ay+bz} + \frac{y}{az+bx} + \frac{z}{ax+by}\geq \frac{3}{a+b}

ب. برای هر a و b و c مثبت، ثابت كنید

[ برای مشاهده لینک ، لطفا با نام کاربری خود وارد شوید یا ثبت نام کنید ]{a}{b+c}+\frac{b}{a+c}+\frac{c}{a +b} \geq \frac{3}{2}
(راهنمایی: برای شروع مي‌توانید صورت همه‌ی عبارت‌های سمت چپ را طوری تغییر دهید كه به شكل [ برای مشاهده لینک ، لطفا با نام کاربری خود وارد شوید یا ثبت نام کنید ] تبدیل شود)



ـــــــــــــــــــــــــ
01 / 07 / 88

جناب Parser زحمت كشيدن و در

برای مشاهده محتوا ، لطفا وارد شوید یا ثبت نام کنید
جوابي براي اين مسأله نوشتن كه ازشون خواهش مي‌كنم اين قسمت رو توضيحي بدن. آيا علامت نامساوي‌هاي زير


[ برای مشاهده لینک ، لطفا با نام کاربری خود وارد شوید یا ثبت نام کنید ] D%7Bz%7D+%5Cfrac%7By%7D%7Bx%7D+%5Cfrac%7Bz%7D%7By% 7D%5Cgeq%203%5Csqrt%5B3%5D%7B%5Cfrac%7Bx%7D%7Bz%7D %5Cfrac%7By%7D%7Bx%7D%5Cfrac%7Bz%7D%7By%7D%7D%5CRi ghtarrow%20a%5Cleft%20%28%20%5Cfrac%7Bx%7D%7Bz%7D+ %5Cfrac%7By%7D%7Bx%7D+%5Cfrac%7Bz%7D%7By%7D%20%5Cr ight%20%29%5Cgeq%203a
[ برای مشاهده لینک ، لطفا با نام کاربری خود وارد شوید یا ثبت نام کنید ] D%7Bz%7D+%5Cfrac%7By%7D%7Bx%7D+%5Cfrac%7Bz%7D%7By% 7D%5Cgeq%203%5Csqrt%5B3%5D%7B%5Cfrac%7Bx%7D%7Bz%7D %5Cfrac%7By%7D%7Bx%7D%5Cfrac%7Bz%7D%7By%7D%7D%5CRi ghtarrow%20b%5Cleft%20%28%20%5Cfrac%7Bx%7D%7Bz%7D+ %5Cfrac%7By%7D%7Bx%7D+%5Cfrac%7Bz%7D%7By%7D%20%5Cr ight%20%29%5Cgeq%203b


با علامت نتيجه‌اي كه از آن گرفته شده يعني


[ برای مشاهده لینک ، لطفا با نام کاربری خود وارد شوید یا ثبت نام کنید ] %28%20%5Cfrac%7Bx%7D%7Bz%7D+%5Cfrac%7By%7D%7Bx%7D+ %5Cfrac%7Bz%7D%7By%7D%20%5Cright%20%29+b%5Cleft%20 %28%20%5Cfrac%7By%7D%7Bz%7D+%5Cfrac%7Bz%7D%7Bx%7D+ %5Cfrac%7Bx%7D%7By%7D%20%5Cright%20%29%5Cleq%203%2 8a+b%29%22%3E

[ برای مشاهده لینک ، لطفا با نام کاربری خود وارد شوید یا ثبت نام کنید ] %28%20%5Cfrac%7Bx%7D%7Bz%7D+%5Cfrac%7By%7D%7Bx%7D+ %5Cfrac%7Bz%7D%7By%7D%20%5Cright%20%29+b%5Cleft%20 %28%20%5Cfrac%7By%7D%7Bz%7D+%5Cfrac%7Bz%7D%7Bx%7D+ %5Cfrac%7Bx%7D%7By%7D%20%5Cright%20%29%5Cleq%203%2 8a+b%29


مطابقت دارد؟

حل مورد نظر به صورت زير است.
نامساوي مورد سوال درست است اگر و فقط اگر نامساوي زير درست باشد


[ برای مشاهده لینک ، لطفا با نام کاربری خود وارد شوید یا ثبت نام کنید ](a&plus;b)\left(\frac{x}{ay&plus;bz}&plus;\frac{y}{az&plus;b x}&plus;\frac{z}{ax&plus;by}\right)\geq&space;3

اگر طرفين را در [ برای مشاهده لینک ، لطفا با نام کاربری خود وارد شوید یا ثبت نام کنید ](xy+yz+zx) كه عددي مثبت است ضرب كنيم، به نامساوي معادل زير مي‌رسيم


[ برای مشاهده لینک ، لطفا با نام کاربری خود وارد شوید یا ثبت نام کنید ](a+b)(xy+yz+zx)\left(\frac{x}{ay+bz}+\ frac{y}{az+bx}+\frac{z}{ax+by}\right)\geq 3(xy+yz+zx)~~(1)

اما داريم


[ برای مشاهده لینک ، لطفا با نام کاربری خود وارد شوید یا ثبت نام کنید ](a&plus;b)(xy&plus;yz&plus;zx)&space;=&space;x(ay&plus;bz)&plus;y(az&plus;bx)&plus;z(ax &plus;by)

بنابراين با استفاده از نامساوي كشي-شوارتز داريم


[ برای مشاهده لینک ، لطفا با نام کاربری خود وارد شوید یا ثبت نام کنید ](%20x(ay+bz)+y(az+bx)+z(ax+by)%20 \right%20)\left(\frac{x}{ay+bz}+\frac{y}{az+bx}+\f rac{z}{ax+by}\right)\geq%20(x+y+z)^2


حال با استفاده از نامساوي هفته‌ي قبل و نامساوي اخير نامساوي (1) به راحتي اثبات مي‌شود.


ب. اين قسمت را Parser با به خوبي حل كردند. راه حل ايشون اينجاست

برای مشاهده محتوا ، لطفا وارد شوید یا ثبت نام کنید

البته براي به دست آوردن اين نامعادله مي توانستيم در قسمت الف به جاي a,b عدد يك را قرار دهيم!! اما هدف آشنايي با ايده‌هاي جديد بود.



ـــــــــــــــــــــــــ
08 / 07 / 88

eh_mn
01-10-2009, 00:04
ثابت كنيد براي هر عدد فرد مانند n تعداد اعداد فرد دنباله‌ي زير، فرد است


[ برای مشاهده لینک ، لطفا با نام کاربری خود وارد شوید یا ثبت نام کنید ]{n}{1},\binom{n}{2},\dots,\binom{n }{\frac{n-1}{2}}


(توجه كنيد كه براي هر n و هر دو عدد حقيقي a و b داريم


[ برای مشاهده لینک ، لطفا با نام کاربری خود وارد شوید یا ثبت نام کنید ](a&plus;b)^n=\sum_{k=0}^n\binom{n}{k}a^kb^{n-k}
.)
ــــــــــــــــــ
08 / 07 / 88

mofidy1
01-10-2009, 21:15
با سلام

چگونه می توان با دو ظرف آب، یکی به گنجایش نه لیتر و دیگری به گنجایش چهار لیتر، شش لیتر آب از یک نهر برداشت؟!!

موفق باشید.

2 مهر 1388

از amintnt و Iron برای راه حل های درستشان در پست 187 ([ برای مشاهده لینک ، لطفا با نام کاربری خود وارد شوید یا ثبت نام کنید ]) و پست 188 ([ برای مشاهده لینک ، لطفا با نام کاربری خود وارد شوید یا ثبت نام کنید ]) تشکر می کنم.

آموزش حل مساله:

ابتدا فرض کنید مساله حل شده است و سعی کنید مرحله به مرحله به عقب برگردید. در مساله ی بالا می توان از این شگرد بسیار خوب، استفاده کرد.

موفق باشید.

9 مهر 1388

mofidy1
01-10-2009, 21:27
با سلام

عبارت زیر را در اعداد حقیقی تجزیه کنید:



[ برای مشاهده لینک ، لطفا با نام کاربری خود وارد شوید یا ثبت نام کنید ](x^4&plus;x^3&plus;x^2&plus;x&plus;1)

موفق باشید.

9 مهر 1388

mir@
02-10-2009, 20:59
حل مسئله شنبه دهم


.
نشان دهید که هر عدد صحیح را می توان به صورت [ برای مشاهده لینک ، لطفا با نام کاربری خود وارد شوید یا ثبت نام کنید ] نوشت که در آن x و y و z اعداد صحیح هستند.

اگر عدد داده شده زوج یا فرد باشد به صورت زیر می توان آن را نوشت:


[ برای مشاهده لینک ، لطفا با نام کاربری خود وارد شوید یا ثبت نام کنید ] 282n%29%5E2-5n%5E2%20%5Cqquad%5Cqquad%5Cqquad%20m=2n+1%20%5Cen d%7Bcases%7D

mir@
02-10-2009, 21:04
تمام ماتریس های دو در دوی B و C را بیابید که


[ برای مشاهده لینک ، لطفا با نام کاربری خود وارد شوید یا ثبت نام کنید ] 20-1%20&%201%20%5C%5C%20%200%20&%20%20-2%20%5Cend%7Bpmatrix%7D

Parser
03-10-2009, 10:09
ثابت كنيد براي هر عدد فرد مانند n تعداد اعداد فرد دنباله‌ي زير، فرد است


[ برای مشاهده لینک ، لطفا با نام کاربری خود وارد شوید یا ثبت نام کنید ] B2%7D,%5Cdots,%5Cbinom%7Bn%7D%7B%5Cfrac%7Bn-1%7D%7B2%7D%7D







همان طور که می دانید مجموع دو عدد فرد، عددی است زوج، همچنین مجموع دو عدد زوج عددی زوج است. و مجموع یک عدد فرد و یک عدد زوج، فرد است.

به راحتی می توان به استقرا نشان داد که اگر مجموع چند عدد طبیعی، فرد باشد، تعداد اعداد فرد، در بین اعدادی که با هم جمع شده اند، فرد است.

اکنون بسط دو جمله ای نیوتون را در نظر بگیرید.


[ برای مشاهده لینک ، لطفا با نام کاربری خود وارد شوید یا ثبت نام کنید ] %5Cbinom%7Bn%7D%7Bk%7Da%5Enb%5E%7Bn-k%7D


اگر n طبیعی و بزرگتر از یک و فرد فرض شود، و a و b هر دو یک در نظر گرفته شوند،



[ برای مشاهده لینک ، لطفا با نام کاربری خود وارد شوید یا ثبت نام کنید ] %5C,%5C,%5C,%5C,,%5C,%5C,%5C,%5C,2%5E%7Bn%7D=%5Csu m_%7Bk=0%7D%5E%7Bn%7D%5Cbinom%7Bn%7D%7Bk%7D=%5Cbin om%7Bn%7D%7B0%7D+%5Cbinom%7Bn%7D%7B1%7D+...+%5Cbin om%7Bn%7D%7Bn%7D

[ برای مشاهده لینک ، لطفا با نام کاربری خود وارد شوید یا ثبت نام کنید ] Cbinom%7Bn%7D%7B0%7D+%5Cbinom%7Bn%7D%7B1%7D+...+%5 Cbinom%7Bn%7D%7B%5Cfrac%7Bn-1%7D%7B2%7D%7D%20%5Cright%20%29+%5Cleft%20%28%20%5 Cbinom%7Bn%7D%7B%5Cfrac%7Bn+1%7D%7B2%7D%7D+...+%5C binom%7Bn%7D%7Bn-1%7D+%5Cbinom%7Bn%7D%7Bn%7D%20%5Cright%20%29


در سمت راست تساوی، پرانتز اوّل، [ برای مشاهده لینک ، لطفا با نام کاربری خود وارد شوید یا ثبت نام کنید ] عدد با هم جمع شده اند. ( دقّت کنید که n فرد است پس [ برای مشاهده لینک ، لطفا با نام کاربری خود وارد شوید یا ثبت نام کنید ] عددی طبیعی است. )

در پرانتز دوم نیز [ برای مشاهده لینک ، لطفا با نام کاربری خود وارد شوید یا ثبت نام کنید ] عدد با یکدیگر جمع گشته اند.

از طرفی می توان نشان داد که [ برای مشاهده لینک ، لطفا با نام کاربری خود وارد شوید یا ثبت نام کنید ] m%7Bn%7D%7Bn-k%7D ( k طبیعی و کوچکتر از n ) ، پس:


[ برای مشاهده لینک ، لطفا با نام کاربری خود وارد شوید یا ثبت نام کنید ] f%7BN%7D%5C,%5C,%5CRightarrow%20%5C,%5C,2%5En=2%5C left%20%28%20%5Cbinom%7Bn%7D%7B0%7D+%5Cbinom%7Bn%7 D%7B1%7D+...+%5Cbinom%7Bn%7D%7B%5Cfrac%7Bn-1%7D%7B2%7D%7D%20%5Cright%20%29
[ برای مشاهده لینک ، لطفا با نام کاربری خود وارد شوید یا ثبت نام کنید ] .+%5Cbinom%7Bn%7D%7B%5Cfrac%7Bn-1%7D%7B2%7D%7D


با توجّه به اینکه n>1، [ برای مشاهده لینک ، لطفا با نام کاربری خود وارد شوید یا ثبت نام کنید ] عددی است زوج. پس [ برای مشاهده لینک ، لطفا با نام کاربری خود وارد شوید یا ثبت نام کنید ] فرد است. بنابراین عبارت سمت راست تساوی فوق عددی فرد است.

با توجّه به مقدّمه ی ابتدای برهان، باید در بین [ برای مشاهده لینک ، لطفا با نام کاربری خود وارد شوید یا ثبت نام کنید ] عدد سمت راست تساوی که با هم جمع گردیده اند، تعداد اعداد فرد، فرد باشد.

Parser
03-10-2009, 14:56
به نام معشوق ازلی


با سلام

عبارت زیر را در اعداد حقیقی تجزیه کنید:



[ برای مشاهده لینک ، لطفا با نام کاربری خود وارد شوید یا ثبت نام کنید ]


سلام


[ برای مشاهده لینک ، لطفا با نام کاربری خود وارد شوید یا ثبت نام کنید ] +%5Cfrac%7B1+%5Csqrt%7B5%7D%7D%7B2%7Dx+1%29%28x%5E 2+%5Cfrac%7B1-%5Csqrt%7B5%7D%7D%7B2%7Dx+1%29

mofidy1
03-10-2009, 23:51
به نام معشوق ازلی



سلام


[ برای مشاهده لینک ، لطفا با نام کاربری خود وارد شوید یا ثبت نام کنید ](x%5E4+x%5E3+x%5E2+x+1)%5Cequiv%20%28x% 5E2+%5Cfrac%7B1+%5Csqrt%7B5%7D%7D%7B2%7Dx+1%29%28x %5E2+%5Cfrac%7B1-%5Csqrt%7B5%7D%7D%7B2%7Dx+1%29


با سلام

چگونه به این تجزیه رسیدید؟ راه حل را توضیح دهید.

با تشکر

11 مهر 1388

ali_hp
05-10-2009, 00:11
احکام زیر را ثابت یا رد کنید:
الف)برای هر عدد طبیعی n،دایره ای در صفحه وجود دارد که دقیقا شامل n نقطه با مختصات صحیح باشد.
ب)برای هر عدد طبیعی n،کره ای در فضا وجود دارد که دقیقا شامل n نقطه با مختصات صحیح باشد.
راهنمایی برای حل مساله یکشنبه دهم:
ابتدا سعی کنید ثابت کنید که:
برای الف:نقطه ای در صفحه وجود دارد که از نقاط با مختصات صحیح در صفحه فواصلی متمایز دارد.
حال دایره ای به مرکز نقطه مذکور طوری در نظر بگیرید که حداقل N نقطه با مختصات صحیح داخلش باشد،حال با توجه به اینکه با کاهش شعاع این دایره هیچ دو نقطه صحیحی به طور همزمان از دایره خارج نمی شوند،می توان شعاع دایره را کاهش داد و به دایره ای رسید که دقیقا N نقطه صحیح داخلش وجود دارد.
قسمت ب نیز با استدلال مشابه حل می شود.
مساله یکشنبه یازدهم:
الف)ثابت کنید که در صفحه نقطه ای وجود دارد که از نقاط صحیح صفحه فواصلی متمایز دارد.
ب)ثابت کنید که در صفحه نقطه ای وجود دارد که از نقاط صحیح فضا فواصلی متمایز دارد.

Parser
05-10-2009, 10:23
به نام معشوق ازلی



با سلام

چگونه به این تجزیه رسیدید؟ راه حل را توضیح دهید.



سلام

عبارت تجزیه شده ی یک چند جمله ای درجه ی چهار را به صورت حاصل ضرب دو چند جمله ای درجه ی دو، یا یک عبارت درجه ی یک و یک درجه ی سه، یا مثلاً به صورت حاصل ضرب سه چند جمله ای، دو عبارت با درجه ی یک و یکی با درجه ی دو و... در نظر گرفت. ( دقّت داشته باشید ک درجه ی عبارات، طبیعی در نظر گرفته شده اند. )

از آنجا که چند جمله ای [ برای مشاهده لینک ، لطفا با نام کاربری خود وارد شوید یا ثبت نام کنید ] ریشه ی حقیقی ندارد، همچنین به دلیل اینکه در صورت سؤال نیز تجزیه در حوزه ی اعداد حقیقی مطرح است، مناسب است که تجزیه ی چند جمله ای درجه ی چهار در صورت سؤال، به فرم حاصل ضرب دو عبارت درجه ی دو باشد.

دقّت کنید که اگر حتّی یکی از عبارات، در شکل تجزیه شده، از درجه ی یک باشد، لازم است که همان عبارت با درجه ی یک، به ازای ریشه ای از [ برای مشاهده لینک ، لطفا با نام کاربری خود وارد شوید یا ثبت نام کنید ] صفر گردد، که در آن صورت، ضریب x یا مقدار ثابت ( یا هر دو ) عدد مختلط با مؤلّفه ی موهومی غیر صفر است.

حال می توان از روش اکتشافی به صورت علمی، بهره برد.


[ برای مشاهده لینک ، لطفا با نام کاربری خود وارد شوید یا ثبت نام کنید ] %20%28x%5E2+ax+c%29%28x%5E2+bx+%5Cfrac%7B1%7D%7Bc% 7D%29


اتّحاد فوق به ازای هر عدد حقیقی برای x برقرار است. یا به عبارتی دامنه ی اتّحاد برای تک متغیّر x ، مجموعه ی اعداد حقیقی است. به علامت [ برای مشاهده لینک ، لطفا با نام کاربری خود وارد شوید یا ثبت نام کنید ] توجّه کنید. عبارت فوق اتّحاد است. نه صرفاً یک معادله که به ازای برخی مقادیر برای x برقرار باشد. پس تحت شرایطی ( از جمله ی شرایط مربوط به دامنه، همچنین بررسی یکتایی جواب و ... که در اینجا برقرار هستند ) می توان به موازنه ی ضرایب طرفین پرداخت.

شاید این سؤال پیش بیاید که چرا ضریب های [ برای مشاهده لینک ، لطفا با نام کاربری خود وارد شوید یا ثبت نام کنید ] ها در دو پرانتز سمت راست، برابر با یک هستند.

در جواب می توان گفت که اگر قرار باشد با ضرب دو پرانتز سمت راست، چند جمله ای سمت چپ حاصل گردد، چون رابطه ی برابری فوق، به ازای هر عدد حقیقی برای x برقرار است، لازم است که ضریب [ برای مشاهده لینک ، لطفا با نام کاربری خود وارد شوید یا ثبت نام کنید ] در طرف چپ، مانند طرف راست، یک باشد. جمله ی درجه ی چهار تنها از دو جمله ی درجه ی دو، ( [ برای مشاهده لینک ، لطفا با نام کاربری خود وارد شوید یا ثبت نام کنید ] ها ) واقع در دو پرانتز، ایجاد می شود، پس باید ضریب های حقیقی و غیر صفر ها، عکس هم باشند، با تغییر آن دو، ضریب های x ها و مقدار ها ثابت نیز به شکلی منطقی تغییر می کنند، و نتیجه ی مطلوب حاصل می شود.


[ برای مشاهده لینک ، لطفا با نام کاربری خود وارد شوید یا ثبت نام کنید ] arrow%20%28x%5E2+ax+c%29%28x%5E2+bx+%5Cfrac%7B1%7D %7Bc%7D%29=%5Calpha%20%5Cfrac%7B1%7D%7B%5Calpha%20 %7D%28x%5E2+ax+c%29%28x%5E2+bx+%5Cfrac%7B1%7D%7Bc% 7D%29

[ برای مشاهده لینک ، لطفا با نام کاربری خود وارد شوید یا ثبت نام کنید ] %20ax+%5Calpha%20c%29%28%5Cfrac%7B1%7D%7B%5Calpha% 20%7Dx%5E2+%5Cfrac%7Bb%7D%7B%5Calpha%20%7Dx+%5Cfra c%7B1%7D%7B%5Calpha%20c%7D%29

اکنون می توان نوشت:


[ برای مشاهده لینک ، لطفا با نام کاربری خود وارد شوید یا ثبت نام کنید ] %20x%5E4+%28a+b%29x%5E3+%28ab+c+%5Cfrac%7B1%7D%7Bc %7D%29x%5E2+%28%5Cfrac%7Ba%7D%7Bc%7D+bc%29x+1


عدد یک در سمت چپ نیز تنها با ضرب مقادیر غیر صفر ثابت، در دو پرانتز ایجاد می گردد. پس دو مقدار حقیقی غیر صفر ثابت گفته شده، عکس هم هستند.

حال، می شود به اکتشاف دست زد!
می توان روابط زیر را به منظور دستیابی به نتیجه ی مورد نظر در نظر گرفت.


[ برای مشاهده لینک ، لطفا با نام کاربری خود وارد شوید یا ثبت نام کنید ] x%7D%20a+b=1%5C,%5C,%5C,%5C,%5C,%5C,%5C,%5C,%5C,%5 C,%5C,%5C,%5C,%5C,%5C%5C%20ab+c+%5Cfrac%7B1%7D%7Bc %7D=1%5C%5C%20%5Cfrac%7Ba%7D%7Bc%7D+bc=1%5C,%5C,%5 C,%5C,%5C,%5C,%5C,%5C,%5C,%20%5Cend%7Bmatrix%7D%5C right.


پس از حلّ دستگاه فوق ( جایگزینیa با b+1 - ) به عنوان یک جواب مطلوب، می توان در نظر گرفت c=1

اکنون باید به دنبال دو عدد گشت ( a,b ) به نحوی که جمع آن دو یک، و حاصل ضربشان منفی یک شود.

با تشکیل و حلّ معادله ی درجه ی دوم [ برای مشاهده لینک ، لطفا با نام کاربری خود وارد شوید یا ثبت نام کنید ] مقدار های [ برای مشاهده لینک ، لطفا با نام کاربری خود وارد شوید یا ثبت نام کنید ] %7B2%7D و [ برای مشاهده لینک ، لطفا با نام کاربری خود وارد شوید یا ثبت نام کنید ]بدست می آیند.

درنتیجه می توان( پس از امتحان درستی عبارت زیر ( برای هر عدد حقیقی ) ) ، اظهار داشت


[ برای مشاهده لینک ، لطفا با نام کاربری خود وارد شوید یا ثبت نام کنید ] %20%28x%5E2+%5Cfrac%7B1-%5Csqrt%7B5%7D%7D%7B2%7Dx+1%29%28x%5E2+%5Cfrac%7B1-%5Csqrt%7B5%7D%7D%7B2%7Dx+1%29


تأکید دوباره: به علامت [ برای مشاهده لینک ، لطفا با نام کاربری خود وارد شوید یا ثبت نام کنید ]توجّه نمایید. x هر عدد حقیقی می تواند باشد.

هر جایی نمی توان به موازنه ی ضرایب پرداخت....

eh_mn
07-10-2009, 19:16
ثابت كنيد براي هر عدد فرد مانند n تعداد اعداد فرد دنباله‌ي زير، فرد است


[ برای مشاهده لینک ، لطفا با نام کاربری خود وارد شوید یا ثبت نام کنید ]{n}{1},\binom{n}{2},\dots,\binom{n }{\frac{n-1}{2}}


(توجه كنيد كه براي هر n و هر دو عدد حقيقي a و b داريم


[ برای مشاهده لینک ، لطفا با نام کاربری خود وارد شوید یا ثبت نام کنید ](a&plus;b)^n=\sum_{k=0}^n\binom{n}{k}a^kb^{n-k}
.)
ــــــــــــــــــ
08 / 07 / 88

با سلام و درود فراوان

از Parser عزيز كه زحمت كشيدن حل اين مسأله رو با توضيحات كافي در اينجا

برای مشاهده محتوا ، لطفا وارد شوید یا ثبت نام کنید
انجام دادن بسيار ممنون.


موفق باشيد


ـــــــــــــــــــ
15 / 07 / 88

eh_mn
07-10-2009, 19:23
نشان دهيد سري زير به عددي اصم (غير گويا) همگراست


[ برای مشاهده لینک ، لطفا با نام کاربری خود وارد شوید یا ثبت نام کنید ]{n=1}^\infty&space;\frac{1}{2^{n!}}




ــــــــــــــــــــ
15 / 07 / 88

CppBuilder2006
08-10-2009, 08:21
نشان دهيد سري زير به عددي اصم (غير گويا) همگراست


[ برای مشاهده لینک ، لطفا با نام کاربری خود وارد شوید یا ثبت نام کنید ]{n=1}^\infty&space;\frac{1}{2^{n!}}




ــــــــــــــــــــ
15 / 07 / 88

[ برای مشاهده لینک ، لطفا با نام کاربری خود وارد شوید یا ثبت نام کنید ] dFkw1kwqzjnV8FVzaUhCLqrdFqp0dSsuzb-o_EQoZapaaEEHg/image_thumb[5].png

mir@
10-10-2009, 02:21
حل مسئله شنبه یازدهم


تمام ماتریس های دو در دوی B و C را بیابید که


[ برای مشاهده لینک ، لطفا با نام کاربری خود وارد شوید یا ثبت نام کنید ] 20-1%20&%201%20%5C%5C%20%200%20&%20%20-2%20%5Cend%7Bpmatrix%7D



[ برای مشاهده لینک ، لطفا با نام کاربری خود وارد شوید یا ثبت نام کنید ] 1%20%5C%5C%200%20&%20-2%20%5Cend%7Bpmatrix%7D%5CRightarrow%20A%5E2+3A+2I =0%20%5CRightarrow%20A%5E3+3A%5E2+2A=0%20%5C%5C%20 %5CRightarrow%20%28A+I%29%5E3=A%5E3+3A%5E2+3A+I=A+ I%20%5CRightarrow%20A=%28A+I%29%5E3-I%5C%5C%5CRightarrow%20%20B=A+I,%20C=-I

mir@
10-10-2009, 02:46
برای سه عدد حقیقی [ برای مشاهده لینک ، لطفا با نام کاربری خود وارد شوید یا ثبت نام کنید ] و تابع f پیوسته روی [ برای مشاهده لینک ، لطفا با نام کاربری خود وارد شوید یا ثبت نام کنید ] و مشتق پذیر روی[ برای مشاهده لینک ، لطفا با نام کاربری خود وارد شوید یا ثبت نام کنید ] مشتق تابع روی کل بازه مطلقا صعودی است. نشان دهید:


[ برای مشاهده لینک ، لطفا با نام کاربری خود وارد شوید یا ثبت نام کنید ]

ali_hp
12-10-2009, 01:18
مساله یکشنبه یازدهم:
الف)ثابت کنید که در صفحه نقطه ای وجود دارد که از نقاط صحیح صفحه فواصلی متمایز دارد.
ب)ثابت کنید که در صفحه نقطه ای وجود دارد که از نقاط صحیح فضا فواصلی متمایز دارد.
حل:
الف)عمود منصف بین هر دو نقطه صحیح در صفحه را در نظر بگیرید،تعداد آنها شماراست(چرا؟) بنابر این تمام صفحه را نمی پوشانند( در غیر اینصورت یک خط غیر از این عمود منصفها را در نظر بگیرید،هرکدام از این شمارا عمود منصف این خط را حداکثر در یک نقطه قطع می کند،پس این خط با شمارا نقطه پوشانده شده است که
تناقض است!)پس نقطه ای وجود دارد که روی این عمود منصف ها نیست،و از نقاط صحیح صفحه فواصلی متمایز دارد.
ب)صفحه عمود منصف بین هر دو نقطه صحیح در فضا را در نظر بگیرید،تعداد آنها شماراست(چرا؟)بنابر این تمام صفحه را نمی پوشانند.(در غیر این صورت یک صفحه غیر از این صفحات عمود منصفها را در نظر بگیرید،هریک از این صفحات عمود منصف این صفحه را حداکثر در یک خط قطع می کند،پس این صفحه حداکثر با تعداد متناهی خط پوشانده شده است که طبق الف تناقض است!) پس نقطه ای وجود دارد که روی این عمود منصف ها نیست،و از نقاط صحیح فضا فواصلی متمایز دارد.

ali_hp
12-10-2009, 01:24
مجموع تعدادی عدد طبیعی برابر هزار است،بیشترین مقدار ممکن برای حاصلضرب آنها چقدر است؟

eh_mn
14-10-2009, 13:32
نشان دهيد سري زير به عددي اصم (غير گويا) همگراست


[ برای مشاهده لینک ، لطفا با نام کاربری خود وارد شوید یا ثبت نام کنید ]{n=1}^\infty&space;\frac{1}{2^{n!}}




ــــــــــــــــــــ
15 / 07 / 88

سلام

از CppBuilder2006 به خاطر اين كه زحمت كشيدن و در

برای مشاهده محتوا ، لطفا وارد شوید یا ثبت نام کنید
نظرشون رو اعلام كردن ممنونم.
اصلا واضح نيست كه

[ برای مشاهده لینک ، لطفا با نام کاربری خود وارد شوید یا ثبت نام کنید ]{n=1}^\infty\sum_{i=0}^mb_i10^{i-n!}
عددي اعشاري است. توجه كنين كه در حساب يك مجموع نامتناهي حدگيري انجام مي‌شود. براي اين كه منظورم واضح تر بشه راه حل CppBuilder2006 رو براي همين مجموع وقتي به جاي n فاكتوريل خود n رو بذاريم بازنويسي كنين.

اما راه حل:

مشابه CppBuilder2006، بدون اين كه به كليت آسيبي برسد ما هم به جاي 2 از 10 استفاده مي‌كنيم. همگرا بودن سري واضح است چون


[ برای مشاهده لینک ، لطفا با نام کاربری خود وارد شوید یا ثبت نام کنید ]{n=1}^\infty\frac{1}{10^{n!}}<\sum_{n=1}^\infty\frac{1}{10^{n}}=\frac{1}{9}

مي‌دانيم كه بسط اعشاري يك عدد گنگ، تناوب ندارد. حال مجموع مورد سوال را در نظر مي‌گيريم. فرض كنيد بخواهيم بسط اعشاري آن را بنويسيم. بايد در مكان اول، در مكان دوم، در مكان 6ام و ... و در مكان [ برای مشاهده لینک ، لطفا با نام کاربری خود وارد شوید یا ثبت نام کنید ]!ام و ... عدد يك را قرار دهيم. فاصله مكان يكِ n ام تا فاصله‌ي مكان يكِ n+1 ام برابر n+1 است و از اين رو اين بسط اعشاري هيچ تناوبي ندارد.



موفق باشيد


ــــــــــــــــــ
22 / 07 / 88

eh_mn
14-10-2009, 13:43
فرض كنيم A يك ماتريس m در n ([ برای مشاهده لینک ، لطفا با نام کاربری خود وارد شوید یا ثبت نام کنید ]{R}^{m\times&space;n})و b يك بردار ستوني m تايي ([ برای مشاهده لینک ، لطفا با نام کاربری خود وارد شوید یا ثبت نام کنید ]{R}^{m})و همگي داراي درايه‌هاي حقيقي باشند. نشان دهيد

دستگاه [ برای مشاهده لینک ، لطفا با نام کاربری خود وارد شوید یا ثبت نام کنید ] Ax=b جواب دارد اگر و تنها اگر براي هر [ برای مشاهده لینک ، لطفا با نام کاربری خود وارد شوید یا ثبت نام کنید ] y\in\mathbb{R}^m كه [ برای مشاهده لینک ، لطفا با نام کاربری خود وارد شوید یا ثبت نام کنید ] y^TA=0 داشته باشيم [ برای مشاهده لینک ، لطفا با نام کاربری خود وارد شوید یا ثبت نام کنید ] y^Tb=0.




ـــــــــــــــــــــــ
22 / 07 / 88

CppBuilder2006
14-10-2009, 14:07
اصلا واضح نيست كه ...


بله اثبات دقیق داره ولی برای من واضحه. البته وقتی !n باشه. توجه کنید که وقتی n به اندازۀ کافی بزرگ باشه حداقل یه رقم بعد از اعشار هست که غیر صفره چون فقط یه بار جمع میشه. مجموع
[ برای مشاهده لینک ، لطفا با نام کاربری خود وارد شوید یا ثبت نام کنید ]{n=1}^\infty\sum_{i=0}^mb_i10^{i-n!}
تا حدی نمایش اعشاری (یا بهتره بگم دو دویی) رو مشخص میکنه.

mofidy1
15-10-2009, 20:17
با سلام

عبارت زیر را در اعداد حقیقی تجزیه کنید:



[ برای مشاهده لینک ، لطفا با نام کاربری خود وارد شوید یا ثبت نام کنید ](x^4&plus;x^3&plus;x^2&plus;x&plus;1)

موفق باشید.

9 مهر 1388

با سلام

ازParser که در پست 205 ([ برای مشاهده لینک ، لطفا با نام کاربری خود وارد شوید یا ثبت نام کنید ]) مساله را به درستی حل کردند، تشکر می کنم. راه حل نسبتاً ساده تری هم هست که عرض می کنم:

از x^2 که فاکتور بگیرید، به عبارت زیر می رسید:


[ برای مشاهده لینک ، لطفا با نام کاربری خود وارد شوید یا ثبت نام کنید ]^2\left&space;[&space;\left&space;(x^2&plus;\frac{1}{x^2}&space;\right&space;)&plus;&space;\left&space;(x&plus;\frac {1}{x}&space;\right&space;)&plus;&space;1&space;\right&space;]

اگر داخل پرانتز دوم را t فرض کنید و پرانتز اول را هم بر حسب t بنویسید(به وسیله ی اتحاد مربع دو جمله ای)، عبارت بالا به صورت زیر در می آید:



[ برای مشاهده لینک ، لطفا با نام کاربری خود وارد شوید یا ثبت نام کنید ]^2(t^2&plus;t-1)

اگر ریشه های چند جمله ای داخل پرانتز را به دست آورید، عبارت بالا به صورت زیر تجزیه می شود:


[ برای مشاهده لینک ، لطفا با نام کاربری خود وارد شوید یا ثبت نام کنید ]^2\left&space;(&space;t&plus;\frac{\sqrt{5}&plus;1}{2}&space;\righ t&space;)\left&space;(&space;t-\frac{\sqrt{5}-1}{2}&space;\right&space;)

حال اگر به جای t مقدارش را قرار دهید و عبارت را ساده کنیم، تجزیه ی مطلوب به صورت زیر به دست می آید:




[ برای مشاهده لینک ، لطفا با نام کاربری خود وارد شوید یا ثبت نام کنید ][&space;2x^2&plus;\left&space;(&space;\sqrt{5}&plus;1&space;\right&space;)x&plus;2&space;\right&space;]\left&space;[&space;2x^2-\left&space;(&space;\sqrt{5}-1&space;\right&space;)x&plus;2&space;\right&space;]


روش حل مساله:

یکی از روش های قدرتمند حل مسائل جبری، تغییر متغیر است که در بالا نمونه ای از آن را دیدید.

موفق باشید.

23 مهر 1388

mofidy1
15-10-2009, 21:12
با سلام

ضریب x^2 وقتی عبارت زیر را بسط می دهیم و ساده می کنیم، چیست؟


[ برای مشاهده لینک ، لطفا با نام کاربری خود وارد شوید یا ثبت نام کنید ](&space;\cdots&space;\left&space;(\left&space;(\left&space;(&space;x-2&space;\right&space;)^2-2&space;\right&space;)^2-2&space;\right&space;)^2-\cdots-&space;2&space;\right&space;)^2

موفق باشید.

23 مهر 1388

mir@
17-10-2009, 00:30
برای سه عدد حقیقی [ برای مشاهده لینک ، لطفا با نام کاربری خود وارد شوید یا ثبت نام کنید ] و تابع f پیوسته روی [ برای مشاهده لینک ، لطفا با نام کاربری خود وارد شوید یا ثبت نام کنید ] و مشتق پذیر روی[ برای مشاهده لینک ، لطفا با نام کاربری خود وارد شوید یا ثبت نام کنید ] مشتق تابع روی کل بازه مطلقا صعودی است. نشان دهید:


[ برای مشاهده لینک ، لطفا با نام کاربری خود وارد شوید یا ثبت نام کنید ]


با استفاده از قضیه مقدار میانی، اعداد حقیقی u و v وجود دارند به نحوی که

[ برای مشاهده لینک ، لطفا با نام کاربری خود وارد شوید یا ثبت نام کنید ]
[ برای مشاهده لینک ، لطفا با نام کاربری خود وارد شوید یا ثبت نام کنید ]

از آنجا که [ برای مشاهده لینک ، لطفا با نام کاربری خود وارد شوید یا ثبت نام کنید ] مشتق هم صعودی است، [ برای مشاهده لینک ، لطفا با نام کاربری خود وارد شوید یا ثبت نام کنید ] و در نتیجه

[ برای مشاهده لینک ، لطفا با نام کاربری خود وارد شوید یا ثبت نام کنید ]

که عبارت خواسته شده را نتیجه می دهد.

mir@
17-10-2009, 02:54
اگر a,b,c اضلاع یک مثلث باشند نشان دهید:


[ برای مشاهده لینک ، لطفا با نام کاربری خود وارد شوید یا ثبت نام کنید ] %7Bc%7D+%5Cfrac%7Bc%7D%7Ba%7D%29%20%5Cge%20%5Cfrac %7Ba%7D%7Bc%7D+%5Cfrac%7Bb%7D%7Ba%7D+%5Cfrac%7Bc%7 D%7Bb%7D+3

اليادران
17-10-2009, 09:42
اصلا سوال چيه جايزه چيه

ali_hp
18-10-2009, 10:44
مجموع تعدادی عدد طبیعی برابر هزار است،بیشترین مقدار ممکن برای حاصلضرب آنها چقدر است؟
فرض کنید تعدادی عدد طبیعی با مجموع هزار طوری هستند که حاصلضرب آنها بیشترین مقدار ممکن را داراست،می توان فرض کرد همه این اعداد کوچکتر مساوی سه هستند!(مثلا فرض کنید x>3 در بین این اعداد باشد،x را با 2 و x-2 جایگزین کنید،مجموع تغییری نیمکند،و به وضوح حاصلضرب نیز کاهش نمی یابد!)
حال دقت کنید که حداکثر دو تا دو در بین این اعداد وجود دارد!چون هر سه تا 2 را می توان با دو تا 3 جایگزین کرد،که مجموع ثابت می ماند،ولی حاصلضرب بیشتر می شود!همچنین واضح است که یک در بین این اعداد وجود ندارد!
با توجه به توضیحات بالا به سادگی نتیجه می شود که این اعداد عبارتن از 332 تا 3 و 2 تا 2 !

ali_hp
18-10-2009, 12:21
b و c دو عدد طبیعی هستند،به طوری که معادله

[ برای مشاهده لینک ، لطفا با نام کاربری خود وارد شوید یا ثبت نام کنید ]^2&plus;bx&plus;c=y^2

در مجموعه اعداد طبیعی نامتناهی جواب دارد،ثابت کنید:

[ برای مشاهده لینک ، لطفا با نام کاربری خود وارد شوید یا ثبت نام کنید ]^2=4c.

eh_mn
22-10-2009, 00:00
فرض كنيم A يك ماتريس m در n ([ برای مشاهده لینک ، لطفا با نام کاربری خود وارد شوید یا ثبت نام کنید ]{R}^{m\times&space;n})و b يك بردار ستوني m تايي ([ برای مشاهده لینک ، لطفا با نام کاربری خود وارد شوید یا ثبت نام کنید ]{R}^{m})و همگي داراي درايه‌هاي حقيقي باشند. نشان دهيد

دستگاه [ برای مشاهده لینک ، لطفا با نام کاربری خود وارد شوید یا ثبت نام کنید ] Ax=b جواب دارد اگر و تنها اگر براي هر [ برای مشاهده لینک ، لطفا با نام کاربری خود وارد شوید یا ثبت نام کنید ] y\in\mathbb{R}^m كه [ برای مشاهده لینک ، لطفا با نام کاربری خود وارد شوید یا ثبت نام کنید ] y^TA=0 داشته باشيم [ برای مشاهده لینک ، لطفا با نام کاربری خود وارد شوید یا ثبت نام کنید ] y^Tb=0.




ـــــــــــــــــــــــ
22 / 07 / 88

با سلام

ابتدا فرض كنيم دستگاه [ برای مشاهده لینک ، لطفا با نام کاربری خود وارد شوید یا ثبت نام کنید ] Ax=b جوابي مانند x داشته باشد و [ برای مشاهده لینک ، لطفا با نام کاربری خود وارد شوید یا ثبت نام کنید ] y^TA=0 در اين صورت


[ برای مشاهده لینک ، لطفا با نام کاربری خود وارد شوید یا ثبت نام کنید ] y^Tb=y^T(Ax)=(y^TA)x=0

اينك اگر دستگاه [ برای مشاهده لینک ، لطفا با نام کاربری خود وارد شوید یا ثبت نام کنید ] Ax=b جواب نداشته باشد آنگاه بردار b مستفل از ستون‌هاي A است. فرض كنيم رتبه‌ي ماتريس A برابر r باشد. در اين صورت رتبه‌ي ماتريس [ برای مشاهده لینک ، لطفا با نام کاربری خود وارد شوید یا ثبت نام کنید ]{pmatrix}&space;A&space;&&space;b&space;\end{pmatrix} برابر r+1 است. پس


[ برای مشاهده لینک ، لطفا با نام کاربری خود وارد شوید یا ثبت نام کنید ]{rank}\,\begin{pmatrix}&space;A&space;&&space;b&space;\\&space;0&space;&&space;-1&space;\end{pmatrix}&space;=&space;r&plus;1

در نتيجه سطر آخر تركيب خطي از ساير سطرهاست. ضرايب اين تركيب خطي بردار yاي را مي‌دهد كه در شرايط قضيه صدق مي‌‌كند. يعني [ برای مشاهده لینک ، لطفا با نام کاربری خود وارد شوید یا ثبت نام کنید ] y^TA=0 و [ برای مشاهده لینک ، لطفا با نام کاربری خود وارد شوید یا ثبت نام کنید ] y^Tb\neq 0


ـــــــــــــــــــــــــ
29 / 07 / 88

eh_mn
22-10-2009, 00:03
نشان دهيد


[ برای مشاهده لینک ، لطفا با نام کاربری خود وارد شوید یا ثبت نام کنید ]^1\int_0^1(xy)^{xy}dx\,dy=\sum_{n= 1}^\infty\frac{(-1)^{n-1}}{n^n}

راهنمايي:[ برای مشاهده لینک ، لطفا با نام کاربری خود وارد شوید یا ثبت نام کنید ]^\beta&space;=&space;e^{\alpha\ln&space;\beta }



ــــــــــــــــــــ
29 / 07 / 88

mofidy1
22-10-2009, 20:31
با سلام

ضریب x^2 وقتی عبارت زیر را بسط می دهیم و ساده می کنیم، چیست؟


[ برای مشاهده لینک ، لطفا با نام کاربری خود وارد شوید یا ثبت نام کنید ](&space;\cdots&space;\left&space;(\left&space;(\left&space;(&space;x-2&space;\right&space;)^2-2&space;\right&space;)^2-2&space;\right&space;)^2-\cdots-&space;2&space;\right&space;)^2

موفق باشید.

23 مهر 1388

با سلام

می توان دید که آخرین سه جمله پس از n بار مربع گیری، عبارت است از:


[ برای مشاهده لینک ، لطفا با نام کاربری خود وارد شوید یا ثبت نام کنید ]^2&space;&plus;&space;a_nx&space;&plus;&space;4

بنابر این


[ برای مشاهده لینک ، لطفا با نام کاربری خود وارد شوید یا ثبت نام کنید ]((b_nx^2&space;&plus;&space;a_nx&space;&plus;&space;4)&space;-&space;2)^2&space;=&space;(a_n^2&space;&plus;&space;4b_n)x^2&space;&plus;&space;4a_n&space;&plus;&space;4&space;=&space;b_{n&space;&plus;&space;1}x^ 2&space;&plus;&space;a_{n&space;&plus;&space;1}&space;&plus;&space;4

که به روابط بازگشتی زیر منجر می شود:


[ برای مشاهده لینک ، لطفا با نام کاربری خود وارد شوید یا ثبت نام کنید ]{n&space;&plus;&space;1}&space;=&space;4a_n,\,b_{n&space;&plus;&space;1}&space;=&space;a_n^2&space;&plus;&space;4 b_n\,\,(a_1&space;=&space;-&space;4&space;,\,b_1&space;=&space;1)

و لذا


[ برای مشاهده لینک ، لطفا با نام کاربری خود وارد شوید یا ثبت نام کنید ]^n,\,b_{n&space;&plus;&space;1}&space;=&space;16^n&space;&plus;&space;4b_n.

با حل رابطه ی بازگشتی بالا، می توان دید که:


[ برای مشاهده لینک ، لطفا با نام کاربری خود وارد شوید یا ثبت نام کنید ]{1}{3}4^{n&space;-&space;1}(4^n&space;-&space;1).

اگر به صفحه ی زیر سری بزنید مسائل جالبی مشاهده خواهید کرد:

journals.cms.math.ca/cgi-bin/vault/public/view/CRUXv17n1/body/HTML/2?template=CRUX



آموزش حل مساله:

در حل بعضی از مسائل، برای به دست آوردن جواب، به جواب مراحل قبلی نیازمندیم. چنین مسائلی را مسائل بازگشتی یا استقرایی می نامند. معمولاً برای حل کامل این مسائل، نیازمند معادله ای به نام معادله ی بازگشتی هستیم که معمولاً حل آن ساده نیست و احتیاج به شگردهای محاسباتی خاصی دارد.

موفق باشید.

30 مهر 1388

mofidy1
22-10-2009, 21:25
با سلام

اگر n عددی طبیعی باشد، ثابت کنید:


[ برای مشاهده لینک ، لطفا با نام کاربری خود وارد شوید یا ثبت نام کنید ]{n}{0}&plus;\binom{n}{3}&plus;\binom{n}{6}&plus;\ cdots=\frac{1}{3}\left&space;(&space;2^n&plus;2cos\frac{n\pi}{3}&space;\r ight&space;)


راهنمایی:

از ریشه ی سوم واحد، قضیه ی دو جمله ای و قانون دموآور استفاده کنید.

موفق باشید.

30 مهر 1388

CppBuilder2006
22-10-2009, 21:36
نشان دهيد


[ برای مشاهده لینک ، لطفا با نام کاربری خود وارد شوید یا ثبت نام کنید ]^1\int_0^1(xy)^{xy}dx\,dy=\sum_{n= 2}^\infty\frac{(-1)^n}{n^n}

راهنمايي:[ برای مشاهده لینک ، لطفا با نام کاربری خود وارد شوید یا ثبت نام کنید ]^\beta&space;=&space;e^{\alpha\ln&space;\beta }



ــــــــــــــــــــ
29 / 07 / 88

من با maple امتحان کردم مقدار دو طرف یکی نمیشه:


[ برای مشاهده لینک ، لطفا با نام کاربری خود وارد شوید یا ثبت نام کنید ] cmJxdqEMijGtt0C1EgiOHgOq5xxiiGq0u2rl1Inip5x5cy9g/image_thumb[6].png

البته شاید نرم افزار اشتباه کرده باشه!

eh_mn
23-10-2009, 12:44
من با maple امتحان کردم مقدار دو طرف یکی نمیشه:


[ برای مشاهده لینک ، لطفا با نام کاربری خود وارد شوید یا ثبت نام کنید ] cmJxdqEMijGtt0C1EgiOHgOq5xxiiGq0u2rl1Inip5x5cy9g/image_thumb[6].png

البته شاید نرم افزار اشتباه کرده باشه!

ممنون از تذكرتون! اصلاح شد!

ali_hp
25-10-2009, 21:38
b و c دو عدد طبیعی هستند،به طوری که معادله

[ برای مشاهده لینک ، لطفا با نام کاربری خود وارد شوید یا ثبت نام کنید ]^2&plus;bx&plus;c=y^2

در مجموعه اعداد طبیعی نامتناهی جواب دارد،ثابت کنید:

[ برای مشاهده لینک ، لطفا با نام کاربری خود وارد شوید یا ثبت نام کنید ]^2=4c.
معادله را در چهار ضرب کنید و بصورت زیر بنویسید:

[ برای مشاهده لینک ، لطفا با نام کاربری خود وارد شوید یا ثبت نام کنید ](2x&plus;b)^2&plus;4c-b^2=(2y)^2
حال از لم زیر استفاده کنید:
لم:فرض کنید a عددی مربع کامل باشد،آنگاه فاصله هر عدد مربع کامل دیگر از a حداقل برابر [ برای مشاهده لینک ، لطفا با نام کاربری خود وارد شوید یا ثبت نام کنید ]|2\sqrt{a}-1|
است.

ali_hp
25-10-2009, 21:52
یک ترازوی دیجیتال سه رقمی بر حسب گرم داریم. و یک کیسه صدتایی از گلوله های قرمز،یک کیسه صدتایی از گلوله های آبی و یک کیسه صدتایی از گلوله های سبز.می دانیم وزن گلوله های هر یک از کیسه ها عددی یک رقمی بر حسب گرم است.(وزن گلوله های داخل یک کیسه با هم برابر است.
با یک بار استفاده از ترازو وزن گلوله های هر کیسه را مشخص کنید!

eh_mn
28-10-2009, 19:25
نشان دهيد


[ برای مشاهده لینک ، لطفا با نام کاربری خود وارد شوید یا ثبت نام کنید ]^1\int_0^1(xy)^{xy}dx\,dy=\sum_{n= 1}^\infty\frac{(-1)^{n-1}}{n^n}

راهنمايي:[ برای مشاهده لینک ، لطفا با نام کاربری خود وارد شوید یا ثبت نام کنید ]^\beta&space;=&space;e^{\alpha\ln&space;\beta }



ــــــــــــــــــــ
29 / 07 / 88

با سلام.

پيش از هر چيز به دو نكته توجه كنيد:
1) چون

[ برای مشاهده لینک ، لطفا با نام کاربری خود وارد شوید یا ثبت نام کنید ]{d}{dx}x^x=(1&plus;\ln&space;x)x^x
بنابراين

[ برای مشاهده لینک ، لطفا با نام کاربری خود وارد شوید یا ثبت نام کنید ]^1(1&plus;\ln&space;x)x^x\,dx=\left.x^x\right]_0^1=0
در نتيجه

[ برای مشاهده لینک ، لطفا با نام کاربری خود وارد شوید یا ثبت نام کنید ]{\displaystyle\int_0^1x^x\ln&space;x\,dx=-\int_0^1x^x\,dx}

2) براي محاسبه انتگرال سمت راست تساوي اخير با توجه به راهنماي و با استفاده از استقرا مي‌توان نوشت


[ برای مشاهده لینک ، لطفا با نام کاربری خود وارد شوید یا ثبت نام کنید ]^1x^x\,dx=\int_0^1e^{x\ln\,x}\,d x=\int_0^1\sum_{n=0}^\infty\frac{(x\ln x)^n}{n!}=\sum_{n=0}^\infty\int_0^1\frac{(x\ln x)^n}{n!}\,dx=\sum_{n=1}^\infty \frac{(-1)^{n-1}}{n^n}

و اينك مسأله‌ي اصلي!!
ابتدا از تغيير متغير [ برای مشاهده لینک ، لطفا با نام کاربری خود وارد شوید یا ثبت نام کنید ] استفاده كنيد و سپس جاي انتگرال‌ها را (با قواعد مربوط به خودش) تغيير دهيد تا نتيجه مطلوب با اندكي محاسبه حاصل شود.


ــــــــــــــــــــ
06 / 08 / 88

eh_mn
28-10-2009, 19:28
نشان دهيد يك عدد طبيعي n را مي‌توان به صورت مجموع دو مربع كامل نوشت اگر و فقط اگر عدد 2n را بتوان به صورت مجموع دو مربع كامل نوشت.


ـــــــــــــــــــ
06 / 08 / 88

mofidy1
29-10-2009, 19:27
با سلام

اگر n عددی طبیعی باشد، ثابت کنید:


[ برای مشاهده لینک ، لطفا با نام کاربری خود وارد شوید یا ثبت نام کنید ]{n}{0}&plus;\binom{n}{3}&plus;\binom{n}{6}&plus;\ cdots=\frac{1}{3}\left&space;(&space;2^n&plus;2cos\frac{n\pi}{3}&space;\r ight&space;)


راهنمایی:

از ریشه ی سوم واحد، قضیه ی دو جمله ای و قانون دموآور استفاده کنید.

موفق باشید.

30 مهر 1388

با سلام

سمت چپ را S بنامید و w را ریشه ی سوم مناسب واحد فرض کنید. در این صورت


[ برای مشاهده لینک ، لطفا با نام کاربری خود وارد شوید یا ثبت نام کنید ]^{\frac{1}{3}n\pi&space;i}=(1&plus;w)^n=S&plus;w\left&space;\ {&space;\binom{n}{1}&plus;\binom{n}{4}&space;&plus;\cdots\right&space;\}&plus;w^2\l eft&space;\{&space;\binom{n}{2}&plus;\binom{n}{5}&space;&plus;\cdots&space;\right&space; \}

قسمت حقیقی عبارت است از


[ برای مشاهده لینک ، لطفا با نام کاربری خود وارد شوید یا ثبت نام کنید ](\frac{n\pi}{3})=S-\frac{1}{2}\left&space;\{&space;\binom{n}{1}&space;&plus;\binom{n}{2}&plus;\bi nom{n}{4}&plus;\binom{n}{5}&plus;\cdots\right&space;\}=S-\frac{1}{2}\left&space;(&space;2^n-S&space;\right&space;)

S را از معادله ی بالا بیابید.


آموزش حل مساله:

مساله ی بالا کاربرد دیگری از اعداد مختلط برای حل مسائل حقیقی است.

موفق باشید.

7 آبان 1388

mofidy1
29-10-2009, 22:51
با سلام

الف) اعداد طبیعی از 1 تا 10 به توان 10 را در نظر بگیرید. از میان این اعداد، آن هایی که رقم 1 دارند بیش ترند یا آن هایی که رقم 1 ندارند؟

ب) در صورتی که اعداد طبیعی 1 تا 222222222 به دنبال هم نوشته شوند، چند صفر نوشته ایم؟

موفق باشید.

7 آبان 1388


میلاد مولانا علی ابن موسی الرضا - علیه الصلاة و السلام- مبارک باد
هشت هشت هشتاد و هشت

Iron
30-10-2009, 11:40
سلام


آسوناشو سوا کردم:31:


نشان دهيد يك عدد طبيعي n را مي‌توان به صورت مجموع دو مربع كامل نوشت اگر و فقط اگر عدد 2n را بتوان به صورت مجموع دو مربع كامل نوشت.


ـــــــــــــــــــ
06 / 08 / 88

[ برای مشاهده لینک ، لطفا با نام کاربری خود وارد شوید یا ثبت نام کنید ]^{2}&plus;b^{2}\Leftrightarrow&space;2n=2a^{2}&plus;2 b^{2}=2a^{2}&plus;2b^{2}-2ab&plus;2ab=(a^{2}&plus;b^{2}&plus;2ab)&plus;(a^{2}&plus;b^{2}-2ab)=(a&plus;b)^{2}&plus;(a-b)^{2}


یک ترازوی دیجیتال سه رقمی بر حسب گرم داریم. و یک کیسه صدتایی از گلوله های قرمز،یک کیسه صدتایی از گلوله های آبی و یک کیسه صدتایی از گلوله های سبز.می دانیم وزن گلوله های هر یک از کیسه ها عددی یک رقمی بر حسب گرم است.(وزن گلوله های داخل یک کیسه با هم برابر است.
با یک بار استفاده از ترازو وزن گلوله های هر کیسه را مشخص کنید!

یک گلوله قرمز و ده گلوله آبی رو میندازیم تو کیسه گلوله های سبز و وزنش می کنیم. رقم اول از راست میشه وزن گلوله قرمز، رقم دوم وزن گلوله آبی و رقم آخر هم وزن گلوله سبز.

eh_mn
30-10-2009, 12:57
سلام


آسوناشو سوا کردم:31:



[ برای مشاهده لینک ، لطفا با نام کاربری خود وارد شوید یا ثبت نام کنید ]^{2}&plus;b^{2}\Leftrightarrow&space;2n=2a^{2}&plus;2 b^{2}=2a^{2}&plus;2b^{2}-2ab&plus;2ab=(a^{2}&plus;b^{2}&plus;2ab)&plus;(a^{2}&plus;b^{2}-2ab)=(a&plus;b)^{2}&plus;(a-b)^{2}




سلام

اگر 2n مجموع دو مربع كامل باشد آيا لزومي دارد كه [ برای مشاهده لینک ، لطفا با نام کاربری خود وارد شوید یا ثبت نام کنید ]^{2}&plus;+2b^2؟ چرا؟

ali_hp
01-11-2009, 16:56
یک ترازوی دیجیتال سه رقمی بر حسب گرم داریم. و یک کیسه صدتایی از گلوله های قرمز،یک کیسه صدتایی از گلوله های آبی و یک کیسه صدتایی از گلوله های سبز.می دانیم وزن گلوله های هر یک از کیسه ها عددی یک رقمی بر حسب گرم است.(وزن گلوله های داخل یک کیسه با هم برابر است.
با یک بار استفاده از ترازو وزن گلوله های هر کیسه را مشخص کنید!
سلام
Iron عزیز به درستی مساله رو حل کردند.
یک گلوله قرمز و ده تا آبی و صدتا سبزو روی ترازو میزاریم،اگر وزن گلوله های قرمز a ،و آبی ها b و سبز ها c باشد،ترازو عدد 100c+10b+a را نشان می دهد،که با توجه به اینکه a,b,c بین صفر تا نه هستند نتیجه می شود که رقم اول(یکان) عددی که ترازو نشان می دهد برابر a است و رقم دوم(دهگان) برابر b و رقم سوم(صدگان) برابر c است.

ali_hp
01-11-2009, 17:06
فرض کنید [ برای مشاهده لینک ، لطفا با نام کاربری خود وارد شوید یا ثبت نام کنید ]{R}\rightarrow\mathbb{R} پیوسته و برای هر x حقیقی داشته باشیم:[ برای مشاهده لینک ، لطفا با نام کاربری خود وارد شوید یا ثبت نام کنید ](x)=f(x^3).

ثابت کنید f تابعی ثابت است.

(سطح سوال:سوم دبیرستان و بالاتر!)

davy jones
01-11-2009, 17:34
فرض کنید [ برای مشاهده لینک ، لطفا با نام کاربری خود وارد شوید یا ثبت نام کنید ] %7BR%7D پیوسته و برای هر x حقیقی داشته باشیم:[ برای مشاهده لینک ، لطفا با نام کاربری خود وارد شوید یا ثبت نام کنید ]

ثابت کنید f تابعی ثابت است.

(سطح سوال:سوم دبیرستان و بالاتر!)

ببخشيد يه سوال داشتم: تابع f ميتونه يك به يك باشه؟ آخه اگه باشه سوال خيلي راحت حل ميشه.:27:

ali_hp
01-11-2009, 17:53
ببخشيد يه سوال داشتم: تابع f ميتونه يك به يك باشه؟ آخه اگه باشه سوال خيلي راحت حل ميشه.:27:
توی فرض سوال نیومده که f یک به یکه،پس نمی تونیم از یک به یک بودنش استفاده کنیم....
باید تا میتونید از پیوستگی و خاصیت عجیبی که f داره استفاده کنید و ثابت کنید تابع ثابته!

Iron
02-11-2009, 20:41
سلام

اگر 2n مجموع دو مربع كامل باشد آيا لزومي دارد كه [ برای مشاهده لینک ، لطفا با نام کاربری خود وارد شوید یا ثبت نام کنید ]^{2}&plus;+2b^2؟ چرا؟

تکمیل می کنم.
اگر

[ برای مشاهده لینک ، لطفا با نام کاربری خود وارد شوید یا ثبت نام کنید ]^{2}&plus;y^{2}

آنگاه

[ برای مشاهده لینک ، لطفا با نام کاربری خود وارد شوید یا ثبت نام کنید ](a&plus;b)^{2}&plus;(a-b)^{2}

که

[ برای مشاهده لینک ، لطفا با نام کاربری خود وارد شوید یا ثبت نام کنید ]{x&plus;y}{2}

[ برای مشاهده لینک ، لطفا با نام کاربری خود وارد شوید یا ثبت نام کنید ]{x-y}{2}

از اونجا که 2n عدد زوج هست، [ برای مشاهده لینک ، لطفا با نام کاربری خود وارد شوید یا ثبت نام کنید ]^{2} و [ برای مشاهده لینک ، لطفا با نام کاربری خود وارد شوید یا ثبت نام کنید ]^{2} هردو زوج یا هردو فردند و بنابراین x و y نیز هردو زوجند یا هردو فردند، پس a و b معرفی شده در بالا اعداد صحیح خواهند بود.

بقیه اثبات همون رابطه برگشت در پست قبلیه.

eh_mn
04-11-2009, 10:09
نشان دهيد يك عدد طبيعي n را مي‌توان به صورت مجموع دو مربع كامل نوشت اگر و فقط اگر عدد 2n را بتوان به صورت مجموع دو مربع كامل نوشت.


ـــــــــــــــــــ
06 / 08 / 88


با سلام

Iron زحمت كشيدن و در

برای مشاهده محتوا ، لطفا وارد شوید یا ثبت نام کنید
راه حل كامل رو ارائه دادن. با تشكر از ايشان.

ـــــــــــــــــــــــ
13 / 08 / 88

eh_mn
04-11-2009, 10:14
نشان دهيد عدد [ برای مشاهده لینک ، لطفا با نام کاربری خود وارد شوید یا ثبت نام کنید ]^{5555}&plus;5555^{2222} بر 7 بخش‌پذير است.



ـــــــــــــــــــــ
13 / 08 / 88

Mahmood_N
05-11-2009, 01:14
این پست اشتباه فرستاده شد ، چرا نمی شه پست رو حذف کرد ؟!

...

only4u-m
05-11-2009, 13:55
با استفاده از همنهشتی اثبات می کنم.

5555 و 2222 را به پیمانه 7 کاهش می دهیم:

[ برای مشاهده لینک ، لطفا با نام کاربری خود وارد شوید یا ثبت نام کنید ]
از طرفی داریم:

[ برای مشاهده لینک ، لطفا با نام کاربری خود وارد شوید یا ثبت نام کنید ]
حالا 5555 و 2222 را به شکل 3k+r می نویسیم (هر دو را بر سه تقسیم می کنیم - الگوریتم تقسیم -):

[ برای مشاهده لینک ، لطفا با نام کاربری خود وارد شوید یا ثبت نام کنید ]
بنابراین:

[ برای مشاهده لینک ، لطفا با نام کاربری خود وارد شوید یا ثبت نام کنید ]
پس عدد مورد نظر بر 7 بخشپذیر است.

mir@
07-11-2009, 02:34
مسئله شنبه چهاردهم

نشان دهید اگر یک خودرو از حالت سکون به راه افتد و فاصله یک مایل را در 1 دقیقه طی کند و در نهایت بایستد و حداکثر سرعتش 90 مایل بر ثانیه باشد، حد اقل در یک نقطه با شتاب تند شونده یا کند شونده 6.6 فوت بر مجذور ثانیه حرکت کرده است.
توجه: یک مایل 5280 فوت است.

حل مسئله شنبه سیزدهم گم شده است!!

CppBuilder2006
08-11-2009, 00:16
فرض کنید [ برای مشاهده لینک ، لطفا با نام کاربری خود وارد شوید یا ثبت نام کنید ]{R}\rightarrow\mathbb{R} پیوسته و برای هر x حقیقی داشته باشیم:[ برای مشاهده لینک ، لطفا با نام کاربری خود وارد شوید یا ثبت نام کنید ](x)=f(x^3).

ثابت کنید f تابعی ثابت است.

(سطح سوال:سوم دبیرستان و بالاتر!)

اگر a صفر نباشد، چون f پیوسته است،f(a^(1/3^n))o به f(1)o میل میکنه.
از طرفی
f(a^(1/3^n))=f(a)o
پس
f(a)= f(1)o
برای صفر هم از پیوستگی نتیجه میشه f(0)=f(1)o.

ali_hp
08-11-2009, 13:44
فرض کنید [ برای مشاهده لینک ، لطفا با نام کاربری خود وارد شوید یا ثبت نام کنید ]{R}\rightarrow\mathbb{R} پیوسته و برای هر x حقیقی داشته باشیم:[ برای مشاهده لینک ، لطفا با نام کاربری خود وارد شوید یا ثبت نام کنید ](x)=f(x^3).

ثابت کنید f تابعی ثابت است.

(سطح سوال:سوم دبیرستان و بالاتر!)

با تشکر از CppBuilder2006 عزیز که در پست 247 مساله رو حل کردند،ایده حل مساله دقیقا همونیه که دوستمون گفتن،من فقط یکم دقیقتر و کاملترشو میگم:
حل:
ابتدا دقت کنید که برای هر a دلخواه داریم:

[ برای مشاهده لینک ، لطفا با نام کاربری خود وارد شوید یا ثبت نام کنید ](a)=f(a^3)=f(a^{3^2})=...=f(a^{3^n})\\&space; f(a)=f(a^\frac{1}{3})=f(a^{(\frac{1}{3})^2})=...=f (a^{(\frac{1}{3})^n})

برای هر a>0 باتوجه به پیوستگیf و با توجه به اینکه [ برای مشاهده لینک ، لطفا با نام کاربری خود وارد شوید یا ثبت نام کنید ]{n\rightarrow\infty}{a^{(\frac{1}{3 })^n}}=1 داریم:

[ برای مشاهده لینک ، لطفا با نام کاربری خود وارد شوید یا ثبت نام کنید ](a)=f(1)

با استدلال مشابه برای هر a<0 بدست می آید:

[ برای مشاهده لینک ، لطفا با نام کاربری خود وارد شوید یا ثبت نام کنید ](a)=f(-1)

از طرفی برای هر a که [ برای مشاهده لینک ، لطفا با نام کاربری خود وارد شوید یا ثبت نام کنید ]|a|<1 با توجه به پیوستگی f و با توجه به اینکه [ برای مشاهده لینک ، لطفا با نام کاربری خود وارد شوید یا ثبت نام کنید ]{n\rightarrow\infty}{a^{3^n}}=0

داریم:

[ برای مشاهده لینک ، لطفا با نام کاربری خود وارد شوید یا ثبت نام کنید ](a)=f(0)

با توجه به رابطه های بدست آمده برای f به سادگی نتیجه می شود که f در سرتا سر اعداد حقیقی تابعی ثابت است.و برای هر a داریم:

[ برای مشاهده لینک ، لطفا با نام کاربری خود وارد شوید یا ثبت نام کنید ](a)=f(1)=f(0)=f(-1).

ali_hp
08-11-2009, 15:33
تابع زیر را در نظر بگیرید:

[ برای مشاهده لینک ، لطفا با نام کاربری خود وارد شوید یا ثبت نام کنید ](m)=(2m-1)([\frac{(2m)!&plus;1}{2m&plus;1}]&plus;[&space;\frac{(2m)!&plus;1}{-2m-1}]&plus;1)&plus;2

نشان دهید این تابع همه اعداد اول را تولید می کند،و فقط هم اعداد اول را تولید می کند.
سطح سوال:همه کسانی که آشنایی خیلی مقدماتی با نظریه اعداد دارند!
توجه: [x] یعنی بزرگنرین عدد صحیح کوچکتر یا مساوی x

eh_mn
11-11-2009, 23:08
نشان دهيد عدد [ برای مشاهده لینک ، لطفا با نام کاربری خود وارد شوید یا ثبت نام کنید ]^{5555}&plus;5555^{2222} بر 7 بخش‌پذير است.



ـــــــــــــــــــــ
13 / 08 / 88

سلام.

ضمن تشكر از only4u-m كه در

برای مشاهده محتوا ، لطفا وارد شوید یا ثبت نام کنید
يك راه حل مقدماتي و زيبا براي اين مسأله نوشتن، راه ديگري را با هم مي‌بينيم:
مي‌دانيم كه اگر n فرد باشد آنگاه [ برای مشاهده لینک ، لطفا با نام کاربری خود وارد شوید یا ثبت نام کنید ]^n&plus;b^n بر [ برای مشاهده لینک ، لطفا با نام کاربری خود وارد شوید یا ثبت نام کنید ] بخش‌پذير است. همچنين براي هر n طبيعي [ برای مشاهده لینک ، لطفا با نام کاربری خود وارد شوید یا ثبت نام کنید ]^n-b^n بر [ برای مشاهده لینک ، لطفا با نام کاربری خود وارد شوید یا ثبت نام کنید ]
بخش‌پذير است. اينك داريم
داريم

[ برای مشاهده لینک ، لطفا با نام کاربری خود وارد شوید یا ثبت نام کنید ]^{2222}&plus;2222^{55555}=(55555^{2222}-4^{2222})&space;&plus;&space;(2222^{5555}&plus;4^{5555})-(4^{5555}-4^{2222})
پرانتز اول و دوم با استفاده از نكات گفته شده بر 7 بخش‌پذيرند. براي پرانتز سوم داريم

[ برای مشاهده لینک ، لطفا با نام کاربری خود وارد شوید یا ثبت نام کنید ](4^{5555}-4^{2222})=4^{2222}(4^{3333}-1)=4^{2222}(64^{1111}-1)
كه بنا به دليل مشابه، بر 63 بخش‌پذير است.


ـــــــــــــــــــــــ
20 / 08 / 88

منبع :

D.O.S hklarskyN, .N .C hent zov, and l.M.Yaglo, The USSR Olympiad Problem Book, DOVER Publications Inc.